Sunteți pe pagina 1din 172

Programa Olmpico de Treinamento

Aula

Curso de Geometria - Nvel 3

Prof. Rodrigo Pinheiro

Quadril
ateros Inscritveis
Um quadrilatero e dito inscritvel se, e somente se, existe uma circunferencia que passa
pelos seus quatro vertices.
Teorema 1. Um quadrilatero e inscritvel se, e somente se, tiver a soma de dois angulos
opostos iguais a 180 .
Demonstrac
ao.
() Seja ABCD um quadrilatero inscritvel em uma circunferencia de centro O.
C
b

B
b

O
b

A
b

BCD
BAD
Pela propriedade de
angulo central, temos que: BAD =
e BCD =
.
2
2

Como BCD + BAD= 360 , temos entao que:


BAD + BCD = 180
() Seja ABCD um quadrilatero onde BAC +BCD = 180 . E suponha que ABCD
nao e inscritvel. Construmos a circunferencia circunscrita ao tri
angulo ABD, e definimos
o ponto E a intersecao da reta CD com a circunferencia (ver figura). Observe que este
ponto pode ser exterior ou interior (mostrado na figura) `a circunferencia. Como ABED e
um quadrilatero inscritvel, ja demonstramos que BAD + BED = 180 , conclumos que
BED = BCD. Mas isso e um absurdo, pois em um tri
angulo o angulo externo tem que
ser sempre maior que os outros dois nao adjacentes a ele, o que nao acontece no BEC.

POT 2012 - Geometria - Nvel 2 - Aula 1 - Prof. Onofre Campos/Rodrigo


Pinheiro
B
b

A
b

O
b

b
b

Problema 1. Seja ABC um tri


angulo ret
angulo em C e CD a bissetriz interna de C. A
perpendicular a AB por D encontra BC em E. Mostre que AD=DE.
Soluc
ao. No quadrilatero ADEC, observe que ADE + ACE = 180 . Logo, ADEC e
inscritvel, de modo que AED = EAD = 45 . Entao o tri
angulo ADE e isosceles e,
portanto, AD = DE.
C
b

E
b

O
b

Teorema 2. Um quadrilatero ABCD e inscritvel se, e somente se, o angulo que uma
diagonal forma com um lado for igual ao angulo que a outra diagonal forma com o lado
oposto.
Demonstrac
ao. Digamos ABD = ACD. Assuma que a circunferencia passa pelos
pontos A, B, C e nao contem D. Perceba que D pode estar no interiro da circunferencia
ou no exterior, faremos a solucao para o caso em que ele est
a na regi
ao interna, o outro caso
e an
alogo. Entao, supondo que D pertence `a regi
ao interior da circunferencia, tomamos
E como sendo a intersecao da reta CD com a circunferencia. Pela propriedade de angulo
inscrito, temos ABE = ACE, mas pela condicao ABD = ACD, temos que ABD =
ABE, entao D pertence `
a circunferencia.

POT 2012 - Geometria - Nvel 2 - Aula 1 - Prof. Onofre Campos/Rodrigo


Pinheiro

C
b

B
b

D
b

b
b

Problema 2. (Extrado de [1]) As alturas AD e BE do tri


angulo ABC se encontram no
ortocentro H. Os pontos medios de AB e CH s
ao X e Y , respectivamente. Prove que XY
e perpendicular a DE.
Soluc
ao.
A

E
X
H
Y
C

Observe que os quadrilateros CDHE e ABDE s


ao inscritveis pois, HDC +HEC =

+ 90 = 180 e ADB = AEB. Como em um tri


angulo ret
angulo, o ponto medio
da hipotenusa e seu circuncentro, entao X e Y s
ao os circuncentros das circunferencias
circunscritas aos quadrilateros ABDE e CDHE, respectivamente. Portanto
90

XD = XE

YD =YE

XDY XEY

(caso LLL),

isto implica dizer que o tri


angulo EXD e isosceles e XY e bissetriz em relacao `a base,
entao tambem e altura e portanto XY e perpendicular a DE.
Problema 3. (Extrado de [1])Seja ABC um tri
angulo tal que BAC = 60o . Dado um
ponto D sobre BC, sejam O1 e O2 os circuncentros dos tri
angulos ABD e ACD, respectivamente, M a intersecao de BO1 e CO2 e N o circuncrculo de DO1 O2 . Mostre que, ao

POT 2012 - Geometria - Nvel 2 - Aula 1 - Prof. Onofre Campos/Rodrigo


Pinheiro

variar D, M N passa por um ponto fixo.


Soluc
ao.
Seja o circuncrculo do tri
angulo ABC. Afirmamos que M . De fato,
BM C = 180 CBM BCM
= (90 DBO1 ) + (90 DCO2 )
= BAD + DAC
= BAC.
Isso garante tambem que os pontos M , O1 , N e O2 s
ao concclicos, pois
O1 M O2 + O1 N O2 = BM C + 2O1 DO2
= BM C + 2(180 BDO1 CDO2 )
= BM C + 2(180 DBO1 DCO2 )
= BM C + 2BM C
= 180 .
Mas entao M N e a bissetriz do
angulo BM C, uma vez que
BM N
e CM N

= O1 M N
= O2 M N

= 30
= 30 .

= O1 O2 N
= O2 O1 N

Assim, M N passa pelo ponto medio do arco BC da circunferencia .


A

O2
b

O1
b
b

B
b

Problema 4. (Extrado de [1]) Mostre que todo tri


angulo acutangulo ABC possui um ponto
P em seu interior tal que os pes das perpendiculares baixadas de P aos lados de ABC s
ao
os vertices de um tri
angulo equil
atero.
Soluc
ao.
4

POT 2012 - Geometria - Nvel 2 - Aula 1 - Prof. Onofre Campos/Rodrigo


Pinheiro

A
b

F
b

P
b

b
b

Sejam D, E, F os pes das perpendiculares baixadas de P aos lados BC, CA, AB, respectivamente. Vamos encontrar condicoes necessarias e suficientes para que o tri
angulo DEF
seja equil
atero.
Necessidade. Suponha DEF equil
atero. Temos
BP C = BAC + P BA + P CA
= BAC + P BF + P CE
= BAC + P DF + P DE
= BAC + 60 ,
onde na pen
ultima passagem utilizamos que os quadrilateros BDP F e CEP D s
ao cclicos.
De maneira an
aloga, temos
AP B = ACB + 60

CP A = CBA + 60

e portanto P e a intersecao de tres circunferencias (especificamente, tres arcos capazes),


que se intersectam em um ponto, pois
(BAC + 60 ) + (ACB + 60 ) + (CBA + 60 ) = 360 .
Sufici
encia. Seja P a intersecao dos tres arcos capazes descritos acima. Pelas mesmas
razoes, temos
F DE = F DP + EDP
= F BP + ECP
= BP C BAC
= 60 .
Isso conclui a prova.
5

POT 2012 - Geometria - Nvel 2 - Aula 1 - Prof. Onofre Campos/Rodrigo


Pinheiro

Problema 5. (Extrado de [1])Sejam ABC um tri


angulo de circuncrculo e incentro I.
Se M e o ponto medio do arco menor BC de , Prove que:
M B = M I = M C.
Soluc
ao.

C
M

Temos
M BI = M BC + CBI
= M AC + CBI
=

BAC + ABC
2

e
M IB = BAI + ABI
=

BAC + ABC
,
2

de modo que M B = M I. Analogamente, M C = M I.


Problema 6. (Extrado de [1]) Seja ABC um tri
angulo com todos os seus angulos agudos,
de alturas AD, BE e CF (com D em BC, E em AC e F em AB). Seja M o ponto medio
do segmento BC. A circunferencia circunscrita ao tri
angulo AEF corta a reta AM em A
e X. A reta AM corta a reta CF em Y . Seja Z o ponto de encontro entre as retas AD e
BX.
Demonstrar que as retas Y Z e BC s
ao paralelas.
Soluc
ao. Note inicialmente que AF HE e inscritvel, pois AEH = AF H = 90o . Da,
AXH = 90o .
6

POT 2012 - Geometria - Nvel 2 - Aula 1 - Prof. Onofre Campos/Rodrigo


Pinheiro

Seja A o ponto sobre a semi-reta AM tal que AM = M A . O quadrilatero ABA C e


um paralelogramo, donde
A BH = A BC + CBH
= ACB + (90 ACB)
= 90
= A XH,
ou seja, o quadrilatero BHXA e inscritvel. Mas BHCA tambem e inscritvel, ja que
BHC + BA C = (180 BAC) + BAC = 180 .
Dessa forma, os pontos B, H, X, C s
ao concclicos, donde
XBM = XBC = XA C = BAM

(1)

Seja T a intersecao entre as retas AB e XH. Note que H e tambem o ortocentro do


tri
angulo AT Y , uma vez que T X AY e Y F AT . Da,
AH T Y

= T Y k BC.

Se mostrarmos que Z T Y , o problema estara terminado. Seja entao Z intersecao entre


AD e T Y . Vamos mostrar que Z = Z . Ora, como
HZ Y = HXY = 90 ,
o quadrilatero HXY Z e inscritvel e portanto
XZ Y = XHY = F AX = BAM.
Das relacoes (1) e (2), segue que
XZ Y = XBM,
ou seja, que os pontos B, Z e X s
ao colineares. Mas entao
Z AD BX = Z
como queramos.

= Z = Z ,

(2)

POT 2012 - Geometria - Nvel 2 - Aula 1 - Prof. Onofre Campos/Rodrigo


Pinheiro
A

T
Z

POT 2012 - Geometria - Nvel 2 - Aula 1 - Prof. Onofre Campos/Rodrigo


Pinheiro

Problemas Propostos
Problema 7. Seja P o centro do quadrado construdo externamente sobre a hipotenusa AC
do tri
angulo ret
angulo ABC. Prove que BP bissecta o angulo ABC.
Problema 8. (IME-97) Quatro retas se interceptam formando quatro tri
angulos conforme
figura abaixo. Prove que os crculos circunscritos aos quatro tri
angulos possuem um ponto
em comum.
b

Problema 9. (IME-93) Seja ABCD um quadrilatero convexo inscrito num crculo e seja I
o ponto de interseccao de suas diagonais. As projecoes ortogonais de I sobre os lados AB,
BC, CD e DA s
ao, respectivamente, M , N , P e Q. Prove que o quadrilatero M N P Q e
circuscritvel a um crculo com centro em I.
Problema 10. No tri
angulo ABC, C = 90 . Seja O o seu circuncentro e AH a altura
relativa ao lado BC. Sabendo que BAH = 10 , calcule BOH.
Problema 11. Num tri
angulo ABC, BAC = 100 , AB = AC. Um ponto D e escolhido
sobre o lado AC de tal modo que ABD = CBD. Prove que AD + BD = BC.
Problema 12. Sobre os lados AB, BC e AC do tri
angulo ABC, respectivamente, escolhemos M , N e P quaisquer. Mostre que as circunferencias circunscritas aos tri
angulos AM P ,
BN M e CP N concorrem em um ponto O comum `as tres circunferencias.
Problema 13. Sejam E e F pontos sobre os lados AB e BC, respectivamente, do quadrado
ABCD tais que EB = CF . Se EDF = 27 , determine a soma F AB + ECB.
Problema 14. (Leningrado) No quadrilatero ABCD, AB = CD. Sejam M e N os pontos
medios dos lados AB e CD, respectivamente e P o ponto de encontro das mediatrizes de
BC e AD. Prove que P tambem esta sobre a mediatriz de M N .
Problema 15. Seja ABC um tri
angulo tal que BAC = 60 . Mostre que o circuncentro,
ortocentro, incentro de ABC, ex-incentro de ABC relativo ao lado BC e os pontos B, C
s
ao concclicos.

Problema 16. (Sao Petersburgo 1996) Seja ABC um tri


angulo tal que BAC = 60 . Seja
tambem O um ponto no interior de ABC para o qual AOB = BOC = 120 . Se D, E
s
ao os pontos medios dos lados AB, AC, prove que A, D, E, O s
ao concclicos.
Problema 17. (Teste Cone Sul Brasil 2005) Seja P um ponto do arco menor AB da circunferencia circunscrita ao quadrado ABCD. Os segmentos AC e P D se intersectam em
Q e AB e P C em R. Mostre que QR e a bissetriz do angulo P QB.
Problema 18. (IMO 2002) Sejam S uma circunferencia de centro O, BC um di
ametro de
S, A um ponto sobre S tal que AOB < 120 e D o ponto medio do arco AB que nao
contem C. Se a reta paralela a DA passando por O intersecta AC em I e a mediatriz de
OA intersecta S em E e F , prove que I e o incentro do tri
angulo CEF .
Problema 19. (Irlanda 1997) Dado um ponto M interior ao tri
angulo equilatero ABC,
sejam D, E e F os pes das perpendiculares tracadas de M aos lados BC, CA e AB, respectivamente. Encontre o lugar geometrico dos pontos M para os quais F DE = 90 .
Problema 20. (R
ussia 1999) No tri
angulo ABC de circuncrculo , pontos D e E s
ao
escolhidos sobre o segmento AC de modo que
AB = AD e BE = EC,
com E entre A e D. Se F e o ponto medio do arco BC de , mostre que os pontos B, D,
E, F s
ao concclicos.
Problema 21. Seja ABC um tri
angulo de circuncentro O e ortocentro H tal que BAC =
60 e AB > AC. Sejam tambem BE, CF as alturas relativas aos lados CA, AB, respectivamente, e M , N pontos sobre os segmentos BH, HF , respectivamente, tais que BM = CN .
Determine o valor da expressao
HM + HN

HO
Problema 22. Num tri
angulo ABC, tomamos pontos X, Y sobre os lados AB, BC, respectivamente. Se AY e CX se intersectam em Z e
AY = Y C

e AB = ZC,

mostre que os pontos B, X, Z, Y s


ao concclicos.

Refer
encias
[1] S. B. Feitosa, B. Holanda, Y. Lima and C. T. Magalhaes, Treinamento Cone Sul 2008.
Fortaleza, Ed. Realce, 2010.
[2] Afined, Geometra, Una vision de la planimetra. Lima, Ed. Lumbreras, 2005.

Programa Olmpico de Treinamento


Aula

Curso de Geometria - Nvel 3

Prof. Rodrigo Pinheiro

Quadril
ateros Inscritveis II
Nesta aula, trataremos de tres teoremas muito utilizados em problemas de quadrilateros
inscritveis. Os teoremas s
ao:
1. Teorema de ptolomeu (Problema 1)
2. Teorema de Simson-Wallace (Problema 12)
3. Teorema da borboleta (Problema 18)
Problema 1. (Teorema de Ptolomeu) Prove que em todo quadrilatero inscritvel, o produto
dos comprimentos de suas diagonais e igual a soma dos produtos dos comprimentos de seus
lados opostos.
Soluc
ao. Dado o quadrilatero ABCD de lados a, b, c,d e diagonais m e n, demonstraremos
que: a.c + b.d = m.n.
b

B
b

m-l
+

E
b

Tracamos BE tal que ABE = DBC = . Se AE = l EC = m l. Perceba que


os tri
angulos ABE e DBC possuem os mesmo angulos, ou seja, s
ao semelhantes, podemos
entao tirar que:
l
a
= a.c = n.l
c
n

POT 2012 - Geometria - Nvel 3 - Aula 1 - Prof. Onofre Campos/Rodrigo


Pinheiro
Da mesma forma, temos que os tri
angulos EBC e ABD s
ao semelhantes pois possuem os
mesmos angulos, portanto:
b
ml
=
b.d = n.(m l)
n
d
Somando as duas relacoes encontradas, temos que:
a.c + b.d = n.l + n.(m l) a.c + b.d = m.n
Problema 2. Seja P um ponto sobre o arco BC do crculo circunscrito ao tri
angulo
equil
atero ABC. Prove que P A = P B + P C.
Soluc
ao. Como o quadrilatero ABP C e inscritvel, pelo teorema de ptolomeu temos que:
P A.BC = P B.AC + P C.AB
So que, AB = BC = CA pois ABC e equil
atero, portanto P A = P B + P C.
Problema 3. Demonstre que em todo quadrilatero inscritvel, a razao dos comprimentos
das diagonais e igual a razao da soma dos produtos dos lados que concorrem nos extremos
de cada diagonal respectivamente.
Soluc
ao.

b
B
b

c
x
a
A

b
b

d
m
a
c
b

Dado um quadrilatero ABCD inscritvel com lados a, b, c, d, e diagonais x e y, mostraremos


que:
a.d + b.c
x
=
y
a.b + c.d

Seja M um ponto sobre a circunferencia tal que m(AM ) = m(CD), portanto AM = CD =

c. E seja N tal que m(DN ) = m(AB), portanto DN = AB = a. Assim sendo, temos


2

POT 2012 - Geometria - Nvel 3 - Aula 1 - Prof. Onofre Campos/Rodrigo


Pinheiro

tambem que BM = CN = m e M C = BN = AD = d, aplicando o teorema de ptolomeu


nos quadrilateros M ABC e N BCD, temos:
Quadrilatero M ABC: m.x = a.d + b.c
Quadrilatero N BCD: m.y = a.b + c.d
Dividindo as duas relacoes, obtemos:
x
a.d + b.c
=
y
a.b + c.d
Problema 4. Em uma circunferencia, sejam A, B, C e D pontos consecutivos tal que

m(AB) = m(BC) = m(CD) e AD = 2.AB. Calcule m(CD).


Problema 5. Em um quadrilatero ABDE, seja L um ponto em BD, tal que ABLE e um
quadrilatero inscritvel, BEA = LED, LD = 2.BL = 8, DE = 2.LE e AB.LE = 40.
Calcule AE.
Problema 6. Segundo a figura, BP.CQ = 10; P e Q s
ao pontos de tangencia. Calcule
BC.P Q.
P
b

C
b

B
A

Problema 7. Segundo a figura, m(CA) = m(AM ) = m(BD) = m(M D) = 120 ,

m(CN ) = m(N B), AB = a e CN.(CN + AB) = b. Calcule BD em funcao de a e b.


Problema 8. Demonstre que se um ponto P esta sobre o crculo circunscrito ao quadrado
ABCD, entao:
PA
PB + PD
=
PB
PA + PC
Problema 9. Um hex
agono esta inscrito em um crculo. Cinco de seus lados medem 81 e
o sexto, denotado por AB mede 31. Calcule a soma das medidas das tres diagonais que
podem ser tracadas a partir do vertica A.
Problema 10. Marcamos quatro pontos A, B, C e D sobre a borda de uma piscina circular
de raio R. As dist
ancias entre A e C e entre A e B s
ao iguais a 50m. O tempo que um
nadador leva para ir de D ate C, de D ate A e de D ate B, com a mesma velocidade
constante s
ao proporcionais a 1, 5, e 7, respectivamente. Determine R.
3

POT 2012 - Geometria - Nvel 3 - Aula 1 - Prof. Onofre Campos/Rodrigo


Pinheiro

Problema 11. (Teorema de Carnot) Seja O o circuncentro do tri


angulo ABC. Sejam Ka ,
Kb e Kc as dist
ancias de O aos lados de ABC. Prove que:
Ka + Kb + Kc = R + r
onde R e r s
ao os raios dos crculos circunscrito e inscrito ao tri
angulo.
Problema 12. (Reta de Simson-Wallace) Seja a circunferencia circunscrita ao tri
angulo
ABC e P um ponto sobre . Sejam X, Y e Z as projecoes ortogonais de P sobre as retas
suportes de BC, CA e AB, respectivamente. Entao X, Y e Z s
ao colineares, e a reta
que contem esses pontos e chamada de Reta de Simson-Walace. A recproca tambem e
verdadeira: Se as projecoes ortogonais de um ponto P sobre os lados de um tri
angulo s
ao
colineares, entao o ponto P encontra-se sobre a circunferencia circunscrita ao tri
angulo.
Soluc
ao. Observe que os quadrilateros P XCY , P Y AZ e P ZBX possuem dois angulos
retos opostos cada um, entao eles s
ao cclicos. Portanto:
P XY

= P CY (Quadril
atero c
clico PXCY)
= P CA(Colinearidade de A, C, Y )

Analogamente, P XZ = P BA. Agora, X, Y e Z s


ao colineares se, e somente se,
P XY = P XZ, onde pelas equacoes acima ocorre se, e somente se, P CA = P BA.
Em outra palavras, se, e somente se, A, B, C e P s
ao cclicos.
Problema 13. (Reta de Simson-Wallace Generalizado) Seja ABC um tri
angulo de circuncrculo . Dados um ponto D e um
angulo , sejam X, Y , Z pontos sobre as retas AB,
BC, CA tais que as retas DX, DY , DZ formam um angulo com as retas AB, BC, CA,
respectivamente, na mesma orientacao. Entao X, Y , Z s
ao colineares se, e somente se D
pertence `a circunferencia .
Problema 14. Sejam A, B, C, P , Q pontos sobre um crculo. Mostre que o angulo formado
entre as retas de Simson de P e Q em relacao ao tri
angulo ABC e igual a metade do arco
P Q.
Problema 15. Sejam A, B, C, D pontos sobre um crculo. Prove que a intersecao da reta
de Simson de A em relacao a BCD com a reta de Simson de B em relacao ACD pertence
`a reta que une C ao ortocentro de ABD.
Problema 16. Se A, B, C, P , Q s
ao cinco pontos sobre um crculo tal que P Q e um
di
ametro, mostre que as retas de Simson de P e Q em relacao `a ABC intercepta em um
ponto concclico com os pontos medios dos lados do tri
angulo ABC.
Problema 17. Seja I o incentro do tri
angulo ABC, e D, E, F as projecoes de I sbore BC,
CA, AB, respectivamente. O crculo inscrito `a ABC encontra os segmentos AI, BI, CI
nos pontos M , N , P , respectivamente. Mostre que as retas de Simson de qualquer pontos
sobre o incirculo em relacao aos tri
angulos DEF e M N P s
ao perpendiculares.

POT 2012 - Geometria - Nvel 3 - Aula 1 - Prof. Onofre Campos/Rodrigo


Pinheiro

Problema 18. (Teorema da borboleta) Dada uma circunferencia , sejam EF uma corda
de e P o ponto medio de EF . Sejam tambem AC e BD cordas de passando por P .
Se AB e CD intersectam EF em M e N , entao P e o ponto medio de M N .

B
b

P
b

b
b

M
O
b

Soluc
ao. Como EP = P F , entao a reta OP e perpendicar a EF . Tracamos AH perpendicular a OP . Portanto, AQ = QH = m, AP = P H = l e P AH = P HA = .
Como EF//AH, temos que AP E = HP F = . Ja que ACHD e inscritvel, entao
CAH = CDH = . Dessa forma, temos que o quadrilatero N P DH e inscritvel pois
HP F = N DH, assim temos que P HN = , portanto AP M HP N pelo caso
ALA. Isso implica dizer que P M = N P .

b
b

M
b

O
H
b

Problema 19. (Teorema da borboleta generalizado) Dadas uma circunferencia de centro


O e uma reta l, seja M a projecao de O sobre l. Sejam tambem r, s duas retas passando
5

POT 2012 - Geometria - Nvel 3 - Aula 1 - Prof. Onofre Campos/Rodrigo


Pinheiro

por M que definem cordas AB e CD em . Se AD e BC intersectam l em X e Y , entao


M e o ponto medio do segmento XY .

Programa Olmpico de Treinamento


Aula

Curso de Geometria - Nvel 3

Prof. Rodrigo Pinheiro

Quadril
ateros Circunscritveis
Um quadrilatero e dito circunscritvel se, e somente se, existe uma circunferencia que
tangencia internamente todos os lados do quadrilatero.
Teorema 1. (Teorema de pitot) Mostre que um quadrilatero pode ser circunscrito a uma
circunferencia se, e somente se, a soma de dois lados opostos for igual `a soma dos outros
dois lados.
Soluc
ao. () Suponha que o quadrilatero ABCD seja circunscrito a uma circunferencia,
e os pontos de tangencia da circunferencia com os lados sejam E, F , G, H, como mostra a
figura abaixo.
b

F
B

b
b

Pelo problema anterior, vemos que: AH = AE; BE = BF ; CF = CG; GD = HD.


Portanto, AE + BE + CG + GD = BF + CF + +HD + AH, isto implica dizer que:
AB + CD = BC + AD.
() Suponha que ABCD seja um quadrilatero tal que AB + CD = BC + AD e nao
seja circunscritvel.

POT 2012 - Geometria - Nvel 2 - Aula 3 - Prof. Onofre Campos/Rodrigo


Pinheiro

y
E

z+w

w-k
b

Sejam AO e BO as bissetrizes internas dos angulos DAB e ABC. Tomamos E, F e


H como sendo os pes das alturas de O aos lados AB, BC e AD, respectivamente. Pelo caso
especial de congruencia temos que AOH AOE e BOE BOF , assim sendo,
AE = AH = x e BE = BF = y. Defina CF = z e HD = w. Pela hip
otese, temos que:
(x + y) + CD = (y + z) + (x + w) CD = z + w
Como CD nao e tangente `
a circunferencia pois, caso contrario, o quadrilatero seria circunscritvel. Tomamos G tal que CG seja tangente a circunferencia e defina CG AD = I,
perceba que pelo problema anterior, temos: CG = CF = z e GI = HI = k. Dessa maneira, ID = w k, mas analisando o tri
angulo CID isso e um absurdo pois CI + ID = CD
e CID e um tri
angulo. Entao se a soma dos lados opostos de um quadrilatero forem iguais,
entao ele sera circunscritvel.
Problema 1. Seja ABCDEF um hex
agono circunscritvel a uma circunferencia. Mostre
que AB + CD + EF = AF + BC + DE.
Problema 2. Dado um quadrilatero convexo ABCD tal que AB e CD se intersectam em
P e BC e AD intersectam-se em Q. Prove que o quadrilatero ABCD e circunscritvel se,
e somente se, uma das seguintes condicoes e verdadeira:
AB + CD = BC + AD, AP + CQ = AQ + CP , BP + BQ = DP + DQ
Soluc
ao. () Seja ABCD um quadrilatero circunscritvel. Entao provaremos que todas
as condicoes s
ao validas. Sejam K, L, M e N os pontos de tangecnia do crculo inscrito
com os lados AB, BC, CD e DA. Entao
AB + CD = AK + BK + CM + DM = AN + BL + CL + DN = BC + AD
AP + CQ = AK + P K + QL CL = AN + P M + QN CM = AQ + CP
BP +BQ = AP AB+BC+CQ = (AP +CQ)+(BCAB) = AQ+CP +CDAD = DP +DQ
() Suponhamos agora que BP + BQ = DP + DQ. Tomemos o crculo que e tangente
aos lados AB, BC e CD. Assuma que a reta AD nao e tangente ao crculo. Tomamos
2

POT 2012 - Geometria - Nvel 2 - Aula 3 - Prof. Onofre Campos/Rodrigo


Pinheiro

A1 D1 paralelo a AD tal que A1 D1 e tangente ao crculo. Seja Q1 a intersecao de BC


e A1 D1 e S sobre AQ tal que Q1 S e paralelo a D1 D. Ja que BP + BQ = DP + DQ
e BP + BQ1 = D1 P + D1 Q1 , segue que QS + SQ1 = QQ1 , absurdo pela desigualdade
triangular, logo AD e tangente ao crculo tambem, portanto, ABCD e circunscritvel.

L
b

A1

N
b
b

D1

Q1

Q
b

Problema 3. (IME) Seja ABCD um quadrilatero inscrito em uma circunferencia. Seja I


o ponto de encontro das diagonais AC e BD; M , N , P e Q s
ao as projecoes ortogonais de
I sobre os lados AB, BC, CD e DA, respectivamente. Prove que o quadrilatero M N P Q
e um quadrilatero circunscritvel a uma circunferencia com centro em I.
Soluc
ao. Como ABCD e inscritvel, temos que ABD = ACD. Observe que os quadrilateros M BN I e N CP I s
ao inscritveis, pois a soma dos angulos opostos e 180 . Sendo
assim, vemos que M BI = M N I e P CI = P N I, como M BI = P CI, temos que
M N I = P N I, portanto N I e bissetriz de M N P . Analogamente, M I e bissetriz de
QM N , QI e bissetriz de M QP e P I e bissetriz de QP N , portanto, I e o ponto de
encontro das bissetrizes do quadrilatero M N P Q, porntanto este e circunscritvel.

POT 2012 - Geometria - Nvel 2 - Aula 3 - Prof. Onofre Campos/Rodrigo


Pinheiro

M
b

b
b

Problema 4. (Selecao para a Olimpada do Cone Sul - 98) No tri


angulo ABC, temos
BC = 2.AC. Seja M o ponto medio de BC. A reta passando por M e tangente `a
1
circunferencia inscrita em ABC encontra o lado AB no ponto N . Mostre que AN
AB = 3 .
Problema 5. (OIM - 1994/2) Seja ABCD um quadrilatero inscritvel. Suponha que existe
uma semicircunferencia com centro em AB, tangente aos outros tres lados do quadrilatero.
Demonstrar que AB = AD + BC.
Calcular, em funcao de x = AB e y = CD, a area m
axima que pode alcancar um
quadrilatero satisfazendo as condicoes do enunciado.
Problema 6. Seja ABCD um quadrilatero convexo. Mostre que ABCD e circunscritvel
se, e somente se, as circunferencias inscritas nos tri
angulos ABC e CDA tocam a diagonal
AC em um ponto comum.
Problema 7. Dizemos que um quadrilatero e bicentrico se ele for inscritvel e circunscritvel
simultaneamente.
Mostre que a
area de um quadrilatero bicentrico pode ser calculada por

[ABCD] = abcd, onde a, b, c e d s


ao os seus lados.
Problema 8. (Romenia - 1999) No quadrilatero convexo ABCD, as bissetrizes dos angulos
A e C encontram-se no ponto I. Mostre que ABCD e circunscritvel se, e somente se,
[AIB] + [CID] = [AID] + [BIC], onde [XY X] denota a area do tri
angulo XY Z.
Problema 9. ABCD e um quadrilatero convexo inscrito em um crculo de centro O, e com
diagonais perpendiculares. Prove que a linha quebrada AOC divide o quadrilatero em duas
partes de mesma
area.
4

POT 2012 - Geometria - Nvel 2 - Aula 3 - Prof. Onofre Campos/Rodrigo


Pinheiro

Problema 10. (USAMO - 91/5) Seja D um ponto arbitr


ario sobre o lado AB de um dado
tri
angulo ABC e seja E um ponto de intersecao do segmento CD com a tangente externa
comum aos crculos inscritos nos tri
angulos ACD e BCD. Mostre que o ponto E descreve
o arco de uma circunferencia quando D varia sobre o lado AB.

Programa Olmpico de Treinamento


Aula

Curso de Geometria - Nvel 3

Prof. Rodrigo Pinheiro

Propriedades do ortocentro
O ortocentro e o ponto de encontro das tres alturas de um tri
angulo arbitr
ario. Se o
tri
angulo for ret
angulo, e imediato que o ortocentro coincide com o vertice de angulo reto.
O ortocentro e exterior ao tri
angulo sempre que o tri
angulo for obtusangulo e e interior
quando for acutangulo.
Teorema 1. Sejam H, O, o ortocentro e o circuncentro de um ABC, respectivamente.
Entao HAB = OAC.
A
b

H O
b
b

B
b

C
b

Demonstrac
ao. Considere o caso em que o tri
angulo e acutangulo. O caso em que o
tri
angulo e obtus
angulo e an
alogo e e um bom exerccio. Tente! Seja HAB = . Tem-se,
ABC = 90 . Sabemos que AOC = 2.ABC = 180 2.. Mas, OAC = OCA,
portanto OAC = . Por isso, dizemos que AH e AO s
ao isogonais (formam angulos iguais
com os lados adjacentes).
Problema 1. No tri
angulo acutangulo ABC, a dist
ancia do vertice A ao ortocentro e igual
ao raio da circunferencia circunscrita. Determine os possveis valores do angulo BAC.
A
b

M
b

D
H
b
b

B
b

C
b

POT 2012 - Geometria - Nvel 2 - Aula 4 - Prof. Onofre Campos/Rodrigo


Pinheiro
Soluc
ao. Sejam H e O o ortocentro e o circuncentro do tri
angulo ABC; D e o pe da altura
relativa ao lado AB e M o ponto medio do lado AC. Entao, como AH = AO, HAD =
OAM , HDA = OM A, segue que os tri
angulos ADH e AM O s
ao congruentes. Logo,
AD
1
AD = AM = AC
.
Logo,
no
tri
a
ngulo
ADC,
temos
cos
A
=
=
,
angulo e
2
AC
2 e como o tri

acutangulo temos BAC = 60 .


Teorema 2. O simetrico do ortocentro em relacao a cada um dos lados do tri
angulo esta
sobre o crculo circunscrito.
Demonstrac
ao. Considere o caso em que o tri
angulo e acutangulo. O caso em que o
tri
angulo e obtus
angulo sera deixado como exerccio. Seja D o pe da altura relativa ao
lado BC e H1 o ponto onde essa altura reencontra o circuncrculo (crculo circunscrito ao

tri
angulo ABC). Logo HBC = DAC = 90 C. Mas H1 BD =
Entao HBD H1 BD HD = DH1 .

H1 C
2

= H1 AC.

A
b

H
b
b

O
D

b
b

H1
b

Problema 2. Usando regua e compasso, construa um tri


angulo ABC conhecendo apenas o
vertice A, o ortocentro H e a circunferencia circunscrita.
Soluc
ao. Facamos o esboco do tri
angulo:
A
b

H
b

O
b

B
b

H1

O prolongamento de AH encontra a circunferencia circunscrita em um ponto H1 tal


que H1 e o simetrico de H em relacao a BC. Assim, BC e mediatriz do segmento HH1 .
Portanto, fazemos a seguinte construcao: ligamos AH ate encontrar a circunferencia
circunscrita no ponto H1 . Entao, construmos a mediatriz de HH1 , que encontra a circunferencia circunscrita nos pontos B e C, e o tri
angulo ABC e construdo.
2

POT 2012 - Geometria - Nvel 2 - Aula 4 - Prof. Onofre Campos/Rodrigo


Pinheiro

Teorema 3. Os simetricos do ortocentro em relacao a cada um dos pontos medios dos lados
de um tri
angulo encontram-se sobre o crculo circunscrito.
Demonstrac
ao. Considere o caso em que o tri
angulo e acutangulo (Veja figura a seguir).
O caso em que o tri
angulo e obtus
angulo sera deixado como exerccio. Seja M o ponto
medio do lado BC do tri
angulo ABC. H e N s
ao o ortocentro e o simetrico de H em
relacao ao ponto M , respectivamente.
No quadrilatero HBN C, as diagonais se cortam os meio (BM = M C e HM = M N ).
Logo, HBN C e um paralelogramo, do modo que BN C = BHC. Mas, BHC = 180
A, e assim, BAC + BN C = A + (180 A) = 180 . Portanto HBN C e inscritvel
e o ponto N encontra-se sobre o circuncrculo de ABC, como queramos demonstrar.
A
b

H
b

O
M

B
b

b
b

Problema 3. (Cone Sul - 1998) Sejam H o ortocentro do tri


angulo ABC e M o ponto medio
do lado BC. A reta HM intersecta o crculo circunscrito de ABC em X, pertencente ao
arco BC que nao contem A, e BH encontra o crculo circunscrito em Y . Mostre que
XY = BC. Soluc
ao.
A

H
b

Y
b

O
M

b
b

Como X esta sobre o arco BC que nao contem A e sobre a reta HM , e facil ver que X
coincide com o simetrico de H em relacao ao ponto medio M . Alem disso, o quadrilatero
BXCH e um paralelogramo, de modo que XC e paralelo a BH. Entao, BXCY e um
trapezio isosceles, e portanto tem as diagonais iguais, ou seja, XY = BC.

POT 2012 - Geometria - Nvel 2 - Aula 4 - Prof. Onofre Campos/Rodrigo


Pinheiro

Tri
angulo Ortico
Considere um tri
angulo nao ret
angulo ABC, e sejam D, E, F os pes das alturas de
ABC. O tri
angulo DEF e chamado tri
angulo ortico do tri
angulo ABC.

Lema 1. O ortocentro H do tri


angulo ABC e o incentro de seu tri
angulo Ortico
DEF .

Demonstrac
ao. Seja ABE = . Entao F CA = 90 A = . Observe que
o quadrilatero BDF H e inscritvel (pois BDH + BF H = 180 ). Logo, F DH =
F BH = . Analogamente, EDH = . Portanto, H esta na bissetriz do angulo D. Do
mesmo modo, e facil verificar que H pertence a bissetriz de F ; ou seja H e o incentro do
tri
angulo DEF .
A
b

E
b

F
b

H
b

D
b

Problema 4. Usando apenas regua e compasso, construa um tri


angulo ABC conhecendo
os pontos que s
ao os simetricos do ortocentro em relacao aos lados AB, BC e CA.
Soluc
ao. Sejam X, Y e Z os simetricos do ortocentro em relacao a BC, CA e AB,
respectivamente. Observe que XY e paralelo `a reta que liga os pes das alturas relativas
a BC e CA. Dessa forma, os lados do tri
angulo XY Z s
ao paralelos, respectivamente, aos
lados do tri
angulo
ortico de ABC. Entao, H e o incentro do tri
angulo XY Z.
Segue a seguinte construcao: Primeiro, encontramos o circuncentro O do tri
angulo XY Z
(encontro das mediatrizes dos lados) e construmos a circunferencia circunscrita (com centro
O e raio OX. Em seguida, construmos as bissetrizes dos angulos de XY Z, que intersectam
a circunferencia circunscrita nos pontos A, B e C, determinando o tri
angulo ABC.
A
b

Y
b

Z
b

H
b

B
b

X
b

POT 2012 - Geometria - Nvel 2 - Aula 4 - Prof. Onofre Campos/Rodrigo


Pinheiro

Problema 5. Sejam D, E e F os pontos medios dos lados AB, BC e AC do tri


angulo
ABC, respectivamente. BL e a altura relativa ao lado AC. Mostre que DF E = DLE.
Problema 6. No tri
angulo ABC, as alturas AD e BE se cortam em H; M , N e P s
ao os
pontos medios de BC, AB e AH, respectivamente. Mostre que M N P = 90 .
Problema 7. Prove que, em todo tri
angulo, a circunferencia cujo di
ametro e um lado do
tri
angulo passa pelos pes das alturas relativas aos outros dois lados.
Problema 8. As bissetrizes internas de um tri
angulo ABC encontram o crculo circunscrito
novamente nos pontos M , N e P . Mostre que o incentro I do tri
angulo ABC e o ortocentro
do tri
angulo M N P .
Problema 9. Sejam AD e BE as alturas relativas aos lados BC e AC, respectivamente,
do tri
angulo ABC, H o ortocentro, M o ponto medio de AB e N o ponto medio de CH.
Mostre que M N e perpendicular e passa pelo ponto medio de DE.
Problema 10. A bissetriz interna do
angulo A do tri
angulo ABC encontra o crculo circunscrito no ponto M . Verifique que M e o ponto medio do arco BC e que M e o circuncentro
do tri
angulo BIC, em que I e o incentro do tri
angulo ABC.
Problema 11. Sejam O o circuncentro e H o ortocentro do tri
angulo ABC. Seja Oa o
simetrico de O em relacao ao lado BC. Mostre que Oa e o circuncentro do tri
angulo BCH.
Problema 12. Seja H o ortocentro do tri
angulo ABC. Mostre que os crculos circunscritos
aos tri
angulos ABH, BCH e CAH tem todos o mesmo raio, o qual e igual ao circunraio
do tri
angulo ABC.
Problema 13. As alturas relativas aos lados AB e AC do tri
angulo ABC encontram o
circuncrculo de ABC nos pontos D e E, respectivamente. Mostre que AD = AE.
Problema 14. O tri
angulo ABC esta inscrito em um crculo de centro O. Seja a circunferencia que passa pelos pontos A, O e B. As retas CA e CB interceptam em D e E,
respectivamente. Prove que CO e perpendicular a DE.
Problema 15. Construa um tri
angulo conhecendo apenas o circuncentro O, o ponto H, pe
da altura relativa ao lado BC e o ponto D, pe da bissetriz interna do angulo A.
Problema 16. Prove que as tres retas atraves dos pontos medios dos lados de um tri
angulo
e paralelas `as bissetrizes dos
angulos opostos s
ao concorrentes em um ponto.
Problema 17. Seja ABC um tri
angulo acutangulo de ortocentro H e circuncentro O. A
mediatriz do segmento AH corta AB no ponto P e AC no ponto Q. Demonstre que
AOP = AOQ.
Problema 18. Sejam H, O o ortocentro e o circuncentro do tri
angulo ABC. AD, BE e
CF s
ao as alturas relativas aos vertices A, B e C. Suponha que OH seja paralelo a AC.
Mostre que os lados do tri
angulo DEF estao em progress
ao aritmetica.
5

POT 2012 - Geometria - Nvel 2 - Aula 4 - Prof. Onofre Campos/Rodrigo


Pinheiro

Problema 19. Sejam AD, BE e CF as alturas do tri


angulo acutangulo ABC. A reta por
D paralela a EF encontra os lados AC e AB nos pontos Q e R respectivamente. A reta
EF intersecta BC no ponto P . Prove que a circunferencia circunscrita ao tri
angulo P QR
passa pelo ponto medio de BC.
Problema 20. Sejam H o ortocentro do tri
angulo ABC, nao ret
angulo, e M o ponto medio
do lado BC. A circunferencia de di
ametro AM encontra a circunferencia circunscrita ao
ABC em um segundo ponto P . Mostre que os pontos P , H e M s
ao colineares.
Problema 21. Seja ABC um tri
angulo acutangulo. Tres retas LA, LB, LC s
ao construdas
atraves dos vertices A, B e C respectivamente de acordo com as seguintes regras: seja H o
pe da altura tracada do vertice A para o lado oposto BC, seja SA o crculo com di
ametro
AH; SA encontra os lados AB e AC em M e N respectivamente, onde M e N s
ao distintos
de A; entao LA e a reta atraves de A perpendicular a M N . As retas LB e LC s
ao
construdas analogamente. Prove que LA, LB, LC s
ao concorrentes.
possvel construirmos um tri
Problema 22. E
angulo sendo conhecidos apenas o ortocentro
e dois dos pontos medios dos lados?
Problema 23. Considere tres crculos congruentes concorrentes em um ponto P . Sejam A,
B, C os outros pontos de intersecao dos crculos. Entao o raio comum destes tres crculos
e igual ao raio do crculo circunscrito de ABC, e P e o ortocentro de ABC.
Problema 24. Sejam H e O o ortocentro e o circuncentro do tri
angulo ABC. Mostre que a
dist
ancia do ortocentro a um vertice e o dobro da dist
ancia do circuncentro ao lado oposto
a este vertice.
Problema 25. Mostre que, em todo tri
angulo, o ortocentro H, o baricentro G e o circuncentro O s
ao colineares. (A reta que contem estes pontos e chamada reta de Euler). Mostre
ainda que H, G e O estao sempre na razao HG : GO = 2 : 1.
Problema 26. (Crculo dos Nove Pontos) Sejam H o ortocentro e O o circuncentro do
tri
angulo ABC. M , N e P s
ao os pontos medios dos lados BC, CA e AB, respectivamente;
D, E e F s
ao os pes das alturas relativas aos lados BC, CA e AB, respectivamente; R, S e
T s
ao os pontos medios de AH, BH e CH, respectivamente. Os nove pontos M , N , P , D,
E, F , R, S, T estao sobre uma circunferencia, com centro no ponto medio de OH e cujo
raio e metade do raio do crculo circunscrito a ABC. O crculo que contem estes pontos e
chamado crculo de Euler ou crculo dos nove pontos do tri
angulo ABC.
Problema 27. Prove que o raio do crculo dos nove pontos e igual a metade do raio do
crculo circunscrito.
Problema 28. Seis diferentes pontos s
ao escolhidos sobre uma circunferencia. O ortocentro
do tri
angulo formado por tres destes pontos e ligado ao baricentro do tri
angulo formado
pelos outros tres. Prove que os 20 segmentos que podem ser determinados desta maneira
s
ao todos concorrentes.

POT 2012 - Geometria - Nvel 2 - Aula 4 - Prof. Onofre Campos/Rodrigo


Pinheiro

Problema 29. Seja H o ortocentro do ABC. Prove que as retas de Euler dos tri
angulos
ABC, BCH, CAH, ABH s
ao todas concorrentes. Em que notavel ponto ABC estas retas
concorrem?
Problema 30. Seja H o ortocentro de um tri
angulo ABC, tal que AC 6= BC. O segmento
que une os pontos medios de HC e AB intercepta a bissetriz do angulo ACB no ponto
N . Sabendo que o circuncentro do tri
angulo ABC pertence `a reta que liga os pontos H e
N , determine a medida do ACB.
Problema 31. (OCSF) ABCD e um paralelogramo, H e o ortocentro de ABC e O e o
circuncentro de ACD. Prove que H, O e D s
ao colineares.

Polos Olmpicos de Treinamento


Aula

Curso de Geometria - Nvel 3

Prof. Ccero Thiago

Pontos Not
aveis II: Baricentro e reta de Euler
Propriedade 1. Num tri
angulo ret
angulo ABC, a mediana BM relativa `a hipotenusa
mede metade da hipotenusa AC.
A

D
M

Demonstrac
ao.
Seja D o ponto sobre o prolongamento da mediana BM tal que BM = M D. Os tri
angulos
AM B e CM D s
ao congruentes, pelo caso LAL. Da, AB = CD e BAM = DCM , ou
seja, AB e CD s
ao segmentos iguais e paralelos e portanto
ABC = DCB = 90 .
Assim, os tri
angulos ABC e DCB s
ao congruentes, pelo caso LAL, e portanto
BD = AC

= 2 BM = AC

= BM =

AC

Afirmac
ao. Uma base media de um tri
angulo e um segmento que une os pontos medios de
dois de seus lados.
Assim, todo tri
angulo possui exatamente tres bases medias.
Propriedade 2. Sejam ABC um tri
angulo e M , N os pontos medios dos lados AB, AC,
respectivamente. Ent
ao
BC
M N k BC e M N =

POT 2012 - Geometria - Nvel 3 - Aula 5 - Prof. Ccero Thiago


A
N

Demonstrac
ao.
Inicialmente, prolonguemos a base media M N ate um ponto P tal que M N = N P . Em
seguida, construmos o tri
angulo CN P . Note que os tri
angulos AN M e CN P s
ao congruentes, pelo caso LAL. Da, CP = AM e M AN = P CN e portanto
CP k AM

CP k BM.

Assim, M BCP e um paralelogramo, pois CP e BM s


ao segmentos paralelos e iguais. Mas
entao M P k BC e
M P = BC

2M N = BC

MN =

BC

Afirmac
ao. A base media de um trapezio e o segmento que une os pontos medios de seus
lados n
ao paralelos.
Propriedade 3. Seja ABCD um trapezio de bases AB e CD, e sejam M e N os pontos
medios dos lados BC e AD, respectivamente. Entao,
M N k AB, M N k CD

N
b

A
b

MN =

AB + CD
.
2

B
b

C
b

facil
Demonstrac
ao. Inicialmente, prolonguemos AM ate encontrar DC no ponto E. E
ver que
ABM CM E (ALA) AB = CE.
Portanto, M N e base media do tri
angulo ADE. Assim,
M N k BE M N k DC M N =
2

DE
.
2

POT 2012 - Geometria - Nvel 3 - Aula 5 - Prof. Ccero Thiago

Finalmente, M N =

DC + AB
DC + CE
=
.
2
2

Propriedade 4. As tres medianas de um tri


angulo intersectam - se num mesmo ponto,
chamado baricentro, que divide cada uma das medianas em duas partes tais que a parte
que contem o vertice e o dobro da outra.

Demonstrac
ao.
A

P
D

N
G1

Sejam N e P os pontos medios dos lados AC e AB, respectivamente, D e E os pontos


medios de BG1 e CG1 , respectivamente. Entao,
N P k BC e N P =
e

BC
2

BC
2
portanto, P DEN e uma paralelogramo. Com isso, BD = DG1 = G1 N , CE = EG1 =
G1 P , entao BG1 = 2G1 N e CG1 = 2G1 P . De maneira an
aloga, as medianas AM e BN
intersectam - se em um ponto G2 tal que AG2 = 2G2 M e BG2 = 2G2 N . Encontramos,
entao, dois pontos distintos G1 e G2 , no interior do segmento BN que o dividem na mesma
raz
ao, o que e uma contradic
ao logo, G1 = G2 = G. Portanto, as tres medianas intersectam
- se em um mesmo ponto G que chamaremos de baricentro.
DE k BC e DE =

POT 2012 - Geometria - Nvel 3 - Aula 5 - Prof. Ccero Thiago

Propriedade 5. O ortocentro, o baricentro e o circuncentro de um tri


angulo, n
ao equilatero,
s
ao colineares. A reta determinada por esses pontos e chamada de Reta de Euler.
Demonstrac
ao.
Sejam M e N os pontos medios de BC e AC, respectivamente. Entao, M N k AB e
AB
. O teorema 1 da aula 4 garante que BAD = OAC. Como O e o cirMN =
2
cuncentro ent
ao OA = OC e, com isso, OAC = OCA. O quadril
atero M CN O e
0
inscritvel ent
ao OCA = N CO = OM N e M ON = 180 ACB. Alem disso,
o quadril
atero DCEH tambem e inscritvel e, com isso, DHE = 1800 ACB. Como
DHE = AHB conclumos que o tri
angulo AHB e semelhante ao tri
angulo M N O e, com
AB
AH
isso,
=
= 2. Temos que HAG = GM O pois AH e paralelo a OM e, como
MN
OM
AG
= 2. Portanto, o tri
angulo AHG e semelhante ao tri
angulo GM O
G e o baricentro,
GM
e, com isso, HGA = M GO provando entao que H, G e O est
ao alinhados e HG = 2GO.
A
b

b
b

H
b

Propriedade 6. Os pes das alturas de um tri


angulo, os pontos medios do tres lados e os
pontos medios dos segmentos que ligam os vertices ao ortocentro est
ao sobre uma circunferencia chamada Circunfer
encia dos 9 pontos.

Demonstrac
ao. Queremos provar que M , L, P , D, E, F , R, S e T s
ao concclicos. E
suficiente provar que R e D est
ao sobre a circunferencia circunscrita ao tri
angulo M LP ,

pois o restante e an
alogo. Considere a circunferencia de di
ametro RM . E facil ver que
D pertence a . Por outro lado, RL k HC, LM k AB e HC AB, o que implica que
RLM = 90 . Portanto, L (e por simetria P ) pertence a .

POT 2012 - Geometria - Nvel 3 - Aula 5 - Prof. Ccero Thiago

A
b

E
b

F
b

H
b

P
b

L
b

N
b

S
b

b
b

b
b

Propriedade 7. O centro da circunferencia dos 9 pontos e o ponto medio do segmento


formado pelo ortocentro e pelo circuncentro.
Demonstrac
ao.
Seja RM um di
ametro da circunferencia dos 9 pontos e seja N a intersecao de RM e OH.
Como R e ponto medio de AH ent
ao RH = OM . Alem disso, AH k OM . Portanto,
RHN N OM , RN = N M e HN = ON .
A
b

R
F

b
b

P
b

L
b

H
b

N
b

G
b

S
b

b
b

POT 2012 - Geometria - Nvel 3 - Aula 5 - Prof. Ccero Thiago

Exerccios Resolvidos
1. (OBM) Considere um tri
angulo acut
angulo ABC com BAC = 30 . Sejam B1 , C1 os
pes das alturas relativas aos lados AC, AB, respectivamente, e B2 , C2 os pontos medios
dos lados AC, AB, respectivamente. Mostre que os segmentos B1 C2 e B2 C1 s
ao perpendiculares.
A

C2

B2

B1

C1
B

Soluc
ao.
Seja O a intersec
ao entre B1 C2 e B2 C1 . O segmento B1 C2 e uma mediana do tri
angulo
ret
angulo AB1 B e portanto
AC2 = B1 C2

C2 B1 A = BAB1 = 30 .

Analogamente, AC1 B2 = 30 . Da,


BC2 B1 = C2 B1 A + BAB1 = 60
e portanto
C1 OC2 = 180 BC2 B1 AC1 B2 = 90 .
2.Sejam ABC um tri
angulo e M o ponto medio do lado BC. Se D, E s
ao os pes das alturas
relativas aos lados AC, AB, respectivamente, prove que M E = M D.
Soluc
ao.
A
D

POT 2012 - Geometria - Nvel 3 - Aula 5 - Prof. Ccero Thiago

Note que M E e mediana relativa `


a hipotenusa do tri
angulo BEC. Da,
M E = BM = CM
e, analogamente,
M D = BM = CM.
Assim, M E = M D.
3. Dado um quadril
atero ABCD, prove que os pontos medios M, N, P, Q dos lados AB,
BC, CD, DA formam um paralelogramo.
Soluc
ao.
A
M

D
N

P
C

Temos
Tri
angulo ABC: M N k AC e M N = AC/2.
Tri
angulo DAC: P Q k AC e P Q = AC/2.
Assim, M N k P Q e M N = P Q, isto e, M N P Q e paralelogramo.
4. (OBM) Seja N o ponto do lado AC do tri
angulo ABC tal que AN = 2N C e M o
ponto do lado AB tal que M N e perpendicular a AB . Sabendo que AC = 12 cm e que o
baricentro G do tri
angulo ABC pertence ao segmento M N , determine o comprimento do
segmento BG.
OBS: Baricentro e o ponto de intersecao das medianas do tri
angulo.
Soluc
ao.
Se BP e uma mediana do tri
angulo ent
ao AP = CP = 6 e P N = 2. Como G e o baricentro
PG
1 PN
1
do tri
angulo ent
ao
= e
= , assim, pela recproca do teorema de Tales, GN e
GB 2 N C
2
paralelo a BC e B = 90 . Como o tri
angulo ABC e ret
angulo entao AP = CP = BP = 6.
Com isso, BG = 4 e GP = 2.

POT 2012 - Geometria - Nvel 3 - Aula 5 - Prof. Ccero Thiago

M
b

b
b

5. Em um tri
angulo n
ao equilatero, a reta que passa pelo baricentro e pelo incentro e paralela a um dos lados tri
angulo. Demonstre que os lados do tri
angulo est
ao em progressao
aritmetica.
Soluc
ao.
Seja AD a bissetriz relativa ao vertice A, I o incentro, AM a mediana relativa ao vertice
A e G o baricentro. Sejam a, b e c as medidas dos lados BC, CA e AB, respectivamente.
b+c
AI
=
. Como a reta que passa
Pelo teorema da bissetriz interna e facil provar que
ID
a
pelo baricentro G e pelo incentro I e paralela a um dos lados, pelo teorema de Tales, temos
AG
AI
b+c
que 2 =
=
=
, ou seja, b + c = 2a. Portanto, os lados do tri
angulo formam
GM
ID
a
uma progress
ao aritmetica.
Exerccios Propostos
Problema 1. Sejam ABC um tri
angulo e M o ponto medio de BC. Se AM = BM = CM ,
prove que BAC = 90 .
Problema 2. (Austr
alia) Sejam ABC um tri
angulo e P um ponto em seu interior de modo
que P AC = P BC. Se L, M s
ao os pes das perpendiculares por P aos lados BC, AC,
respectivamente, e D e o ponto medio de AB, prove que DL = DM .
Problema 3. Uma reta r passa pelo baricentro de um tri
angulo ABC deixando o vertice A
em um semiplano e os vertices B e C no outro semiplano determinado por r. As projecoes
de A, B e C sobre a reta r s
ao M , N e P , respectivamente. Prove que AM = BN + CP .
Problema 4. (OBM) Seja ABCD um quadril
atero convexo, onde N e o ponto medio de
DC, M e o ponto medio de BC, e O e a intersecao entre as diagonais AC e BD. Mostre
8

POT 2012 - Geometria - Nvel 3 - Aula 5 - Prof. Ccero Thiago

que O e o baricentro do tri


angulo AM N se, e somente se, ABCD e um paralelogramo.
Problema 5. Prove que se a reta de Euler passa pelo incentro do tri
angulo, entao o tri
angulo
e is
osceles.
Problema 6. (Bulg
aria) Seja ABC um tri
angulo is
osceles (AC = BC) tal que A1 , B1
e C1 s
ao os pontos medios de BC, AC e AB, respectivamente. Os pontos A2 e B2 s
ao
os simetricos de A1 e B1 com relac
ao ao lado AB. Seja M a intersecao de CA2 e A1 C1 e
seja N a intersec
ao de CB2 e B1 C1 . Seja P a intersecao de AN e BM , prove que AP = BP .
Problema 7. (Portugal) No tri
angulo ABC as medianas dos lados AB e AC s
ao perpendiculares. Sabendo que AB = 6 e AC = 8, determine BC.
Problema 8. (Bulg
aria) Os pontos A1 , B1 e C1 est
ao sobre os lados BC, CA e AB do
tri
angulo ABC, respectivamente, tais que AA1 , BB1 e CC1 concorrem no ponto M . Prove
que se M e o baricentro do tri
angulo A1 B1 C1 entao M e tambem o baricentro do tri
angulo
ABC.
Problema 9. (Estonia) As medianas relativas aos vertices A e B do tri
angulo ABC s
ao
perpendiculares. Prove que AB e o menor lado do tri
angulo ABC.
Problema 10. (OCM) Seja ABC um tri
angulo tal que as medianas BM e CN , que se
cortam em G, s
ao iguais. Prove que o tri
angulo ABC e is
osceles.
Problema 11. Prove que a soma dos quadrados das dist
ancias de um ponto P aos vertices
de um tri
angulo ABC e mnima quando P e o baricentro do tri
angulo.
Problema 12. (Espanha) Seja G o baricentro do tri
angulo ABC. Se
AB + GC = AC + GB,
prove que o tri
angulo e is
osceles.
Problema 13. (OBM) Sejam ABC um tri
angulo acut
angulo e F o seu ponto de Fermat,
isto e, o ponto interior ao tri
angulo ABC tal que os tres angulos AF B, BF C e CF A,
medem 120 graus. Para cada um dos tri
angulos ABF , ACF e BCF trace a sua reta de
Euler, ou seja, a reta que liga o seu circuncentro e o seu baricentro. Prove que essas tres
retas concorrem em um ponto.

POT 2012 - Geometria - Nvel 3 - Aula 5 - Prof. Ccero Thiago

Solu
co
es
1. Se AD = BD = CD, ent
ao ABD = BAD = e DAC = ACD = . A soma dos
angulos internos do tri
angulo ABC garante que + = 90 , ou seja, A = 90 .

2. Seja N o ponto medio de AP e K o ponto medio de BP . Entao, AN = N M = N P


e LK = BK = KP . Com isso, P N M = 2P AC = 2P BC = P KL. Alem disso,
BP
, DK k AP
DN e DK s
ao bases medias do tri
angulo ABP assim, DN k BP , DN =
2
AP
. Portanto, DN P K e um paralelogramo e DN P = DKP . Finalmente,
e DK =
2
DN M DKL, pelo caso LAL. Assim, DL = DM .
A
N
M
D
P
B

3. Seja AD um mediana e Q o ponto medio de N P . Entao, DQ e a base media do trapezio


BN + CP
. Como G e o baricentro do tri
angulo ABC
N BCP assim DQ k BN e DQ =
2
facil ver que AM G GQD, entao AM = DQ. Portanto,
entao AG = 2GD. E
2
AM = BN + CP .

10

POT 2012 - Geometria - Nvel 3 - Aula 5 - Prof. Ccero Thiago

A
b

G
b

b
b

4. () Suponha que ABCD e um paralelogramo, entao AO = OC e BO = BD. Se M


BD
e N s
ao os pontos medios de BC e CD entao M N k BD e M N =
. E facil concluir
2
DO
BO
, N P k DO e N P =
.
que P e o ponto de medio de OC ent
ao M P k BO, M P =
2
2
Portanto, N P = P M e AO = 2OP , ou seja, O e o baricentro de AM N .
() Suponha que O e o baricentro do tri
angulo AM N entao N P = P M e AO = 2OP . Se
BD
. E facil concluir
M e N s
ao os pontos medios de BC e CD entao M N k BD e M N =
2
BO
, N P k DO e
que P e o ponto de medio de OC ent
ao OP = P C, M P k BO, M P =
2
DO
. Da, AO = OC e DO = OB, ou seja, ABCD e um paralelogramo.
NP =
2

11

POT 2012 - Geometria - Nvel 3 - Aula 5 - Prof. Ccero Thiago

A
b

b
b

O
b

P
b

b
b

5. Sejam O o circuncentro, I o incentro e H o ortocentro do tri


angulo ABC. As retas AI
e BI intersectam o crculo circunscrito do tri
angulo ABC nos pontos A1 e B1 . Suponha
OA1
OI
OB1
OI
=
e
=
. Como OB1 = OC1
que o tri
angulo ABC n
ao e is
osceles, entao
IH
AH BH
BH
entao AH = BH e, portanto AC = BC. Contradicao.
6. Como CC1 k A1 A2 e CC1 = A1 A2 , temos que CC1 A2 A1 e um paralelogramo. Entao,
A1 M = C1 M . Mas A1 B1 C1 B e tambem um paralelogramo e, portanto, a intersecao BM
e AC e B1 . Ent
ao, P est
a sobre a mediana BB1 . Analogamente, P est
a sobre a mediana
AA1 . No tri
angulo is
osceles ABC as medianas AA1 e BB1 possuem o mesmo comprimento.
2
2
Portanto, AP = AA1 = BB1 = BP .
3
3
7. Sejam M e N os pontos medios de AB e AC, respectivamente, e G o ponto de encontro
das medianas M C e N B. Aplicando o teorema de Pit
agoras BIM e CN I, temos:
GM 2 + 4GN 2 = GM 2 + GB 2 = BM 2 = 32 = 9
e
4GM 2 + GN 2 = GC 2 + GN 2 = CN 2 = 42 = 16.

Deste modo, 5GM 2 + 5GN 2 = 9 + 16 = 25, logo N M = 5. Portanto, BC = 2 5.

12

POT 2012 - Geometria - Nvel 3 - Aula 5 - Prof. Ccero Thiago

N
b

8. Seja M o baricentro do tri


angulo A1 B1 C1 . Seja A2 um ponto sobre a reta M A tal que
B1 A1 C1 A2 e um paralelogramo. Os pontos B2 e C2 s
ao construdos analogamente. Como
A1 C1 k A1 B1 k C1 B2 ent
ao os pontos A2 , C1 e B2 s
ao colineares e C1 e o ponto medio de
A2 B2 . O mesmo e verdade para os pontos A2 , B1 e C2 e C2 , A1 e B2 . Vamos mostrar que
A2 = A, B2 = B e C2 = C, o que resolve o problema. Assuma que A2 6= A e A est
a entre
A2 e M . Ent
ao C2 est
a entre C e M , B est
a entre B2 e M e consequentemente A2 est
a
entre A e M , que e uma contradic
ao.
9. As medianas intersectam - se no ponto M e a mediana que parte do vertice C intersecta
AB no ponto F . Ent
ao, F e o ponto medio da hipotenusa do tri
angulo ret
angulo ABM ,
ou seja, AB = 2F M . Como M divide a mediana CF na raz
ao 2 : 1, entao AB = CM .
O maior angulo do tri
angulo AM C e o angulo obtuso AM C, portanto AC e o maior lado
deste tri
angulo. Assim, AC > M C = AB. De maneira an
aloga BC > AB.
m
= GN e
10. Seja BM = CN = m. Como G e o baricentro de ABC, temos GM =
3
2m
BG =
= CG. Da, segue que os tri
angulos BGN e CGM s
ao congruentes (pelo caso
3
LAL), de modo que BN = CM . Logo, AB = 2 BN = 2 CM = AC, e o tri
angulo ABC
e is
osceles.

13

POT 2012 - Geometria - Nvel 3 - Aula 5 - Prof. Ccero Thiago

A
b

N
b

G
b

11. Seja ABC um tri


angulo com BC = a, AC = b e AB = c. Seja M o ponto medio de
BC, G o baricentro do tri
angulo ABC e P um ponto qualquer. Usando que, a soma dos
quadrados de dois dos lados de um tri
angulo e igual a duas vezes o quadrado da mediana
relativa ao terceiro lado mais a metade do quadrado do terceiro lado (a demonstracao desse
resultado usa lei dos Cossenos e ser
a provado na aula de relacoes metricas), no tri
angulo
P BC com mediana P M temos:
P B 2 + P C 2 = 2P M 2 +

a2
. (I)
2

O baricentro G e tal que GA = 2GM . Faca GM = m; GA = 2m e tome H em AG tal que


GH = AH = m. Assim, o tri
angulo HP M , com mediana P G satisfaz
1
P H 2 + P M 2 = 2P G2 + (2m)2 = 2P G2 + 2m2 (II)
2
e o tri
angulo AP G com mediana P H satisfaz
1
P A2 + P G2 = 2P H 2 + (2m)2 = 2P H 2 + 2m2 . (III)
2
Somando (I) e (III)
P A2 + P B 2 + P C 2 + P G2 = 2P M 2 +
14

a2
+ 2P H 2 + 2m2 =
2

POT 2012 - Geometria - Nvel 3 - Aula 5 - Prof. Ccero Thiago

= 2(P M 2 + P H 2 ) +

a2
+ 2m2 = por (II)
2

2(2P G2 + 2m2 ) +

a2
+ 2m2 =
2

4P G2 + 6m2 +
Portanto, P A2 + P B 2 + P C 2 = 3P G2 + 6m2 +

a2
.
2

a2
. (IV)
2

Como o tri
angulo a e m s
ao constantes, P A2 + P B 2 + P C 2 e mnimo quando P G = 0, ou
seja, P = G e o baricentro do tri
angulo ABC.
A
b

H
b

b
b

12. Sejam D, E e F os pontos medios dos lados BC, AC e AB, respectivamente. Dividindo
por 2 a condic
ao do enunciado temos
F A + F G = EA + EG,
ou seja, os pontos F e E est
ao sobre uma elipse de focos A e G. Seja M o ponto medio de
EF , M est
a sobre a mediana AD e n
ao e o centro a elipse (ponto medio de AG), portanto
EF e perpendicular a AD e, ent
ao, AD alem de mediana e tambem uma altura fazendo
com que o tri
angulo seja is
osceles.
13. Construa um tri
angulo equilatero BXC, externo a ABC. O ponto O1 e o circuncentro
de BF C e tambem de BXC. Como G e o baricentro do tri
angulo ABC entao:
XO1
AG
=2=
O1 G k XF.
GM
O1 M
15

POT 2012 - Geometria - Nvel 3 - Aula 5 - Prof. Ccero Thiago

Mas O3 A = O3 F e O2 A = O2 F AF O2 O3 O1 G O2 O3 . Analogamente, temos


O2 G O1 O3 e O3 G O2 O3 G e ortocentro do tri
angulo O1 O2 O3 . Sendo G1 o bariAG
F G1
=2=
G1 G k AF G1 G O2 O3 como
centro do tri
angulo BF C temos
G1 M
GM
G e o ortocentro de O1 O2 O3 , ent
ao G1 est
a na altura relativa a O2 O3 . Portanto, O1 G1 ,
O2 G2 e O3 G3 s
ao concorrentes em G (seu ortocentro).

A
b

O2

O3
b

F
b

G1
b
b

Bibliografia
1. Lecture Notes on Mathematical Olympiad Courses
For Junior Section, vol. 1
Xu Jiagu
2. Puntos Notables - Teora - Demostraciones - Trazos Auxiliares
440 problemas resueltos e propuestos
Julio Orihuela Bastidas
Editorial Cuzcan
3. Geometra
Radmila Bulajich Manfrino e Jose Antonio G
omez Ortega
Cuadernos de Olimpiadas de Matematicas
4. T
opicos de Matematica Elementar, vol. 2
Geometria Euclidiana Plana
Antonio Caminha Muniz Neto

16

POT 2012 - Geometria - Nvel 3 - Aula 5 - Prof. Ccero Thiago

SBM
5. Episodes in Nineteenth and Twentieth Euclidean Geometry
Ross Honsberger
MAA
6. Problems in Plane and Solid Geometry, vol. 1 - Plane Geometry
Viktor Prasolov
7. Advanced Euclidean Geometry
Alfred Posamentier
8. Lessons in Geometry
I. Plane Geometry
Jacques Hadamard
AMS
9. Hadamards Plane Geometry
A Readers Companion
Mark Saul
AMS
10. Colecao Elementos da Matematica
Geometria Plana, vol. 2
Marcelo Rufino de Oliveira e Marcio Rodrigo da Rocha Pinheiro
11. Olimpadas Cearenses de Matematica, Ensino Medio, 1981 - 2005
Emanuel Carneiro, Francisco Ant
onio M. de Paiva e Onofre Campos
12. Problemas de las Olimpiadas Matematicas del Cono Sur (I a IV)
Fauring - Wagner - Wykowski - Gutierrez - Pedraza - Moreira
Red Olmpica
13. Fundamentos de Matematica Elementar, vol. 9 - Geometria Plana
Osvaldo Dolce e Jose Nicolau Pompeo
14. Olimpiada Matematica Espa
nola
15000 problemas de diferentes Olimpiadas de Matematica en el mundo

17

Polos Olmpicos de Treinamento


Aula

Curso de Geometria - Nvel 3

Prof. Ccero Thiago

Relac
oes entre
areas

Teorema 1. (F
ormula tradicional.)

A area do tri
angulo ABC pode ser calculada por [ABC] =

BC AD
.
2

Teorema 2. (Area
de um tri
angulo em fun
c
ao do raio da circunfer
encia inscrita.)
Sejam a, b e c as medidas dos lados BC, CA e AB do tri
angulo ABC, respectivamente, e
seja r a medida do raio da circunferencia inscrita. Entao, a area do tri
angulo ABC pode
ser calculada por
[ABC] = p r,
em que p =

a+b+c
.
2

Demonstrac
ao.

POT 2012 - Geometria - Nvel 3 - Aula 6 - Prof. Ccero Thiago

A
b

r
b

r
b
b

[ABC] = [BIC] + [CIA] + [AIB]


ar br cr
+
+

2
2
2


a+b+c
[ABC] =
r
2

[ABC] =

[ABC] = p r.
Teorema 3. (F
ormula trigonom
etrica da
area de um tri
angulo.)
Sejam a, b e c as medidas dos lados BC, CA e AB do tri
angulo ABC, respectivamente.
A area do tri
angulo ABC pode ser calculada por
[ABC] =

a c sinB
a b sinC
b c sinA
=
=
.
2
2
2

Demonstrac
ao. Vamos demonstrar uma das igualdades. As outras s
ao an
alogas.

POT 2012 - Geometria - Nvel 3 - Aula 6 - Prof. Ccero Thiago

B
b

Seja A = . Temos que

AC BD
aH
=
.
2
2
H
H = c sin , entao
Por outro lado, no tri
angulo ABD, temos sin =
c
[ABC] =

[ABC] =

a c sin
.
2

Teorema 4. (Area
de um tri
angulo em fun
c
ao do raio da circunfer
encia circunscrita.)
Sejam a, b e c as medidas dos lados BC, CA e AB do tri
angulo ABC, respectivamente,
e seja R o raio da circunferencia circunscrita. Entao, a area do tri
angulo [ABC] pode ser
calculada por
abc
[ABC] =
.
4R
Demonstrac
ao.

POT 2012 - Geometria - Nvel 3 - Aula 6 - Prof. Ccero Thiago

A
b

O
b

b
b

Sejam a, b e c as medidas dos lados BC, CA e AB do tri


angulo ABC, respectivamente.
Temos que
a c sin
[ABC] =
.
2
Por outro lado, seja AD um di
ametro entao, no ACD, temos que
sin =

b
.
2R

Portanto,
[ABC] =

abc
.
4R

Teorema 5. (Area
de um tri
angulo em fun
c
ao do raio de uma circunfer
encia ex
- inscrita.)
Sejam a, b e c as medidas dos lados BC, CA e AB do tri
angulo ABC, respectivamente,
e sejam ra , rb e rc os raios das circunferencias ex - inscritas relativas aos lados a, b e c,
respectivamente. Ent
ao, a
area do tri
angulo ABC pode ser calculada por
[ABC] = ra (p a) = rb (p b) = rc (p c),
em que p =

a+b+c
.
2

Demonstrac
ao.

POT 2012 - Geometria - Nvel 3 - Aula 6 - Prof. Ccero Thiago

F
b

ax

ra

C
b

Ia
b

ax

ra
ra
D

x
b

Pela propriedade dos segmentos tangentes, temos que DB = BE = x e DC = CF = a x.


Entao,
[ABC] = [AIa E] + [AIa F ] 2[BCIa ]

(c + x) ra (b + a x) ra
a ra
+
2

2
2
2
ra
ra
[ABC] =
(a + b + c 2a) =
(2p 2a) = ra (p a).
2
2
Analogamente,
[ABC] =

[ABC] = rb (p b) = rc (p c),
Teorema 6. (F
ormula de Heron.)
Sejam a, b e c as medidas dos lados BC, CA e AB do tri
angulo ABC, respectivamente.
Entao, a area do tri
angulo ABC pode ser calculada por
p
[ABC] = p (p a) (p b) (p c),
em que p =

a+b+c
.
2

Demonstrac
ao.

POT 2012 - Geometria - Nvel 3 - Aula 6 - Prof. Ccero Thiago

A
b

am

Aplicando o teorema de Pit


agoras nos tri
angulos ABD e ACD, temos:
1. c2 = m2 + h2 .
2. b2 = (a m)2 + h2 .
De (2), temos:
b2 = (a m)2 + h2
b2 = a2 2am + m2 + h2
b2 = a2 2am + c2
a2 + c2 b2
.
m=
2a

Substituindo em (1), temos:


2
a2 + c2 b2
+ h2
c =
2a
 2
2
a + c2 b2
2
2
h =c

2a

 

a2 + c2 b2
a2 + c2 b2
2
h = c+
c

2a
2a

 

2ac + a2 + c2 b2
2ac a2 c2 + b2
2
h =

2a
2a
2

4a2 h2 = [(a + c)2 b2 ] [(b2 (a c)2 ]

4a2 h2 = (a + c + b) (a + c b) (b + a c) (b + c a)

4a2 h2 = (a + b + c) (b + c a) (a + c b) (a + b c)
4a2 h2 = 2p (2p 2a) (2p 2b) (2p 2c)
a2 h2
= p (p a) (p b) (p c)
2
[ABC]2 = p (p a) (p b) (p c)
p
[ABC] = p (p a) (p b) (p c).
6

POT 2012 - Geometria - Nvel 3 - Aula 6 - Prof. Ccero Thiago

Teorema 7. (Rela
c
ao entre as
areas de tri
angulos semelhantes.)
Sejam ABC e DEF dois tri
angulos semelhantes tais que
[ABC]
= k2 .
[DEF ]

AB
AC
BC
=
=
= k, entao
DE
DF
EF

Demonstrac
ao.
AC
BC
AG
AB
=
=
=
= k, entao
DE
DF
EF
DH

Se ABC DEF com

BC AG
[ABC]
BC AG
2
=

= k k = k2 .
=
EF DH
[DEF ]
EF DH
2
A
b

D
b

b
b

Teorema 8. Sejam r e s retas paralelas. Sejam A e B pontos distintos sobre a reta s e C1


e C2 pontos distintos sobre a reta r. Entao, [ABC1 ] = [ABC2 ].
Demonstrac
ao. O resultado e imediato pois [ABC1 ] = [ABC2 ] =

r
b

C1
b

s
b

C2

AB H
.
2

POT 2012 - Geometria - Nvel 3 - Aula 6 - Prof. Ccero Thiago

Teorema 9. (Usando
areas para calcular raz
ao de segmentos.)
Seja ABC um tri
angulo e D, E e F pontos sobre os lados BC, CA e AB tais que AD,
BE e CF s
ao concorrentes no ponto P . Defina K = [ABC], KA = [P BC], KB = [P CA]
e KC = [P AB]. Como K = KA + KB + KC , entao
(a)
BD
KC CE
KA AF
KB
=
=
=
,
e
.
DC
KB EA
KC F B
KA
(b)
AP
KB + KC BP
KA + KC CP
KA + KB
=
,
=
e
=
PD
KA
PE
KB
PF
KC
Demonstrac
ao.
A
b

F
b

H2

P
b

H1
b

(a) Temos que


BD
[ABD]
[BP D]
[ABD] [BP D]
[AP B]
KC
=
=
=
=
=
.
CD
[ACD]
[CP D]
[ACD] [CP D]
[ACP ]
KB
Da mesma maneira demonstra - se que

KA AF
KB
CE
=
e
=
.
EA
KC F B
KA

(b) Temos que


ADS P DR
AD
H2
KA + KB + KC
[ABC]
=
=
=

PD
H1
[BP C]
KA
AP
KB + KC
=
.
PD
KA
Da mesma maneira demonstra - se que

BP
KA + KC CP
KA + KB
=
e
=
.
PE
KB
PF
KC

POT 2012 - Geometria - Nvel 3 - Aula 6 - Prof. Ccero Thiago

Teorema 10. (Area


de quadril
atero convexo qualquer.)
Seja ABCD um quadril
atero convexo qualquer tal que e o menor angulo entre as diagoAC BD sin
.
nais. Entao, [ABCD] =
2
Demonstrac
ao.
Temos que
[ABCD] = [AP D] + [BP C] + [CP D] + [DP A]

P A P D sin P A P B sin P B P C sin P C P D sin


+
+
+

2
2
2
2
(P A P D + P A P B + P B P C + P C P D) sin

[ABCD] =
2
AC BD sin
(P A + P C)(P B + P D) sin
[ABCD] =
.
[ABCD] =
2
2
Exerccios Resolvidos
[ABCD] =

1. (IMO) Considere um tri


angulo P1 P2 P3 e um ponto P no interior no tri
angulo. Cevianas P1 P, P2 P, P3 P intersectam os lados opostos em pontos Q1 , Q2 , Q3 , respectivamente. Prove que, entre n
umeros
P1 P P2 P P3 P
,
,
,
P Q1 P Q2 P Q3
pelo menos um e menor ou igual a 2 e pelo menos um e maior ou igual a 2.
Soluc
ao. Defina que [P P2 P3 ] = KA , [P P1 P3 ] = KB e [P P1 P2 ] = KC . Usando
o teorema 9 e, supondo sem perda de generalidade, que KA KB KC .
Entao,
KA + KB
KC + KC
P3 P
=

= 2,
P Q3
KC
KC
e
P1 P
KB + KC
KA + KA
=

= 2.
P Q1
KA
KA
2. A area de um tri
angulo e dada pela formula S =
lados. Determine os
angulos do tri
angulo.
Soluc
ao.

a2 + b2
, onde a e b s
ao dois de seus
4

POT 2012 - Geometria - Nvel 3 - Aula 6 - Prof. Ccero Thiago


a 2 + b2
a b sin C
=
. Entao,
2
4
a2 + b2
a2 + b2
(a b)2
sin C =
sin C 1 =
1 sin C 1 =
0.
2ab
2ab
2ab

Temos que [ABC] =

Assim, 1 sin C 1 sin C = 1 C = 90 . A igualdade s


o acontece se, e
somente se, a = b. Portanto, os
angulos do tri
angulo s
ao 45 , 45 , 90 .
3. S
ao dados 1000 pontos no plano n
ao colineares tais que se tres deles determinam um
tri
angulo ent
ao sua
area e menor ou igual a 1. Prove que todos os pontos est
ao em
um tri
angulo de
area menor ou igual a quatro.
Soluc
ao.
b

b
b

Y
D
b

b
b

Como existe um n
umero finito de tri
angulos que podem ser construdos usando os
1000 pontos ent
ao, escolhemos aquele de area m
axima que chamaremos de XY Z.
Seja ABC o tri
angulo tal que X, Y e Z s
ao os pontos medios de BC, CA e AB,
respectivamente, ent
ao [ABC] = 4[XY Z] 4. Seja D, um ponto no conjunto
dos 1000 pontos dados, no exterior do tri
angulo ABC entao [XY Z] < [XZD],
o que contradiz a escolha de ABC. Portanto, todos os pontos est
ao no interior do
tri
angulo ABC.
4. (Hungria) S e um ponto no interior do ABC tal que as areas dos tri
angulos ABS,
BCS, CAS s
ao todas iguais. Prove que S e o baricentro de ABC.
Soluc
ao.
Seja T a
area dos tri
angulos ABS, BCS, CAS. Da, sendo M , N e P as inCN
AP
T
BM
=
=
= = 1,
tersecoes de AS, BS e CS com os lados opostos, temos
MC
NA
PB
T
isto e, M , N e P s
ao os pontos medios dos lados BC, CA e AB e, portanto, S e o
10

POT 2012 - Geometria - Nvel 3 - Aula 6 - Prof. Ccero Thiago

baricentro de ABC.
5. (IMO Short List) Seja ABC um tri
angulo acut
angulo com circuncentro O e circunraio
R. Seja A1 6= O o ponto de intersecao de AO com a circunferencia circunscrita ao
tri
angulo BOC e defina analogamente B1 e C1 . Mostre que
OA1 OB1 OC1 8R3 .
Quando ocorre a igualdade?
Soluc
ao. Sejam D, E e F as intersecoes de AO, BO e CO com BC, CA e AB,
facil ver que AO = BO = CO = R. Usando o teorema 9 temos
respectivamente. E
que:
[AOB] + [AOC]
AO
=
,
OD
[BOC]
[AOB] + [BOC]
BO
=
,
OE
[AOC]
CO
[AOC] + [BOC]
=
.
OF
[AOB]
Faca [AOB] = x, [AOC] = y e [BOC] = z.
DCO, ent
ao
OA1
R
=
OA1 =
OD
R
Analogamente, OB1 =

facil perceber que OA1 C


E
R2
.
OD

R2
R2
e OC1 =
. Assim,
OE
OF

R
R
R
OA BO CO
R6
=

R3 =

R3 =
OD OE OF
OD OE OF
OD OE OF

2 xy 2 yz 2 zx
(x + y)(x + z)(y + z)
8zyz
3
=
R
R3 =
R3 = 8R3 .
xyz
xyz
xyz

OA1 OB1 OC1 =

A igualdade ocorre quando x = y = z. O exerccio 4 garante que O e o baricentro.

11

POT 2012 - Geometria - Nvel 3 - Aula 6 - Prof. Ccero Thiago


A

A1

6. (Coreia) Seja ABCD um quadril


atero convexo e seja P o ponto de intersecao das
diagonais. Prove que
[P AB] + [P CD] = [P BC] + [P DA]
se, e somente se, P e o ponto medio de AC ou BD.
1
PA PB
4
P C P D sin P . Os n
umeros [P AB], [P CD] e [P BC], [P DA] tem a mesma
soma e o mesmo produto, ent
ao [P AB] = [P BC] e [P CD] = [P DA] ou
[P AB] = [P DA] e [P BC] = [P CD], ou seja, P e o ponto medio de AC ou
BD.

Soluc
ao. Observe que [P AB] [P CD] = [P BC] [P DA] =

7. (AMC) Seja P um ponto no interior de um quadril


atero convexo ABCD, com area
2002, tal que P A = 24, P B = 32, P C = 28 e P D = 45. Determine o permetro de
ABCD.
Soluc
ao. Temos que

1
AC BD,
2
com igualdade acontecendo se, e somente se, AC BD. Temos que
[ABCD]

2002 = [ABCD]

1
1
52 77
AC BD (AP + P C) (BP + P D) =
= 2002.
2
2
2

Portanto as diagonais
AC e BD s
ao perpendiculares
e se intersectam

em P . Dessa

forma, AB = 242 + 322 = 40, BC = 282 + 322 = 4 113, CD = 282 + 452 = 53


12

POT 2012 - Geometria - Nvel 3 - Aula 6 - Prof. Ccero Thiago

e DA =

452 + 242 = 51. Assim, o permetro de ABCD e 144 + 4 113.

8. (OCM) Seja P QRS um quadril


atero convexo de area A e O um ponto em seu interior.
Prove que se 2A = OP 2 + OQ2 + OR2 + OS 2 , entao P QRS e um quadrado e O e o
seu centro.
Soluc
ao.
[P QRS] = [P OQ] + [QOR] + [ROS] + [SOP ]
1
1
OP OQ sin (P OQ) + OQ OR sin (QOR)
2
2
1
1
+ OR OS sin (ROS) + OS OP sin (SOP ).
2
2

Usando que sin 1, para todo [0, 360 ], com igualdade ocorrendo se, e somente
x2 + y 2
se, = 90 , e que xy
, para quaisquer reais x e y, com igualdade ocorrendo
2
se, e somente se, x = y, obtemos:
2A OP OQ + OQ OR + OR OS + OS OP

OP 2 + OQ2 OQ2 + OR2 OR2 + OS 2 OS 2 + OP 2


+
+
+
2
2
2
2
= OP 2 + OQ2 + OR2 + OS 2 .

Pelo enunciado, na u
ltima desigualdade ocorre a igualdade. Dessa forma, temos:
P OQ = QOR = ROS = SOP = 90
e
OP = OQ = OR = OS.
Isto significa que P QRS e um quadrado e O e o seu centro.
9. (Rioplatense) Em um tri
angulo ABC, sejam D, E e F pontos sobre os lados BC,
CA e AB, respectivamente, tais que [AF E] = [BF D] = [CDE]. Mostre que
[DEF ]
1
.
[ABC]
4
Soluc
ao.

13

POT 2012 - Geometria - Nvel 3 - Aula 6 - Prof. Ccero Thiago

(1 y)b

zc

E
b

F
b

yb
(1 z)c
b

xa

(1 x)a

Se [AF E] = [BF D] = [CDE] = S entao [DEF ] = [ABC] 3S. Entao,


[DEF ]
1

[ABC]
4
[ABC] 3S
1

[ABC]
4
[ABC] 4S.
Por outro lado,
S = [AF E] =

b c sin A
zc (1 y)b sin A
= z(1 y)
= z(1 y) [ABC].
2
2

Analogamente,
S = [BF D] = x(1 z) [ABC] e
S = [CDE] = y(1 x) [ABC].
Multiplicando as igualdades encontradas temos
S 3 = x(1 x)y(1 y)z(1 z) [ABC]3 .

14

POT 2012 - Geometria - Nvel 3 - Aula 6 - Prof. Ccero Thiago


1
Usando a desigualdade entre as medias e facil provar que x(1x) , com igualdade
4
1
acontecendo se, e somente se, x = .
2
Portanto,
[ABC]3

64
[ABC] 4S.

S3

10. (OCM) Os lados de um tri


angulo s
ao expressos, em cm, por tres inteiros consecutivos
2
e sua area, em cm , e dada por um inteiro. Prove que o menor lado do tri
angulo e
mpar.
Soluc
ao.
Sejam x1, x, x+1 os lados do tri
angulo. Pela formula de Heron, a area do tri
angulo
e
r
3x (x + 2) x (x 2)
[ABC] =


2
2
2
2
r
1p 2 2
3x2 (x2 4)
3x (x 4).
=
=
16
4
Como [ABC] Z, devemos ter 3x2 (x2 4) par, o que nos diz que x deve ser par.
Portanto, o menor lado do tri
angulo, que e x 1, deve ser mpar.
11. (Hong Kong) Seja ABC um tri
angulo e sejam X, Y e Z pontos sobre os lados AB,
AX
4 BY
6 CZ
8
BC e CA, respectivamente, tais que
= ,
= e
= . Se a area do
XB
5 YC
7 ZA
9
tri
angulo ABC e 1989, determine a area do tri
angulo XY Z.
Soluc
ao.


[AXZ] [BXY ] [CY Z]
[XY Z]
=1
+
+
1989
1989
1989
1989


4 9
5 6
7 8
=1

+
+

9 17 9 13 13 17
1482
,
1989
Portanto, a
area do tri
angulo XY Z e 1989 1482 = 507.
1

15

POT 2012 - Geometria - Nvel 3 - Aula 6 - Prof. Ccero Thiago

Exerccios Propostos
1. Num tri
angulo ABC tem - se AB = BC, e D e um ponto sobre a base AC tal que
o raio do crculo inscrito no tri
angulo ABD e igual ao raio do crculo tangente ao
segmento DC e aos prolongamentos das retas BD e BC. Prove que o raio deste
1
angulo ABC.
crculo e igual a da medida h de uma das alturas iguais do tri
4
2. No tri
angulo ABC, os pontos L, M e N est
ao sobre BC, CA e AB respectivamente,
e AL, BM e CN s
ao concorrentes no ponto P .
(a) Encontre o valor numerico de
PL PM
PN
+
+
AL BM
CN
(b) Encontre o valor numerico de
BP
CP
AP
+
+
AL BM
CN
3. (Ibero) Se AD, BE e CF s
ao tres cevianas concorrentes no circuncentro O do
tri
angulo ABC, demonstre que
1
1
2
1
+
+
= .
AD BE CF
R
4. (AIME) Num tri
angulo ABC, A1 , B1 e C1 est
ao sobre os lados BC, CA e AB,
respectivamente. Dado que AA1 , BB1 e CC1 s
ao concorrentes no ponto O, e que
AO
BO
CO
AO BO CO
+
+
= 92. Encontre o valor de

.
OA1 OB1 OC1
OA1 OB1 OC1
5. Em um ABC, AD, BE e CF s
ao concorrentes no ponto P tal que AP = P D = 6,
EP = 3, P B = 9 e CF = 20. Qual e a area do ABC?
6. Em um tri
angulo ABC, sejam S o ponto medio da mediana correspondente ao vertice
A e Q o ponto de intersec
ao de BS com o lado AC. Demonstrar que BS = 3QS.
7. Tres segmentos C1 A2 , C2 B1 e A1 B2 com extremos sobre os lados do tri
angulo ABC
s
ao paralelos aos lados e passam pelo ponto P . Prove que as areas dos tri
angulos
A1 B1 C1 e A2 B2 C2 s
ao iguais.
dado um quadril
8. (OBM) E
atero convexo ABCD. Sejam E, F, G e H os pontos
medios dos lados AB, BC, CD e DA, respectivamente. Determine a posicao de um
ponto P de forma que os quadril
ateros P HAE, P EBF , P F CG e P GDH tenham a
mesma
area.

16

POT 2012 - Geometria - Nvel 3 - Aula 6 - Prof. Ccero Thiago

9. Seja ABCDE um pent


agono convexo (n
ao necessariamente regular) tal que os tri
angulos
ABC, BCD, CDE, DEA e EAB tem area 1. Qual a area do pentagono?
10. Seja ABCD um quadril
atero convexo e EH, EI, EF e EG s
ao segmentos paralelos
e iguais a AB, BC, CD e DA, como mostra a figura abaixo. Determine a raz
ao entre
as areas dos tri
angulos HIF G e ABCD.
G
D

C
F
E

H
A

B
I

11. (AIME) Quadrados S1 e S2 s


ao inscritos em um tri
angulo ret
angulo ABC, como
mostrado na figura abaixo. Determine AC + CB se area(S1 ) = 441 e area(S2 ) = 440.
A

S2

S1
b

12. Seja P um ponto no interior de um tri


angulo equilatero ABC, e sejam D, E e F os
simetricos de P em relac
ao aos lados BC, CA e AB, respectivamente. Qual e maior,
a area do tri
angulo ABC ou a
area do tri
angulo DEF ?

17

POT 2012 - Geometria - Nvel 3 - Aula 6 - Prof. Ccero Thiago


C
E
D
P

B
F

13. (Portugal) Seja [ABC] um tri


angulo ret
angulo em A. Considere um ponto E sobre
a hipotenusa e traca - se a partir desse ponto uma paralela ao cateto AC. Seja a
intersec
ao desta paralela com o cateto AB. Prove que
BC 2
BD DE
+
=
,
DE BD
2S
sendo S a
area do tri
angulo ABC.
14. (Portugal) Os lados AB, BC e AC do tri
angulo representado na figura medem, respectivamente, 7, 11 e 8. Tracam - se W R, U P e V Q, perpendiculares aos lados.
Sabendo que U W mede 2, determine a raz
ao entre a area do tri
angulo U V W e a
area do tri
angulo ABC.
A
b

R
b

U
b

V
b

b
b

18

POT 2012 - Geometria - Nvel 3 - Aula 6 - Prof. Ccero Thiago

15. (OBM) ABCD e um quadril


atero convexo e inscritvel e M e um ponto sobre o lado
CD, tal que o tri
angulo ADM e o quadril
atero ABCM tem a mesma area e o mesmo
permetro. Prove que ABCD tem dois lados de comprimentos iguais.
16. Uma reta corta um quadril
atero circunscritvel em dois polgonos com iguais areas e
permetros. Prove que a reta passa pelo centro da circunferencia inscrita.
17. Os pontos medios das diagonais AC, CE, EA, BD, DF e F B do hexagono convexo
ABCDEF s
ao vertices de um novo hexagono. Calcular a relacao entre as areas do
dois hexagonos.
18. (Mandelbrot) Seja ABCD um quadril
atero convexo tal que AB = 12, BC = 6 e
CD = 20. Suponha que ABCD possui uma circunferencia inscrita que e tangente ao
lado BC em seu ponto medio. Qual e a area do quadril
atero ABCD?
Sugest
oes
1. Use os teoremas 2 e 5.
2. Use o teorema 9.
3. Use o exerccio 2.
4. Use o teorema 9.
5. Use o teorema 9.
6. Use o teorema 9.
7. Use o teorema 8.
8. Use o teorema 8.
9. Use o teorema 8.
10. Use o teorema 8.
11. Use o teorema 7.

19

POT 2012 - Geometria - Nvel 3 - Aula 6 - Prof. Ccero Thiago

12. Use o teorema 3.


13. Use o teorema 7 e o teorema de Pit
agoras.
14. Use os teoremas 3 e 7.
15. Use o teorema 3.
16. Use o teorema 2.
17. Use base media e o teorema 3.
18. Use os teoremas 2 e 6.
Bibliografia
1. Colec
ao Elementos da Matematica, vol.2 - Geometria Plana
Marcelo Rufino de Oliveira e Marcio Rodrigo da Rocha Pinheiro.
2. Olimpadas Cearenses de Matematica, Ensino Medio, 1981 - 1985
Emanuel Carneiro, Francisco Ant
onio M. de Paiva e Onofre Campos.
3. Olimpadas de Matematica, Categoria B, 10 , 11 e 12 anos, vol.1
Jorge Picado e Paulo Eduardo Oliveira.
4. T
opicos de Matematica Elementar, vol.2, Geometria Euclidiana Plana
Antonio Caminha Muniz Neto.
5. Area y Volumen, en la geometria elemental.
Jose Araujo, Guilermo Keilhauer, Norma Pietrocola e Valeri Vavilov.
6. Which Way did the Bicycle Go? And other intriguing mathematical mysteries
Joseph D. E. Konhauser, Dan Velleman e Stan Wagon.
7. 360 Problems for Mathematical Contests
Titu Andreescu e Dorin Andrica.

8. Areas
para achar raz
oes de segmentos
Ccero Thiago e Marcelo Mendes.
20

POT 2012 - Geometria - Nvel 3 - Aula 6 - Prof. Ccero Thiago

Revista Eureka 25
9. Mathematical Olympiad Treasures
Titu Andreescu e Bogdan Enescu
10. Mandelbrot Morsels
Sam Vandervelde.

21

Polos Olmpicos de Treinamento


Aula

Curso de Geometria - Nvel 3

Prof. Ccero Thiago

Teorema de Ceva

Teorema 1. Sejam D, E e F pontos sobre os lados BC, AC e AB, respectivamente, do


tri
angulo ABC. Os segmentos AD, BE e CF intersectam - se em um ponto P se, e
BD CE AF

= 1.
somente se,
CD EA F B
Demonstrac
ao.

A
b

F
b

b
b

Defina K = [ABC], KA = [P BC], KB = [P CA] e KC = [P AB].


Temos que
BD
[ABD]
[BP D]
[ABD] [BP D]
[AP B]
KC
=
=
=
=
=
.
CD
[ACD]
[CP D]
[ACD] [CP D]
[ACP ]
KB
KA AF
KB
BD CE AF
KC KA KB
CE
=
e
=
. Assim,

= 1.
EA
KC F B
KA
CD EA F B
KB KC KA
BD CE AF
Sejam D, E e F pontos sobre os lados BC, CA e AB tais que

=1
CD EA F B
mas AD, BE e CF n
ao s
ao concorrentes. Seja F1 sobre AB tal que AD, BE e CF1 s
ao
De maneira an
aloga,

POT 2012 - Geometria - Nvel 3 - Aula 7 - Prof. Ccero Thiago

concorrentes em P . Assim,

BD CE AF1
AF
AF1

= 1. Dessa forma,
=
F = F1 .
CD EA F1 B
FB
F1 B
A
b

F1
b

F
b

Exerccios resolvidos
1. Prove que as medianas de um tri
angulo s
ao concorrentes em um ponto que se chama
baricentro.
Soluc
ao.
Sejam M , N e R os pontos medios de AC, BC e BA, respectivamente. Entao
AM CN BR

= 1,
M C N B RA
ou seja, AN , BM e CR s
ao concorrentes.
2. Prove que as bissetrizes internas de um tri
angulo s
ao concorrentes em um ponto que
se chama incentro.
Soluc
ao.
Sejam X, Y e Z os pes das bissetrizes relativas aos lados BC, AC e AB, respectivamente. Pelo teorema das bissetrizes internas temos que
AY CX BZ
AB CA BC

= 1,
Y C XB ZA
BC AB CA
ou seja, AX, BY e CZ s
ao concorrentes.
3. Prove que as alturas de um tri
angulo s
ao concorrentes em um ponto que se chama
ortocentro.

POT 2012 - Geometria - Nvel 3 - Aula 7 - Prof. Ccero Thiago

Soluc
ao.

A
b

M
b

N
b

b
b

b
b

facil ver que


Sejam AL, BM e CN as alturas do tri
angulo ABC. E
AN C AM B

AC
AN
=
(I)
MA
AB

BL
AB
=
(II)
NB
BC
CM
BC
CM B CLA
=
(III).
LC
AC
BLA BN C

Multiplicando (I), (II) e (III) temos que


AN BL CM
AC AB BC

= 1,
M A N B LC
AB BC AC
ou seja, as alturas s
ao concorrentes.
4. Seja ABCDEF um hexagono convexo tal que cada uma das diagonais AD, BE e
CF dividem o hexagono em duas regi
oes de areas iguais. Prove que AD, BE e CF
s
ao concorrentes.
Soluc
ao.

POT 2012 - Geometria - Nvel 3 - Aula 7 - Prof. Ccero Thiago

F
b
b

A
Y
b

b
b

X
b
b

Sejam X a intersecc
ao de AD e CE, Y a interseccao de AE e CF e Z e a interseccao
de AC e BE. Denotaremos por [M N P ] a area do tri
angulo M N P , e seja K a area
facil ver que
do hexagono ABCDEF . E
K
[ABC]
[ACX]
[CDX]
[ACX] + [CDX]
[ACD]
CX
=
=
=
=
= 2
.
K
XE
[AXE]
[DEX]
[AXE] + [DEX]
[ADE]
[AEF ]
2
De maneira an
aloga,
K
[CDE]
EY
= 2
K
YA
[ABC]
2
e
K
[AF E]
AZ
.
= 2
K
ZC
[CDE]
2
Portanto,
K
K
K
[ABC]
[CDE]
[AF E]
CX EY AZ
2
2

2
= 1.

=
K
K
K
XE Y A ZC
[AEF ]
[ABC]
[CDE]
2
2
2
4

POT 2012 - Geometria - Nvel 3 - Aula 7 - Prof. Ccero Thiago

Pela recproca do teorema de Ceva no tri


angulo ACE temos que AX, CY e EZ
s
ao concorrentes e, com isso, AD, BE e CF s
ao concorrentes.
5. Seja ABC um tri
angulo e seja AD uma altura, com D em BC. Sejam E e F pontos
sobre AC e AB, respectivamente, tais que AD, BE e CF s
ao concorrentes. Entao a
medida dos
angulos EDA = F DA s
ao iguais.
Soluc
ao.
r

Q
b

P
b

b
b

Seja r uma reta que passa por A e e paralela BC. Sejam Q e P as interseccoes de
facil ver que BF D AF P assim
DE e DF com r, respectivamente. E
AP
BD AF
BD
=
AP =
(1)
BF
AF
BF
e CED AEQ, ou seja,
CD
AQ
CD AE
=
AQ =
. (2)
CE
AE
CE
Por outro lado, pelo teorema de Ceva, aplicado ao tri
angulo ABC de cevianas
concorrentes AD, BE e CF ,
AF BD CE

=1
BF CD AE

BD AF
CD AE
=
.
BF
CE

Da u
ltima igualdade e de (1) e (2), temos que AP = AQ, ou seja, o tri
angulo
DQP e is
osceles e, com isso, a altura DA ser
a bissetriz do angulo QDP , entao
ADE = ADF .

POT 2012 - Geometria - Nvel 3 - Aula 7 - Prof. Ccero Thiago

6. Seja ABC um tri


angulo e sejam P e Q pontos sobre os lados AB e AC, respectivamente, tais que P Q k BC. Prove que P C, QB e a mediana AM , com M em BC,
s
ao concorrentes.
Soluc
ao.
Como P Q k BC, ent
ao
AP
AQ
AP QC
=

= 1 (I).
PB
QC
P B AQ
Como AM e um mediana ent
ao BM = M C, assim
BM
= 1 (II).
MC
Multiplicando (I) e (II), temos
AP QC BM

= 1.
P B AQ M C
Pela recproca do teorema de Ceva temos que AM , QB e P C s
ao concorrentes.
b

P
b

b
b

Exerccios propostos
1. Sejam D, E e F os pontos de contato da circunferencia inscrita com os lados BC,
CA e AB, respectivamente, do tri
angulo ABC. Prove que AD, BE e CF s
ao concorrentes em um ponto que se chama Ponto de Gergonne.

POT 2012 - Geometria - Nvel 3 - Aula 7 - Prof. Ccero Thiago

2. Sejam l e l1 duas retas paralelas dadas no plano. Usando apenas regua encontre o
ponto medio do segmento AB que est
a na reta l.
3. Seja P um ponto no interior de um tri
angulo acut
angulo ABC e sejam D, E e
F os pontos de intersecc
ao das retas AP , BP e CP com os lados BC, CA e AB,
respectivamente. Determine P de maneira que a area do tri
angulo DEF seja m
axima.
4. (Coreia) Seja ABC um tri
angulo com AB 6= AC, seja V a interseccao da bissetriz
do angulo A com BC e seja D pe da altura relativa ao vertice A. Se E e F s
ao as
intersecc
oes dos crculos circunscritos aos tri
angulos AV D com CA e AB, respectivamente, mostre que AD, BE e CF s
ao concorrentes.
5. Seja P um ponto no interior de um tri
angulo. As bissetrizes de BP C, CP A e
AP B intersectam BC, CA e AB em X, Y e Z, respectivamente. Prove que AX,
BY e CZ s
ao concorrentes.

Sugest
oes/Solu
co
es
2. Use o exerccio resolvido 6.
Bibliografia
1. Advanced Euclidean Geometry
Alfred Posamentier
2. Geometric Transformations III
I. M. Yaglom
3. Methods of Problem Solving, Book 3
JB Tabov, EM Kolev e PJ Taylor
4. III Olimpiada Nacional Escolar de Matematica 2006
Jorge Tipe, John Cuya, Claudio Espinoza e Sergio Vera.

Polos Olmpicos de Treinamento


Aula

Curso de Geometria - Nvel 3

Prof. Ccero Thiago

Teorema de Menelaus e problemas de colinearidade


Teorema 1. Se uma reta intersecta as retas BC, CA e AB de um tri
angulo ABC nos
pontos L, M e N , respectivamente, entao
CL BN AM

= 1.
BL N A M C
Inversamente, se L, M e N s
ao pontos sobre os lados BC, CA e AB do tri
angulo ABC
CL BN AM

= 1, ent
ao L, M e N s
ao colineares.
tais que
BL N A M C
Demonstrac
ao.
A
b

Q
b

N
b
b

P
b

M
b

Sejam AP , BQ e CR as perpendiculares tracadas a partir de A, B e C, respectivamente,


facil ver que os tri
`a reta em que se encontram L, M e N . E
angulos ret
angulos AP N e
BQN s
ao semelhantes, assim como os tri
angulos ret
angulos QBL e RCL. Entao
BQ CL
RC
BN
=
e
=
.
AN
AP
BL
QB
Por outro lado, os tri
angulos ret
angulos AP M e CRM tambem s
ao semelhantes. De modo
que
AM
AP
=
.
CM
CR

POT 2012 - Geometria - Nvel 3 - Aula 8 - Prof. Ccero Thiago

Portanto,
BQ RC AP
BN CL AM

= 1.
AN BL CM
AP QB CR

A
b

N
b

N1
b

CL BN AM
Suponha, de maneira falsa, que

= 1 e os pontos L, M e N n
ao s
ao
BL N A M C
colineares. Prolongue LM ate intersectar AB em N1 . Pelo que foi provado acima temos
CL BN1 AM

= 1, assim
que
BL N1 A M C
BN
BN1
=
N = N1 .
N1 A
NA
Dessa forma, L, M e N s
ao colineares.
Teorema 2. (Pascal) Seja ABCDEF um hexagono inscrito em um crculo e sejam H, K
e I os pontos de intersecc
ao de AB e ED, BC e F E e AF e CD, respectivamente. Entao,
H, K e I s
ao colineares.
Demonstrac
ao. As retas AB, CD e EF determinam o tri
angulo XY Z. Considere as
retas AF I, BCK e HDE que cortam as retas que formam o tri
angulo XY Z. Aplicando o
teorema de Menelaus, temos
XA ZF Y I

= 1,
AZ F Y IX
XB ZK Y C

= 1,
BZ KY CX
XH ZE Y D

= 1.
HZ EY DX
Multiplicando as tres igualdades e considerando que XA XB = XC XD =, Y C Y D =
Y E Y F e ZE ZF = ZAZB(provaremos a validade destas igualdades na aula de Potencia

POT 2012 - Geometria - Nvel 3 - Aula 8 - Prof. Ccero Thiago


Y I XH ZK

= 1. Pelo teorema de Menelaus temos que


de Ponto e Eixo radical), obtemos
IX HZ KY
I, K e H s
ao colineares. O teorema de Pascal permite variacoes como o exerccio resolvido 2.

b
b

E
b

b
b
b

B
b
b

K
H
b

Exerccios resolvidos
1. Dadas tres circunferencias C1 , C2 e C3 de centros O1 , O2 e O3 e raios r1 , r2 e r3 ,
respectivamente. Seja X a interseccao das tangentes comuns externas de C1 e C2 ,
Y a intersecc
ao das tangentes comuns externas de C1 e C3 e, finalmente, Z a interseccao das tangentes comuns externas de C2 e C3 . Prove que X, Y e Z s
ao colineares.
facil verificar que X, O1 e O2 s
Soluc
ao. E
ao colineares. Assim, XO1 P1 XO2 P2
O1 P1
r3 O2 Z
r2
r1
O3 Y
O1 X
=
=
= . Portanto,
= . Analogamente,
e
e, com isso,
O2 X
O2 P2
r2
O1 Y
r1 O3 Z
r3
O1 X O3 Y O2 Z

= 1.
O2 X O1 Y O3 Z
Pela recproca do teorema de Menelaus conclumos que X, Y e Z s
ao colineares. Este
resultado e conhecido como teorema de Monge.

POT 2012 - Geometria - Nvel 3 - Aula 8 - Prof. Ccero Thiago

Z
b

Y
b

O2
O1

b
b

X
b

P1
b

P2

O3
b

2. Seja P um ponto no interior do tri


angulo ABC. Sejam M e N as projecoes de P sobre
AB e AC, respectivamente. Seja K a projecao de A sobre CP e seja L a projecao de
A sobre BP . Prove que KM , LM e BC s
ao concorrentes.
Soluc
ao.
facil ver que A, K, M , P , N e L s
E
ao concclicos. No hexagono AKM P N L temos
que
AM LP = B,
AN KP = C,
KM LN = Q.

POT 2012 - Geometria - Nvel 3 - Aula 8 - Prof. Ccero Thiago

Pelo teorema de Pascal temos que B, C e Q s


ao colineares, ou seja, KM , LM e BC
s
ao concorrentes.
b

b
b

b
b

3. Prove que as bissetrizes internas de dois angulos de um tri


angulo is
osceles e a bissetriz
externa do terceiro
angulo do tri
angulo intersectam os lados opostos em tres pontos
colineares.
Soluc
ao.
No tri
angulo ABC, BM e CN s
ao bissetrizes internas dos angulos B e C, respectivamente, e AL e a bissetriz externa do angulo A. Pelo teorema da bissetriz
interna temos que
AB BN
BC
AM
=
e
=
.
MC
BC N A
AC
Alem disso, pelo teorema da bissetriz externa temos que
CL
AC
=
.
BL
AB
Assim,

POT 2012 - Geometria - Nvel 3 - Aula 8 - Prof. Ccero Thiago

AB BC AC
AM BN CL

= 1.
M C N A BL
BC AC AB
Pela recproca do teorema de Menelaus temos que N , M e L s
ao colineares.

A
b

N
b

4. Seja G o baricentro do tri


angulo ABC e sejam AM , BN e CK as bissetrizes internas
com M em BC, N em AC e K em AB. Prove que uma das alturas do tri
angulo ABC
e igual a soma das outras duas se, e somente se, G pertence a um lado do tri
angulo
M N K.
Soluc
ao. Vamos supor que o baricentro G pertence ao lado N K do tri
angulo M N K.
Seja X AN o ponto medio de AC.
B
b

K
b

b
b

Considere o tri
angulo ABX, em que K, G e N s
ao colineares e pertencem, respectivamente, `
as retas AB, BX e XA. Pelo teorema de Menelaus temos
AK BG XN

= 1. (1)
KB GX N A
Alem disso, pelo toerema da bissetriz interna e por G ser o baricentro temos
AC BG
AK
=
e
= 2. (2)
KB
BC GX
6

POT 2012 - Geometria - Nvel 3 - Aula 8 - Prof. Ccero Thiago

Se BC = a, AC = b e AB = c e substituindo (2) em (1) temos


XN
b
2
= 1. (3)
a
NA
Temos que AN =

bc
pelo teorema da bissetriz interna. Por outro lado,
a+c
XN = AN AX =

bc
b
bc ba
=
.
a+c 2
2(a + c)

Entao,
XN
bc ba a + c
ca
=

=
.
AN
2(a + c)
bc
2c
Substituindo em (3) temos
1 =2

b ca

ac = bc ab
a
2c
bc = ac + ab. (4)

Se s = [ABC] e hA , hB e hC s
ao as alturas do tri
angulo ABC correspondentes aos
vertices A, B e C temos que
2s = a hA = b hB = c hC .
Entao, se em (4) multiplicarmos por

s
, temos que
abc
s
s s
= + ,
a
b c

ou seja,
hA = hB + hC .
Reciprocamente, se hA = hB + hC , fazendo o processo inverso chegamos em
AK BG XN

= 1.
KB GX N A
Assim, pelo teorema de Menelaus temos que K, G e N s
ao colineares.

POT 2012 - Geometria - Nvel 3 - Aula 8 - Prof. Ccero Thiago

5. (Reta de Newton) Seja ABCD um quadril


atero tal que BA e CD intersectam - se
em E, AD e BC intersectam - se em F e sejam N , L e M os pontos medios de EF ,
AC e BD, respectivamente. Prove que N , L e M s
ao colineares.
Soluc
ao.
b

A
b

P
b

D
b

L
b

Sejam P , Q e R os pontos medios de EB, EC, BC, respectivamente. Pelo teorema


da base media temos que Q, L e R s
ao colineares e
EA
QL
=
.
LR
AB
Da mesma forma, P , M R e R s
ao colineares e
CD
RM
=
,
MP
DE
e N , Q e P s
ao colineares e
PN
BF
=
.
NQ
FC
Aplicando o teorema de Menelaus no tri
angulo EBC cortado pela transversal ADF
temos
EA BF CD

= 1,
AB F C DE
QL RM P N
EA CD BF

= 1. Pelo teorema de Menelaus apliLR M P N Q


AB DE F C
cado ao tri
angulo P QR e os pontos M , L e N conclumos que M , L e N s
ao colineares.
portanto

Exerccios propostos

POT 2012 - Geometria - Nvel 3 - Aula 8 - Prof. Ccero Thiago

1. Prove que as bissetrizes externas dos angulos de um tri


angulo, n
ao is
osceles, intersectam os lados opostos em tres pontos colineares.
2. O ortocentro de um tri
angulo ABC e o ponto medio da altura relativa ao vertice
C. Prove que cos C = cos A cos B, em que A, B e C s
ao os angulos do
tri
angulo ABC.
3. A bissetriz AD de um tri
angulo ABC divide o lado BC na raz
ao 2 : 1. Determine a
raz
ao em que a mediana CE divide a bissetriz.
4. (Macedonia) Seja a circunferencia circunscrita ao tri
angulo ABC. Seja D a interseccao da tangente `
a , em A, com o lado BC, E a interseccao da tangente `a , em
B, com o lado AC e F a interseccao da reta tangente `a , em C, com o lado AB.
Prove que D, E e F s
ao colineares.
5. (OBM) No tri
angulo ABC, D e ponto medio de AB e E ponto sobre o lado BC tal
que BE = 2 EC. Sabendo que ADC = BAE, calcule o valor de BAC.
6. (IMO) As diagonais AC e CE de um hexagono regular ABCDEF s
ao divididas interCN
AM
=
= r. Determine
namente pelos pontos M e N , respectivamente, na raz
ao
AC
CE
r se B, M e N s
ao colineares.
7. Seja ABC um tri
angulo e sejam E e D pontos sobre o lado BC tal que CE = ED =
DB. Seja F o ponto medio de AC e G o ponto medio de AB. Seja H a interseccao
EH
de EG e F D. Determine o valor de
.
HG
8. Seja ABCD um trapezio com AB k CD e seja X um ponto no segmento AB. Se P
e a intersecc
ao de BC e AD, Y a interseccao de CD e P X, R a interseccao de AY e
1
1
1
=
+
.
BD e T a intersecc
ao de P R e A. Prove que
AX
AX
AB
9. (Cone Sul) Seja C uma circunferencia de centro O, AB um di
ametro dela e R um
ponto qualquer em C distinto de A e de B. Seja P a interseccao da perpendicular
tracada por O a AR. Sobre a reta OP se marca o ponto Q, de maneira que QP e a
metade de P O e Q n
ao pertence ao segmento OP . Por Q tracamos a paralela a AB
que corta a reta AR em T . Chamamos de H o ponto de interseccao das retas AQ e
OT . Provar que H, R e B s
ao colineares.

POT 2012 - Geometria - Nvel 3 - Aula 8 - Prof. Ccero Thiago

10. Seja ABCD um quadril


atero. Seja P a interseccao de BC e AD, Q a interseccao de
CA e BD e R a intersecc
ao de AB e CD. Prove que os pontos de interseccao de BC
e QR, de CA e RP e de AB e P Q s
ao colineares.
Bibliografia
1. Leccture Notes on Mathematical Olympiad Courses
For senior Section, vol. 1
Xu Jiagu
2. Advanced Euclidean Geometry
Alfred Posamentier
3. III Olimpiada Nacional Escolar de Matematica 2006
Jorge Tipe, John Cuya, Claudio Espinoza e Sergio Vera.
4. Explorations in Geometry
Bruce Shawyer
5. Colec
ao Elementos de Matematica, vol.2
Marcelo Rufino de Oliveira
6. The theorem of Menelaus
B. Orach
Quantum - May/Jun 2001
7. Problemas de Geometra - Planimetria
I. Shariguin

10

Polos Olmpicos de Treinamento


Aula

Curso de Geometria - Nvel 3

Prof. Ccero Thiago

Relac
oes m
etricas no tri
angulo.

Teorema 1. (Lei dos Senos) Seja ABC um tri


angulo tal que BC = a, CA = b e AB = c.
Seja R o raio da circunferencia circunscrita. Entao
b
c
a
=
=
= 2R.
sin A
sin B
sin C
Demonstrac
ao.
A
b

b
O
b

B
b

facil ver que ABC = ADC. Assim, no tri


Seja AD um di
ametro. E
angulo ADC,
b
b
sin B =

= 2R. Analogamente,
2R
sin B
c
a
=
= 2R.
sin A
sin C

POT 2012 - Geometria - Nvel 3 - Aula 9 - Prof. Ccero Thiago

Finalmente,
b
c
a
=
=
= 2R.
sin A
sin B
sin C
Teorema 2. (Lei dos Cossenos) Seja ABC um tri
angulo tal que BC = a, CA = b e
AB = c. Ent
ao,
a2 = b2 + c2 2bc cos A,
b2 = a2 + c2 2ac cos B,
c2 = a2 + b2 2ab cos C.

Demonstrac
ao.

A
b

B
m

am

Vamos fazer o caso em que o tri


angulo e acut
angulo. O caso em que o tri
angulo e obtus
angulo fica como exerccio. Aplicando o teorema de Pit
agoras nos tri
angulos ABD e
ACD, temos:
c2 = m2 + H 2 e
b2 = (a m)2 + H 2

b2 = a2 2am + m2 + H 2 .
m
Assim, b2 = a2 + c2 2am. Por outro lado, cos B =
m = c cos B. Finalmente,
c
2
2
2
b = a + c 2ac cos B. Analogamente,
a2 = b2 + c2 2bc cos A e
c2 = a2 + b2 2ab cos C.
Teorema 3. (Stewart) Seja ABC um tri
angulo tal que BC = a, CA = b e AB = c. Seja
D um ponto sobre o lado BC tal que BD = x, CD = y e AD = z. Entao,
c2 y + b2 x z 2 a = axy.
2

POT 2012 - Geometria - Nvel 3 - Aula 9 - Prof. Ccero Thiago

Demonstrac
ao.
A
b

c
z

180
b

Aplicando a lei dos Cossenos nos tri


angulos ABD e ACD, temos
c2 = x2 + z 2 2xz cos(180 )
c2
z2
=x+
2z cos(180 ). (1)
x
x
E
b2 = y 2 + z 2 2yz cos

z2
b2
=y+
2z cos . (2)
y
y

Adicionando (1) e (2), encontramos


b2 c2
z2 z2
+
=x+y+
+

y
x
y
x
z2 z2
b2 c2
+
=a+
+

y
x
y
x
c2 y + b2 x z 2 a = axy.
Teorema 4. (Ceva trigonom
etrico) Seja ABC um tri
angulo e sejam D, E e F pontos
sobre os lados BC, CA e AB, respectivamente. Entao, AD, BE e CF s
ao concorrentes se,
e somente se,
sin CAD sin ABE sin BCF

= 1.
sin BAD sin CBE sin ACF
Demonstrac
ao.
Suponha que AD, BE e CF s
ao concorrentes em P .

POT 2012 - Geometria - Nvel 3 - Aula 9 - Prof. Ccero Thiago

A
b

E
b

P
b

Aplicando lei dos senos nos tri


angulos ABP , BCP e CP A, respectivamente, temos
1)

BP
AP
sin ABE
AP
=

=
.
sin BAD
sin ABE
sin BAD
BP

CP
sin BCF
BP
BP
=

=
.
sin BCF
sin CBE
sin CBE
CP
AP
sin CAD
CP
CP
=

=
.
3)
sin CAD
sin ACF
sin ACF
AP

2)

Portanto,

sin ABE sin BCF sin CAD


AP BP CP

= 1.
BP CP AP
sin BAD sin CBE sin ACF
sin CAD sin ABE sin BCF
Para demonstrar a recproca, ou seja, se

= 1 entao
sin BAD sin CBE sin ACF
AD, BE e CF s
ao concorrentes, usaremos o seguinte
Lema: Seja ABC um tri
angulo e AD uma ceviana qualquer. Entao,
BD
AB sin BAD
=

.
CD
AC sin CAD
Demonstrac
ao.
A
b

POT 2012 - Geometria - Nvel 3 - Aula 9 - Prof. Ccero Thiago

Aplicando a lei dos senos nos tri


angulos ABD e ACD, respectivamente, temos
BD
AB
=
sin BAD
sin ADB
AC
CD
=
.
2)
sin CAD
sin ADC
Por outro lado, sin ADB = sin ADC pois ADB + ADC = 180 .
1)

Assim,

AB sin BAD
BD
=

.
CD
AC sin CAD

De maneira an
aloga, sejam BE e CF cevianas quaisquer, entao
CE
BC sin CBE
=

,
EA
AB sin ABE
AF
AC sin ACF
=

.
FB
BC sin BCF
BD CE AF

= 1. Pela recproca do
Multiplicando todas as igualdades encontramos
CD EA F B
teorema de Ceva, segue o resultado.
Exerccios resolvidos

1. (China Western) Em um trapezio ABCD, AD//BC . Sejam E um ponto variando


sobre o lado AB, O1 e O2 os circuncentros dos tri
angulos AED e BEC, respectivamente. Prove que o comprimento de O1 O2 e fixo.
Soluc
ao.
A
b
b

E
b

O1

O2

facil ver que AEO1 = 90 ADE e BEO2 = 90 BCE. Entao,


E
O1 EO2 = ADE + ECB.
5

POT 2012 - Geometria - Nvel 3 - Aula 9 - Prof. Ccero Thiago

Como AD k BC, construa uma paralela a AD, por E. Dessa forma DEC =
ADE + BCE, ou seja, O1 EO2 = DEC. Usando lei dos senos, temos
DE
2O1 E sin A
O1 E
=
=
.
EC
2O2 E sin B
O2 E
Assim, DEC O1 EO2 . Portanto,
O1 O2
O1 E
O1 E
1
=
=
=
.
DC
DE
2O1 E sin A
2 sin A
Portanto, O1 O2 =

DC
, que e um valor fixo.
2 sin A

2. Seja ABCD um quadril


atero inscrito em uma circunferencia de di
ametro AD. Se
AB = BC = 1 e AD = 3, ache o comprimento da corda CD.
Soluc
ao.

C
b

D
b

Temos que AD = 3, AB = BC = 1. Aplicando o teorema de Pit


agoras no tri
angulo
ABD, temos

AD 2 = AB 2 + BD 2 32 = 11 + BD 2 BD = 2 2.
6

POT 2012 - Geometria - Nvel 3 - Aula 9 - Prof. Ccero Thiago

BD
2 2
Alem disso, cos =
=
. Aplicando a lei dos cossenos no tri
angulo BCD,
AD
3
temos
BC 2 = BD 2 + CD 2 2 BD CD cos

2 2
2
2

1 = 8 + CD 2 2 2 CD
3
7
CD = 3 ou .
3
Como o di
ametro mede 3, ent
ao CD =

7
.
3

3. (Teste de selec
ao do Brasil para a Cone Sul) Em um tri
angulo acut
angulo ABC,
A = 30 , H e seu ortocentro e M e o ponto medio de BC. Sobre a reta HM
tomemos um ponto T 6= H tal que HM = M T . Mostre que AT = 2BC.
Soluc
ao.
C
b

H
b

M
b

T
b

HBT C e um paralelogramo pois M e o ponto medio de BC e HM = M T . Alem


disso, BC AC e BH k AC, assim CT AC, ou seja, T CA = 90 . Com
isso, T pertence `
a circunferencia circunferencia circunscrita a ABC e AT e di
ametro.
Portanto,
BC
BC
=
= 2BC.
AT = 2R =
sin A
sin 30
4. Seja ABC um tri
angulo com BAC = 40 e ABC = 60 . Sejam D e E pontos
sobre os lados AC e AB, respectivamente, tais que CBD = 40 e BCE = 70 e
F a intersec
ao de BD e CE. Prove que AF BC.
Soluc
ao.

POT 2012 - Geometria - Nvel 3 - Aula 9 - Prof. Ccero Thiago

A
b

E
b

b
b

facil ver que BAG = 30 e, com


Seja G o pe da altura relativa ao lado BC. E
isso, CAG = 10 . Como CBD = 40 , entao ABD = 20 . Alem disso, como
BCE = 70 , ent
ao ACE = 10 . Aplicando o teorema de Ceva trigonometrico
temos
sin 10 sin 20 sin 70
2 sin 20 cos 20
sin CAG sin ABD sin BCE

=
= 1.
sin BAG sin CBD sin ACE
sin 30 sin 40 sin 10
sin 40
Portanto, AG, BD e CE s
ao concorrentes.
Exerccios propostos
1. Seja ABC um tri
angulo tal que ABC = 45o . Seja D o ponto sobre o segmento BC
tal que 2BD = CD e DAB = 15o . Determine o angulo ACB.
2. (AIME) Seja ABC um tri
angulo tal que AB = 13, BC = 15 e CA = 14. Seja D o
ponto do segmento BC tal que CD = 6. Seja E o ponto de BC tal que CE > CD e
BAE = CAD. Determine BE.
3. (OCM) Determine a
area de um hexagono convexo que est
a inscrito em um crculo e
tem tres lados consecutivos iguais a 3 cm e os outros tres com comprimentos iguais a
2 cm.
4. (OCM) As retas r, s e t s
ao paralelas. A reta s est
a situada entre r e t de tal modo
que a dist
ancia de s a r e 3m e a dist
ancia de s a t e 1m. Calcule a area de um
8

POT 2012 - Geometria - Nvel 3 - Aula 9 - Prof. Ccero Thiago

tri
angulo equilatero onde os vertices se encontram sobre cada uma das tres retas.
5. Em um tri
angulo ABC, BAC = 100o e AB = AC. Seja BD a bissetriz de ABC,
com D sobre o lado AC. Prove que AD + BD = BC.
6. Os lados a > b >c de
tri
a
ngulo est
ao em P.A. de raz
ao k > 0.
 um 
C
1
A
tg
= .
(i) Prove que tg
2
2
3
2k
  .
(ii) Se r e o inraio, prove que r =   
C
A
tg
3 tg
2
2
7. Seja P um ponto no interior do tri
angulo ABC tal que P AB = 10 , P BA = 20 ,

P CA = 30 e P AC = 40 . Prove que o tri


angulo ABC e is
osceles.
8. Seja ABC um tri
angulo, prove que r = 4R sin

B
C
A
sin sin .
2
2
2

9. Seja ABC um tri


angulo tal que max{A, B} = C +30 . Prove que ABC e um tri
angulo
R
= 3 + 1.
ret
angulo se, e somente se,
r
10. (IMO) Seja I o incentro do tri
angulo ABC. Sejam K, L, M os pontos onde o crculo
inscrito em ABC toca os lados BC, CA e AB, respectivamente. A reta paralela a
M K passando por B encontra as retas LM e LK em R e S, respectivamente. Mostre
que o angulo RIS e agudo.
11. Um tri
angulo equilatero ABC tem lado 2 cm e e a sua circunfer
ancia inscrita.
Demonstre que para todo ponto de a soma dos quadrados de suas dist
ancias aos
vertices A, B e C e igual a 5.
12. (IMO) Em um tri
angulo ABC a bissetriz do angulo BCA intersecta o crculo circunscrito do tri
angulo ABC novamente no ponto R, a mediatriz de BC em P , a
mediatriz de AC em Q. O ponto medio de BC e K e o ponto medio de AC e L.
Prove que os tri
angulos RP K e RQL tem a mesma area.
13. IMO Shortlist)Seja A1 o centro de um quadrado inscrito em um tri
angulo acut
angulo
ABC, com dois de seus vertices sobre o lado BC e os outros dois vertices, est
ao sobre
os lados AB e AC. Pontos B1 e C1 s
ao definidos de maneira similar, inscrevendo
quadrados com dois de seus vertices sobre os lados AC e AB, respectivamente. Prove
9

POT 2012 - Geometria - Nvel 3 - Aula 9 - Prof. Ccero Thiago

que as retas AA1 , BB1 e CC1 s


ao concorrentes.
14. (Teste de selec
ao do Brasil para a IMO)Seja uma circunferencia de centro O tangente aos lados AB e AC do tri
angulo ABC nos pontos E e F . A reta perpendicular
ao lado BC por O intersecta EF no ponto D. Mostre que A, D e M (ponto medio
de BC) s
ao colineares.

Bibliografia
1. 103 Trigonometry Problems - From the training of the USA IMO team
Titu Andreescu
2. Precalculus
Richard Rusczyk
3. Olimpadas de Matematica 97
Antonio Caminha, Onofre Campos e Paulo Rodrigues
4. Olimpadas Cearenses de Matematica, Ensino Medio, 1981 - 1985
Emanuel Carneiro, Francisco Ant
onio M. de Paiva e Onofre Campos.

10

Polos Olmpicos de Treinamento


Aula

Curso de Geometria - Nvel 2

10

Prof. Ccero Thiago

Pot
encia de ponto
1. Defini
c
ao
Seja uma circunferencia de centro O e raio R. Seja P um ponto que est
a a uma dist
ancia
d de O, vamos definir a potencia do ponto P em relacao `a circunferencia por
PotP = d2 R2 .

facil ver que se P e um ponto no exterior de entao a potencia ser


E
a positiva, se P e um
ponto sobre a circunferencia ent
ao sua potencia ser
a zero e se P e um ponto no interior da
circunferencia ent
ao sua potencia ser
a negativa.
Teorema 1. Seja P um ponto e uma circunferencia. Se uma reta que passa por P
intersecta a circunferencia nos pontos A e B, entao o produto P A P B e constante.
Demonstrac
ao.

1 caso: P e um ponto no exterior.

m
m

M
b

POT 2012 - Geometria - Nvel 3 - Aula 10 - Prof. Ccero Thiago

Seja OM a mediatriz de AB. Ent


ao
P A P B = (P M m) (P M + m) = P M 2 m2 = P M 2 + OM 2 (OM 2 + m2 )
= P O 2 R2 = PotP .
Vamos analisar tambem o caso em que pelo ponto P e tracada uma tangente a .

T
b

O
b

Dessa forma pelo teorema de Pit


agoras temos que
P O 2 = P T 2 + R2 P T 2 = P O 2 R2 = PotP .
2 caso: P e um ponto no interior.

POT 2012 - Geometria - Nvel 3 - Aula 10 - Prof. Ccero Thiago

m
b

A
b

P
b

Seja OM a mediatriz de AB. Ent


ao
P A P B = (m P M ) (m + P M ) = m2 P M 2 = m2 + OM 2 (OM 2 + P M 2 )
= R2 P O 2 = PotP .
Problema 1. Dois crculos 1 e 2 intersectam - se em P e Q. Uma reta passando por P
intersecta 1 e 2 novamente em A e B, respectivamente, se X e o ponto medio de AB e a
reta que passa por Q e X intersecta 1 e 2 novamente em Y e Z, respectivamente. Prove
que X e o ponto medio de Y Z.
Soluc
ao.
PotX
2 = XP XB = XZ XQ,
PotX
1 = XP XA = XY XQ.
Entao,
XZ XQ
XP XB
=

XP XA
XY XQ
XY = XZ.

POT 2012 - Geometria - Nvel 3 - Aula 10 - Prof. Ccero Thiago

1
b

2
P

X
A

b
b

O1
b

Z
b

O2

Problema 2. (OCM) Duas tangentes OA e OB s


ao tracadas a um crculo de um ponto
externo O. Uma corda AC e construda paralela a OB e uma secante OC e desenhada
intersectando o crculo em E. Se K e o ponto de intersecao de OB com o prolongamento
de AE, prove que OK = KB.
Soluc
ao.
Temos que KOC = ECA pois OB k AC e ECA = EAO pois OA e tangente ao
crculo. Entao OKE AKO assim
KE
OK
=
OK 2 = KE KA.
KA
OK

Usando a potencia de K com relac


ao `
a circunferencia temos
KB 2 = KE KA.
Portanto, OK = KB.

POT 2012 - Geometria - Nvel 3 - Aula 10 - Prof. Ccero Thiago

A
b

E
b

Problema 3. Seja ABCD um quadril


atero inscrito em um semicrculo s de di
ametro AB.
As retas AC e BD se intersectam em E e as retas AD e BC em F . A reta EF intersecta
o semicrculo s em G e a reta AB em H. Prove que E e o ponto medio do segmento GH
se, e somente se, G e o ponto medio do segmento F H.
Soluc
ao.
Como AC e BD s
ao alturas do tri
angulo ABF entao E e o ortocentro desse tri
angulo.
Assim, F E e perpendicular a AB. Os tri
angulos HEB e HAF s
ao semelhantes, temos que
HE
HB
2
=
. Ent
ao, HE HF = HA HB = HG e a equivalencia e clara.
HA
HF

POT 2012 - Geometria - Nvel 3 - Aula 10 - Prof. Ccero Thiago

F
b

G
b

D
b

E
b

b
b

Problema 4. Seja C uma semicircunferencia de centro O e di


ametro AB e D e o ponto
medio do arco AB. Sobre a reta OD toma - se o ponto E, do mesmo lado de D com relacao
a AB, tal que OE = BD. Se BE corta a semicircunferencia em F e P e o ponto de AB
AB
.
tal que F P e perpendicular a AB. Prove que BP =
3
Soluc
ao.

Sem perda
de
generalidade
fa
c
a
OA
=
OB
=
1.
Logo,
OD
=
1,
OE
=
BD
=
2 e

ametro AB temos
EB = 3. Utilizando a potencia de E com relacao `a circunferencia de di
EF EB = EO 2 R2 = EO2 1.
Assim,
EF

3 = ( 2)2 1 EF =

2 3
3
e FB =
.
3
3

Alem disso, BP F BOE ent


ao
BF
2
BP
=
BP = .
BO
BE
3
Portanto,

2
1
AB
BP
= 3 = BP =
.
AB
2
3
3
6

POT 2012 - Geometria - Nvel 3 - Aula 10 - Prof. Ccero Thiago

E
b

F
b

b
b

Exerccios propostos
1. Em um tri
angulo ABC, a bissetriz do angulo A e a mediana relativa a BC intersectam este lado em pontos distintos O e M , respectivamente. O crculo circunscrito ao
tri
angulo AOM intersecta os lados AB e AC em E e F , respectivamente. Prove que
BE = CF .
2. Seja BD a bissetriz do
angulo B do tri
angulo ABC. Se o crculo circunscrito ao
tri
angulo BDC intersecta AB em E e o crculo circunscrito ao tri
angulo ABD intersecta BC em F , prove que AE = CF .
3. Um tri
angulo acut
angulo ABC est
a inscrito numa circunferencia de centro O. As
alturas do tri
angulo s
ao AD, BE e CF . A reta EF intersecta a circunferencia em P
e Q.
(a) Prove que OA e perpendicular a P Q.
7

POT 2012 - Geometria - Nvel 3 - Aula 10 - Prof. Ccero Thiago

(b) Se M e o ponto medio de BC, prove que AP 2 = 2AD.OM .


4. Seja C um ponto sobre o semicrculo de di
ametro AB e seja D o ponto medio do arco
AC. Se E e a projec
ao de D sobre BC e F e a intersecao de AE com o semicrculo,
prove que BF bissecta o segmento DE.
5. Seja P um ponto no interior de um crculo tal que existem tres cordas que passam
por P e tem o mesmo comprimento. Prove que P e o centro do crculo.
6. Sejam 1 e 2 crculos concentricos, com 2 no interior de 1 . Partindo de um ponto
A pertencente a 1 , e desenhada uma tangente AB `a 2 (B 2 ). Seja C o segundo
ponto de intersec
ao de AB com 1 , e D o ponto medio de AB. Um reta passando
por A intersecta 2 em E e F de tal maneira que as mediatrizes de DE e CF se
AM
.
intersectam em um ponto M sobre AC. Determine a raz
ao
MC
Bibliografia
1. Problemas de las olimpiadas matematicas del Cono Sur (I a a IV a )
Fauring - Wagner - Wykowski - Gutierrez - Pedraza - Moreira
2. Olimpadas Cearenses de Matematica - Ensino Fundamental - 1981 - 2005
Emanuel Carneiro, Francisco Ant
onio M. de Paiva e Onofre Campos
3. Potencia de um ponto em relac
ao a uma circunferencia
Eduardo Wagner
Revista do professor de matem
atica - N
umero 45
4. Mathematical Olympiad Challenges
Titu Andreescu e Razvan Gelca

Polos Olmpicos de Treinamento


Aula

Curso de Geometria - Nvel 3

11

Prof. Ccero Thiago

Pot
encia de ponto e eixo radical
Chamaremos de Eixo radical o lugar geometrico dos pontos que possuem a mesma
potencia com relac
ao a duas circunferencias dadas.
Teorema 1. O conjunto dos pontos que possuem a mesma potencia com relacao a duas
circunferencias dadas e uma reta perpendicular `a reta que contem os centros.
Demonstrac
ao.

P
b

O1

O2

Sejam 1 e 2 circunferencias com centros O1 e O2 e raios R1 e R2 , respectivamente. Alem


disso, seja P um ponto que possui a mesma potencia com relacao as duas circunferencias.
Assim,
PotP1 = PotP2

POT 2012 - Geometria - Nvel 3 - Aula 11 - Prof. Ccero Thiago


P O12 R12 = P O22 R22
P O12 P O22 = R12 R22 .
Seja M o ponto medio de O1 O2 , Q a projecao de P sobre O1 O2 e P M Q = . Aplicando
a lei dos cossenos nos tri
angulos P O1 M e P O2 M temos
P O12 = O1 M 2 + P M 2 2 O1 M P M cos(180 ) =
P O12 = O1 M 2 + P M 2 + 2 O1 M P M cos
P O22 = O2 M 2 + P M 2 2 O2 M P M cos .
Entao,
P O12 P O22 = 2 O1 O2 P M cos .
MQ
M Q = P M cos , com isso
Por outro lado, cos =
PM
R2 R22
= Fixo.
MQ = 1
2 O1 O2
Portanto, o lugar geometrico dos pontos P e a reta perpendicular a O1 O2 que passa por Q.
Por outro lado, seja P1 um ponto de P Q. Vamos provar que P1 possui a mesma potencia
com relacao `
as duas circunferencias. Assim, pelo toerema de Pit
agoras
P1 O12 = O1 Q2 + P1 Q2 ,
P1 O22 = O2 Q2 + P1 Q2 .
Entao,
P1 O12 P1 O22 = O1 Q2 O2 Q2 .

P
b

P1
b

O1

O2

POT 2012 - Geometria - Nvel 3 - Aula 11 - Prof. Ccero Thiago

Alem disso,
P O12 = O1 Q2 + P Q2 ,
P O22 = O2 Q2 + P Q2 .
Entao,
P O12 P O22 = R12 R22 = O1 Q2 O2 Q2 = P1 O12 P1 O22
P1 O12 R12 = P1 O22 R22
PotP11 = PotP21 .
Teorema 2. (Euler) Seja O o circuncentro, I o incentro, R o raio da circunferencia
circunscrita e r o raio da circunferencia inscrita de um tri
angulo ABC, entao
OI 2 = R2 2Rr.
Demonstrac
ao.
Lema: Seja D a intersecc
ao de AI com a circunferencia circunscrita ao tri
angulo ABC.
Entao,
DI = DB = DC.
A
b

POT 2012 - Geometria - Nvel 3 - Aula 11 - Prof. Ccero Thiago

Demonstrac
ao. Seja D a intersecc
ao de AI com a circunferencia circunscrita ao tri
angulo
facil ver que DBC = DCB = , ou
ABC e seja BI a bissetriz do
angulo B = 2. E
seja, DB = DC. Pela propriedade do
angulo externo, BID = + = IBD, ou seja,
DI = DB.
Vamos, agora, demonstrar que OI 2 = R2 2Rr. Seja IE = r, DH = 2R e a circunferencia
facil ver que IEA DBH entao
circunscrita ao tri
angulo ABC. E
DB
2R
=
AI DB = 2Rr.
AI
r
Mas, pelo lema, DB = DI. Ent
ao,
AI DI = 2Rr PotI = 2Rr R2 OI 2 = 2Rr OI 2 = R2 2Rr.

O
b

Teorema 3. (Centro radical) Considere tres crculos 1 , 2 e 3 tais que seus centros
O1 , O2 e O3 , respectivamente, n
ao est
ao alinhados. Sejam r, s e t os eixos radicais de 1
4

POT 2012 - Geometria - Nvel 3 - Aula 11 - Prof. Ccero Thiago

e 2 , 1 e 3 e 2 e 3 , respectivamente. Entao, r, s e t s
ao concorrentes em um ponto
chamado centro radical.
Demonstrac
ao.
Seja P um ponto sobre r s, ou seja, P possui a mesma potencia com relacao 1 , 2 e 3 .
Portanto, P est
a sobre a reta t.

O1
b

O2
b

P
b

O3

Problema 1. (India TST) Seja ABC um tri


angulo com AB = AC e seja a sua circunferencia circunscrita. Suponha que sua circunferencia inscrita se desloca sobre BC
em direcao ao vertice B. Prove que quando tangenciar internamente, ela tambem ir
a
tangenciar a altura relativa ao vertice A.
Soluc
ao.

POT 2012 - Geometria - Nvel 3 - Aula 11 - Prof. Ccero Thiago

A
b

b
b

Seja a posic
ao de quando tangencia , e seja K seu centro. Seja O o centro de e I
o centro de . Como AB = AC, ent
ao O e I est
ao sobre a altura AD. Se T e o ponto de
intersecao de e , ent
ao T , K e O s
ao colineares. Dessa forma, OK = OT KT = R r,
onde R e r s
ao, respectivamente os raios das circunferencias circunscrita e inscrita de ABC.
facil ver que K e I est
E
ao sobre a paralela a BC que passa por I. Entao KI e perpendicular
a AD em I. Aplicando o teorema de Pit
agoras no tri
angulo KOI temos
OK 2 = OI 2 + IK 2 .
Mas OI 2 = R2 2Rr, ent
ao
IK 2 = OK 2 OI 2 = (R r)2 (R2 2Rr) = r 2 .
Portanto, IK = r mostrando que tangencia AD em I.
Problema 2. (USAMO) Sejam 1 e 2 crculos que se intersectam em X e Y . Seja l1 uma
reta que passa pelo centro de 1 intersectando 2 nos pontos P e Q e seja l2 uma reta que
passa pelo centro de 2 intersectando 1 nos pontos R e S. Prove que se P , Q, R e S est
ao
6

POT 2012 - Geometria - Nvel 3 - Aula 11 - Prof. Ccero Thiago

sobre uma circunferencia ent


ao o centro desse crculo est
a sobre XY .
Soluc
ao.

Y
b

O1

O2

P
b

H
b

b
b

X
Q
b

Seja a circunferencia circuncrita de QRP S e seja O seu centro. A reta XY e eixo radical
suficiente mostrar que O tem igual potencia com relacao as
das circunferencias 1 e 2 . E
circunferencias 1 e 2 , ou seja,
OO12 O1 S 2 = OO22 O2 Q2 ou OO12 + O2 Q2 = OO22 + O1 S 2 .
Sejam M e N a intersec
oes de O2 O e l1 e O1 O e l2 . Como as circunferencias e 2 se
intersectam em P e Q, ent
ao P Q OO2 . Entao,
OO12 OQ2 = (OM 2 + M O12 ) (OM 2 + M Q2 )
= (O2 M 2 + M O12 ) (O2 M 2 + M Q2 ) = O2 O12 O2 Q2
O2 O12 + OQ2 = OO12 + O2 Q2 .
Temos que, O2 O12 + OS 2 = OO22 + O1 S 2 . Como OS = OQ, entao OO12 + O2 Q2 =
OO22 + O1 S 2 .

POT 2012 - Geometria - Nvel 3 - Aula 11 - Prof. Ccero Thiago

Problema 3. Seja ABCD um quadril


atero inscritvel e E a intersecao das diagonais AC e
BD. Se F e um ponto qualquer e as circunferencias 1 e 2 circunscritas a F AC e a F BD
se intersectam novamente em G, mostre que E, F , G s
ao colineares.
Soluc
ao.

A
b

b
b

C
b

E
O eixo radical de 1 e 2 e a reta F G. Entao E F G PotE
1 = Pot2 . Mas ABCD
E
inscritvel implica que PotE
ao
1 = AE EC = BE ED = Pot2 , e portanto E, F e G s
colineares.

Exerccios propostos

1. (IMO) Seja H o ortocentro de um tri


angulo acut
angulo ABC. A circunferencia de
centro no ponto medio de BC e que passa por H corta BC nos pontos A1 e A2 .
Analogamente, definem - se os pontos B1 e B2 sobre CA e os pontos C1 e C2 sobre
AB. Mostre que os seis pontos A1 , A2 , B1 , B2 , C1 e C2 est
ao sobre uma mesma
circunferencia.
2. As bissetrizes internas dos
angulos CAB, ABC e BCA de um tri
angulo ABC concorrem em I e cortam o crculo circunscrito de ABC em L, M e N , respectivamente.
A circunferencia de di
ametro IL corta o lado BC em D e E, a circunferencia de
di
ametro IM corta o lado CA em F e G, circunferencia de di
ametro IN corta o lado
AB em H e J. Mostre que D, E, F , G, H e J est
ao sobre uma mesma circunferencia.

POT 2012 - Geometria - Nvel 3 - Aula 11 - Prof. Ccero Thiago

3. (IMO) Um crculo de centro O passa pelos vertices A e C de um tri


angulo ABC e
intersecta os segmentos AB e BC novamente em pontos distintos K e N , respectivamente. Os crculos circunscritos aos tri
angulos ABC e KBN se intersectam em
exatamente 2 pontos distintos B e M . Prove que OM B = 90 .
4. (China) Seja ABCD um quadril
atero inscritvel inscrito em . Seja P a intersecao
de AB e DC e seja Q a intersec
ao AD e BC. Sejam QE e QF tangentes a em E
e F , respectivamente. Prove que P , E e F s
ao colineares.
5. (Balcanica) Uma reta passando pelo incentro I do tri
angulo ABC intersecta a circunferencia circunscrita 1 (O, R) de ABC nos pontos F e G e a circunferencia inscrita
2 (I, r) nos pontos D e E, com D entre I e F . Prove que DF EG r 2 . Quando
ocorre a igualdade?
Bibliografia
1. Lecture Notes on Mathematical Olympiad Courses - For senior section - Vol. 1
Xu Jiagu
2. Algunas maneras de usar la potencia
Jose Antonio G
omez Ortega
Olimpiada Mexicana de Matematicas
3. Mathematical Olympiad Challenges
Titu Andreescu e Razvan Gelca
4. T
opicos de Matematica Elementar - Vol.2
Antonio Caminha Muniz Neto
5. Potencia de um ponto em relacao a uma circunferencia
Eduardo Wagner
Revista do professor de matem
atica - N
umero 45

Polos Olmpicos de Treinamento


Aula

Curso de Geometria - Nvel 3

12

Prof. Ccero Thiago

Circunfer
encias ex - inscritas

Teorema 1. Seja XOY um


angulo dado e P um ponto em seu interior. Entao, a dist
ancia
de P a XO e igual a dist
ancia de P a Y O se, e somente se, o ponto P pertence a bissetriz.
Demonstrac
ao.
b

M
b

b
b

Suponhamos inicialmente que o ponto P pertence `a bissetriz. Entao, XOP = Y OP . Sejam M e N os pes das perpendiculares baixadas desde P sobre OX e OY , respectivamente.
Podemos concluir, que M OP N OP , pelo caso L.A.A.. Portanto, P M = P N .
Reciprocamente, suponhamos agora que P M = P N . Pelo caso especial de congruencia de
tri
angulos, cateto - hipotenusa, os tri
angulos M OP e N OP s
ao congruentes. Portanto,
M OP = N OP e, assim, P pertence `a bissetriz.
Teorema 2. As bissetrizes externas de quaisquer dois angulos de um tri
angulo s
ao concorrentes com a bissetriz interna do terceiro angulo.
Demonstrac
ao.

POT 2012 - Geometria - Nvel 3 - Aula 12 - Prof. Ccero Thiago

E
b

No tri
angulo ABC tracamos as bissetrizes externas dos angulos A e B os quais se
intersectam em P . Do teorema 1, como P pertence `a bissetriz externa do angulo A, entao
P E = P F . Alem disso, P pertence `
a bissetriz externa do angulo B, entao P F = P D.
Como P D = P E, pelo teorema 1, conclumos que P pertence `a bissetriz do angulo C.
Dessa forma, se P equidista dos tres lados do tri
angulo ABC e e um ponto no exterior do
tri
anglo entao P e o centro de uma das tres circunferencias ex - inscritas do tr
angulo ABC.
A circunferencia com centro Ia e raio ra e uma das tres circunferencias ex - inscritas que
representaremos apenas por (Ia , ra ). Analogamente s
ao definidas as circunferencias (Ib , rb )
e (Ic , rc ). Os pontos Ia , Ib e Ic s
ao os ex - incentros. Cada circunferencia ex - inscrita
toca um dos lados do tri
agulo internamente e os outros dois externamente, ou seja, toca no
prolongamento. Na figura a seguir, observe que pela propriedade de segmentos tangentes a
uma circunferencia, vulgarmente conhecido com Teorema do bico, temos que BL = BG,
alem disso
BL + BG = (BC + CL) + (AG + AB)
= BC + CE + AE + AB = a + b + c = 2p.
Portanto, as tangentes tracadas por B a` circunferencia (Ib , rb ) tem medida p. Dessa forma
e facil ver que
AJ = AK = BG = BL = CH = CM = p.
Alem disso, CL = BL BC = p a. Entao,
BM = BF = CL = CE = p a,
CK = CD = AH = AF = p b,
AG = AE = BJ = BD = p c.

POT 2012 - Geometria - Nvel 3 - Aula 12 - Prof. Ccero Thiago

G
b

Ib

H
b

A
b

Ic
b

I
b

E
b

L
b

b
b

K
b

Ia

Problema 1. Sejam ABC um tri


angulo, M o pe da bissetriz interna do angulo A e N o
pe da bissetriz interna do
angulo B. Suponha que M N seja bissetriz do angulo AM C.
Calcule a medida do
angulo A.
Soluc
ao.

E facil ver que N e um dos ex - incentros do tri


angulo ABC pois e a intersecao da bissetriz externa do
angulo AM B e da bissetriz interna do angulo B. Logo, AN e bissetriz
externa do angulo A. Portanto, A = 120 .

POT 2012 - Geometria - Nvel 3 - Aula 12 - Prof. Ccero Thiago

b
b

Problema 2. (OBM) Um tri


angulo ABC, de lados AB = c, AC = b e BC = a, tem
permetro 2p. Uma circunferencia tangencia o lado BC e os prolongamentos dos lados AB
e AC nos pontos P , Q e R, respectivamente. O comprimento AR e igual a:
(a) p a (b) p b (c) p c (d) p (e) 2p
Soluc
ao.
A
b

b
b

R
b

Q
b

IA

POT 2012 - Geometria - Nvel 3 - Aula 12 - Prof. Ccero Thiago

Pelo teorema 2 e facil ver que AR = AQ = p. Portanto, a resposta e o item (b).


Problema 3. No quadril
atero ABCD determine a medida do angulo AED.
C
b

D
b

60
60

E
b

70
40
b

Soluc
ao.
b

60
D
b

60
60

E
b

70

40
40

70

40

Na figura, F DC = 60 e GBC = 70 . Entao, BC e DC s


ao bissetrizes externas dos
angulos ABD e ADB. Dessa forma, AC e bissetriz interna do angulo BAD. Portanto,
DAE = BAE = 40 . Finalmente, AED = 80 .
Exerccios propostos
1. Prove que os tres segmentos determinados por um vertice e pelo ponto de tangencia
da circunferencia ex - inscrita com o lado oposto a esse vertice s
ao concorrentes em
um ponto chamado ponto de Nagel.
2. (OBM) A medida do
angulo B de um tri
angulo ABC e 120 . Sejam M um ponto sobre o lado AC e K um ponto sobre o prolongamento do lado AB, tais que BM e a bis5

POT 2012 - Geometria - Nvel 3 - Aula 12 - Prof. Ccero Thiago

setriz interna do
angulo ABC e CK e a bissetriz externa correspondente ao angulo
ACB. O segmento M K intersecta BC no ponto P . Prove que AP M = 30 .
3. (Leningrado) Sejam AF , BG e CH as bissetrizes de um tri
angulo ABC que tem
o
o
angulo A medindo 120 . Prove que o angulo GF H mede 90 .
4. (Belarus) Seja O o centro do crculo ex - inscrito do tri
angulo ABC oposto ao vertice
A. Seja M o ponto medio de AC e seja P a intersecao das retas M O e BC. Prove
que se BAC = 2ACB, ent
ao AB = BP .
5. (IMO) Dado um tri
angulo ABC, o ponto J e o centro da circunferencia ex-inscrita
oposta ao vertice A. Esta circunferencia ex-inscrita e tangente ao lado BC em M ,
e `as retas AB e AC em K e L, respectivamente. As retas LM e BJ intersectam-se
em F , e as retas KM e CJ intersectam-se em G. Seja S o ponto de intersecao das
retas AF e BC, e seja T o ponto de intersecao das retas AG e BC. Prove que M e
o ponto medio de ST .
(A circunferencia ex-inscrita de ABC oposta ao vertice A e a circunferencia tangente
ao segmento BC, ao prolongamento do segmento AB no sentido de A para B e ao
prolongamento do segmento AC no sentido de A para C.)
Bibliografia
1. T
opicos de Matematica Elementar - Vol. 2
Antonio Caminha Muniz Neto
2. Geometria
Radmila Bulajich Manfrino e Jose Antonio G
omez Ortega

Polos Olmpicos de Treinamento


Aula

Curso de Geometria - Nvel 3

13

Prof. Ccero Thiago

Revis
ao I

Problema 1. Em um tri
angulo ABC, BAC = 100 e AB = AC. Seja BD a bissetriz de
ABC, com D sobre o lado AC. Prove que AD + BD = BC.
Soluc
ao.
facil ver que ABD = DBC = 20 . Seja E um ponto sobre BC tal que BD = BE.
E
Basta provar que EC = AD. Veja que BDE = BED = 80 . Como BED = 80 e
BCD = 40 , ent
ao EDC = 40 , ou seja, ED = EC. Por outro lado, ABED e um
quadril
atero inscritvel pois BAD + BED = 180 , assim EAD = EBD = 20 e
AED = ABD = 20 . Portanto, AD = ED = EC e, dessa forma, BC = AD + BD.
A
b

Problema 2. (Inglaterra) No tri


angulo acut
angulo ABC, CF e altura, com F em AB e
BM e mediana, com M em CA. Se BM = CF e M BC = F CA, prove que o tri
angulo
ABC e equilatero.
Soluc
ao.

POT 2012 - Geometria - Nvel 3 - Aula 13 - Prof. Ccero Thiago

A
b

F
b

Temos que F M = AM = M C e, com isso, M F C = F CM , ou seja, o quadril


atero

F BCM e inscritvel. Dessa forma, F CM = F BM e BM C = BF C = 90 . E


facil ver que BM C BM A, pelo caso A.L.A, entao AB = BC. Veja tambem que
BM C BF C, pelo caso cateto - hipotenusa, entao BCM = CBF e, portanto,
AC = AB. Finalmente, AB = AC = BC.
Problema 3. Seja M um ponto no interior de um quadril
atero convexo ABCD tal que
ABM D e um paralelogramo. Prove que se CBM = CDM , entao ACD = BCM .
Soluc
ao.

POT 2012 - Geometria - Nvel 3 - Aula 13 - Prof. Ccero Thiago

A
b

Seja N um ponto tal que BN k M C e N C k BM . Entao N A k CD, N CB =


CBM = CDM = N AB, ou seja, os pontos A, B, N e C s
ao concclicos. Entao,
ACD = N BC = BCM .
Problema 4. (Seletiva do Brasil para a Cone Sul) Prove que as dist
ancias entre um ponto
sobre uma circunferencia e os quatro vertices de um quadrado nesta inscrita n
ao podem
ser todas n
umeros racionais.
Soluc
ao.

POT 2012 - Geometria - Nvel 3 - Aula 13 - Prof. Ccero Thiago

Como ABCD e um quadrado ent


ao AB = BC = CD = DA = a. Pelo toerema de
Pit
agoras
angulo ABC temos que AC 2 = AB 2 + BC 2 AC 2 = a2 + a2 = 2 a2
no tri
AC = 2 a. Aplicando o teorema de Ptolomeu no quadril
atero ABCP , temos

AC BP = AP BC + CP AB 2 a BP = AP a + CP a

AP + CP
.
BP
Se todas as medidas fossem n
umeros racionais estaramos afirmando, de maneira falsa, que

BP
2 Q. Se P coincidir com um dos vertices, ou seja, P D, entao
= 2. Assim, as
CP
medidas n
ao podem ser todas racionais.
2=

Problema 5. (Ira) Seja ABC um tri


angulo com BC > CA > AB. Seja D um ponto
sobre o lado BC e seja E o ponto no prolongamento de BA, com A entre E e B, tal que
BD = BE = CA. Seja P o ponto sobre AC tal que E, B, D e P s
ao concclicos e seja Q
o segundo ponto de intersec
ao de BP com o crculo circunscrito ao tri
angulo ABC. Prove
que AQ + CQ = BP .
Soluc
ao.

POT 2012 - Geometria - Nvel 3 - Aula 13 - Prof. Ccero Thiago

A
b

Q
b

Veja que AQC EP D, pois CAQ = CBQ = DEP e AQC = 180 ABD =
EP D. Por outro lado, pelo teorema de Ptolomeu, temos
BP DE = BE DP + BD EP.
Entao,
EP
CQ
AQ
DP
+ BD
= CA
+ CA
= AQ + CQ.
DE
DE
CA
CA
Problemas propostos
BP = BE

1. (Cone Sul) Seja ABCD um quadril


atero convexo tal que suas diagonais AC e BD
s
ao perpendiculares. Seja P a interseccao de AC e BD e seja M o ponto medio de
AB. Mostre que o quadril
atero ABCD e inscritvel se, e somente se, as retas P M e
CD s
ao perpendiculares.
2. Prove que as bissetrizes internas dos quatro angulos de um quadril
atero convexo determinam um quadril
atero inscritvel.
3. (OBM) As diagonais de um quadril
atero inscritvel ABCD se intersectam em O. Os
crculos circunscritos aos tri
angulos AOB e COD intersectam as retas BC e AD,
5

POT 2012 - Geometria - Nvel 3 - Aula 13 - Prof. Ccero Thiago

pela segunda vez, nos pontos M , N , O e Q. Prove que o quadril


atero M N P Q est
a
inscrito em um crculo de centro O.
4. Um quadril
atero convexo est
a inscrito em um crculo de centro O. As diagonais AC
e BD intersectam - se em P . Os crculos circunscritos aos tri
angulos ABP e CDP
intersectam - se novamente em Q. Se O, P e Q s
ao tres pontos distintos, prove que
OQ e perpendicular a P Q.
5. (Ibero) Num tri
angulo escaleno ABC traca-se a bissetriz interna BD, com D sobre
AC. Sejam E e F , respectivamente, os pes das perpendiculares tracadas desde A e
C ate `a reta BD, e seja M o ponto sobre o lado BC tal que DM e perpendicular a
BC. Prove que EM D = DM F.
6. Seja M o ponto de intersec
ao das diagonais de um quadril
atero inscritvel ABCD,
em que AM B e agudo. O tri
angulo is
osceles BCK e construdo exteriormente ao
quadril
atero, com base a base sendo BC, tal que KBC + AM B = 90 . Prove que
KM e perpendicular a AD.
7. Seja ABCD um quadril
atero convexo tal que
AC BD = AB CD + AD BC.
Prove que ABCD e inscritvel.
8. Seja ABCD um quadrado. Determine o lugar geometrico dos pontos P , no mesmo
plano do quadrado ABCD, tais que
1
m
ax {P A, P C} = (P B + P D).
2
9. Uma circunferencia passa pelo vertice A de um paralelogramo ABCD intersectando
os lados AB e AD nos pontos P e R, respectivamente. Alem disso intersecta a diagonal AC no ponto Q. Prove que AQ AC = AP AB + AR AD.
10. Um ponto P e escolhido o interior do paralelogramo ABCD de tal forma que AP B+
CP D = 180 . Prove que AB AD = BP DP + AP CP .

Polos Olmpicos de Treinamento


Aula

Curso de Geometria - Nvel 3

14

Prof. Ccero Thiago

Caminhos mnimos e desigualdades envolvendo elementos


geom
etricos
1. Proposi
c
ao
Se dois lados de um tri
angulo n
ao s
ao congruentes, entao os angulos opostos a estes lados
n
ao s
ao congruentes, e o maior
angulo e oposto ao maior lado.
Demonstra
c
ao
A

Suponhamos BC > AC. Seja D o ponto sobre o lado BC tal que AC = CD. Entao o
tri
angulo ADC e is
osceles de base AD e, com isso, CAD = CDA. Pelo Teorema do
angulo externo, CDA > ABC, mas CAD = CDA, entao CAD > ABC. Por
outro lado, BAC = BAD + CAD, entao e facil concluir, que BAC > ABC.
Exerccio
1. Prove que se dois
angulos de um tri
angulo n
ao s
ao congruentes, entao os lados opostos
a eles n
ao s
ao congruentes, e o maior lado e oposto ao maior angulo.
2. Desigualdade Triangular
A soma dos comprimentos de dois lados quaisquer de um tri
angulo e maior que o comprimento do terceiro lado.

POT 2012 - Geometria - Nvel 3 - Aula 14 - Prof. Ccero Thiago


D
A

Seja D o ponto sobre o prolongamento do lado AC tal que AD = AB. Entao o tri
angulo
BAD e is
osceles de base BD e, com isso, BDA = DBA. Alem disso, CD = DA + AC.
facil ver que DBC > DBA = BDA. Entao, pelo exerccio anterior, DC > BC,
E
mas DC = DA + AC = AB + AC. Portanto, BC < AB + AC.
Consequ
encia da desigualdade triangular
Sejam A, P1 , P2 , . . . , Pn e B pontos do plano, entao
AP1 + P1 P2 + . . . + Pn B AB.
Alem disso, a igualdade ocorre se, e somente se, os pontos A, P1 , P2 , . . . , Pn e B s
ao colineares e aparecem nessa ordem.
Exerccios Resolvidos
1. Dados n pontos A1 , A2 , . . . , An e um crculo unit
ario, prove que e possvel encontrar um
ponto M sobre o crculo tal que M A1 + M A2 + . . . + M An n.
Solu
c
ao:
Sejam M1 e M2 pontos diametralmente opostos no crculo. Entao M1 Ak +M2 Ak M1 M2 =
2. Adicionando essas desigualdades para k = 1, 2, . . . , n temos
(M1 A1 + . . . + M1 An ) + (M2 A1 + . . . + M2 An ) 2n.
Portanto, M1 A1 + . . . + M1 An n assim, basta fazer, M = M1 ou M2 A1 + . . . + M2 An n,
fazendo M = M2 .
2. Prove que a media aritmetica dos comprimentos dos lados de um polgono convexo
arbitrario e menor que a media aritmetica dos comprimentos de todas as diagonais.
Solu
c
ao:
Sejam Ap Ap+1 e Aq Aq+1 dois lados n
ao adjacentes de um n-agono convexo A1 , A2 , . . . , An
(i.e., |p q| 2). Ent
ao
Ap Ap+1 + Aq Aq+1 < Ap Aq + Ap+1 Aq+1 .
Vamos escrever todas as desigualdades e, em seguida, som
a - las. Para cada lado existem precisamente n 3 lados n
ao adjacentes a ele e, portanto, cada lado aparece em
2

POT 2012 - Geometria - Nvel 3 - Aula 14 - Prof. Ccero Thiago

n 3 desigualdades, i.e, no lado esquerdo da desigualdade obteremos a soma (n 3)p,


onde p representa a soma dos comprimentos de todos os lados do n-agono. Cada diagonal aparece em duas desigualdades portanto, o lado direito da desigualdade ser
a 2d,
onde d representa a soma dos comprimentos de todas as diagonais do n-agono. Assim,
p
d
(n 3)p < 2d < n(n3) .
n
2

3. A id
eia do menor caminho
Dados dois pontos A e B de um mesmo lado de uma reta r, determinar o ponto P sobre
r de forma que P A + P B seja mnimo.
Solu
c
ao:

A
B

r
P

C
Para acharmos o ponto P que minimiza P A + P B basta tomar o simetrico de A, que
chamaremos de C, com relac
ao `
a reta r e em seguida ligarmos o ponto C ao ponto B. A
nossa construc
ao garante que P A = P C, entao, a menor dist
ancia entre C e B ser
a uma
reta que liga os dois. A intersec
ao desta reta com a reta r ser
a o nosso ponto P .
4. Defini
c
ao
Dados dois pontos distintos F1 e F2 , pertencentes a um plano , seja 2c a dist
ancia entre
eles.
Elipse e o conjunto dos pontos de cuja soma das dist
ancias a F1 e F2 e constante 2a
(sendo 2a > 2c), ou seja,
Elipse = {P |P F1 + P F2 = 2a} .

POT 2012 - Geometria - Nvel 3 - Aula 14 - Prof. Ccero Thiago


P

A1

F1

A2

F2

Dado um ponto P1 no interior de uma elipse, entao P1 F1 + P1 F2 < 2a.


P
P1

P2

F2

F1

Com efeito, 2a = F1 P2 + P2 F2 = F1 P1 + P1 P2 + P2 F2 > P1 F1 + P1 F2 . Prove agora que se


P3 for um ponto externo `
a elipse ent
ao P3 F1 + P3 F2 > 2a.
5. Teorema
Seja l uma reta tangente a uma elipse no ponto P . Entao l e a bissetriz externa do angulo
F1 P F2 (Figura abaixo).
P

F1

F2

Prova:
Seja X um ponto da reta l diferente de P . Como X est
a no exterior de uma elipse, entao
XF1 + XF2 > P F1 + P F2 , isto e, de todos os pontos de l, P e o ponto que minimiza a
soma das dist
ancias a F1 e F2 . Isto mostra que os angulos que P F1 e P F2 faz com l s
ao
iguais.

Exerccios

POT 2012 - Geometria - Nvel 3 - Aula 14 - Prof. Ccero Thiago

1. (R
ussia) Sejam AB e CD segmentos de comprimento 1. Se eles se intersectam em O
e, AOC = 60o , prove que AC + BD 1.
2. (China) Seja ABCD um quadril
atero convexo tal que BAD = BCD = 90o .
Sabendo que a bissetriz do
angulo BAD e paralela a BC, perpendicular a CD e
intersecta BD em E, prove que AE < 21 CD.
3. (Col
ombia) Seja ABCD um trapezio, com AB paralelo a CD, e AB CD. Prove
que
AD + BC > AB CD BC AD
e determine todos os possveis casos de igualdade.
4. (Eslovaquia) Seja ABCD um tetraedro com
BAC + CAD + DAB = ABC + CBD + DBA = 180o .
Prove que CD AB.
5. (USAMO) Um tetraedro ABCD e is
osceles, isto e, AB = CD, AC = BD, AD = BC.
Prove que as faces do tetraedro s
ao tri
angulos acut
angulos.
6. (Teorema de Steiner) Se duas bissetrizes de um tri
angulo s
ao congruentes, entao o
tri
angulo e is
osceles.
7. (Problema de Fagnano) Determine o tri
angulo de permetro mnimo inscrito em um
tri
angulo acut
angulo.
8. (Ponto de Fermat) Seja ABC um tri
angulo acut
angulo. Encontrar o ponto interior
que minimiza a soma AP + BP + CP .
9. Em um quadril
atero convexo qual e o ponto que minimiza a soma das dist
ancias aos
vertices? Qual e a soluc
ao se o quadril
atero n
ao e convexo?
10. (Maio) Considere uma pir
amide cuja base e um tri
angulo equilatero e cujas outras
faces s
ao tri
angulos is
osceles e ret
angulos, no vertice A. Uma formiga parte do vertice
B e chega em um ponto P da aresta CD, em seguida, partindo de P chega a um
ponto Q a aresta AC e retorna ao ponto B. Sabendo que o caminho percorrido foi
mnimo, determine a medida do angulo P QA.
11. (Baltic Way) Seja ABC um tri
angulo com A = 120o . Sejam K e L pontos sobre
os lados AB e AC, respectivamente. Sejam BKP eCLQ tri
angulos equilateros
3
(AB + AC).
construdos no exterior do tri
angulo. Prove que P Q
2
12. (Baltic Way) Seja ABCD um quadril
atero convexo e seja N o ponto medio de BC.
1
o
Se AN D = 135 , prove que AB + CD + BC AD.
2

POT 2012 - Geometria - Nvel 3 - Aula 14 - Prof. Ccero Thiago

13. Seja ABCD um quadril


atero convexo tal que BAD = 30o e AC = BC + CD + BD.
Prove que BCD = 120o .
14. (Seletiva Cone Sul do Peru) AM BCN D um hexagono tal que AM B = CN D =
90o e o quadril
atero ABCD e circunscritvel. Prove que BC + AD M N .
15. Seja ABC um tri
angulo acut
angulo e R o raio de sua circunferencia circunscrita.
Prove que AB + BC + CA > 4R.
16. (Shortlist IMO) Seja ABC um tri
angulo e M um ponto em seu interior. Prove que
min {M A, M B, M C} + M A + M B + M C < AB + AC + BC.
17. Seja ABC um tri
angulo is
osceles de base AC tal que B = 20o . Prove que:
a) AB < 3AC.
b) AB > 2AC.
18. Entre todos os quadril
ateros ABCD com AB = 3, CD = 2 e AM B = 120o , onde
M e o ponto medio de CD, ache aquele que possui o permetro mnimo.
19. Considere um tri
angulo com base fixa BC tal que o vertice V est
a sobre uma reta
r paralela a BC. Seja Q a intersecao da mediatriz de BC e a reta r. Prove que
quanto mais pr
oximo de Q estiver o vertice V entao maior ser
a a medida do raio da
circunferencia inscrita no tri
angulo V BC.
20. (Ibero) Demonstre que entre todos os tri
angulos cujos vertices distam 3, 5 e 7 de um
ponto P dado, o que tem maior permetro admite P como incentro.
21. (IMO) Seja ABCDEF um hexagono convexo com AB = BC = CD e DE = EF =

F A, tal que BCD = EF A = . Sejam G e H os pontos no interior do hexagono


3
2
tais que AGB = DHE =
. Prove que
3
AG + GB + GH + DH + HE CF.
Bibliografia
1. Advanced Euclidean Geometry; Alfred S. Posamentier.
2. Geometry of Conics; A. V. Akopyan e A. A. Zaslavsky.
3. Geometric inequalities; Nicholas D. Kazarinoff.
4. Olimpiada Nacional Escolar de Matematica VII; Jorge Tipe Villanueva, Claudio Espiniza Choqquepura e John Cuya Barrios.
5. Geometric Problems on Maxima and Minima; Titu Andreescu, Oleg Mushkarov e
Luchezar Stoyanov.
6

POT 2012 - Geometria - Nvel 3 - Aula 14 - Prof. Ccero Thiago

6. Inequalities; Radmila Bulajich Manfrino, Jose Antonio G


omez Ortega e Rogelio Valdez Delgado.
7. When less is more: Visualizing basic inequalities; Claudi Alsina e Roger B. Nelsen.
8. Problems in plane and solid geometry, v.1, Plane Geometry; Viktor Prasolov.

Polos Olmpicos de Treinamento


Aula

Curso de Geometria - Nvel 3

15

Prof. Ccero Thiago

Qu
adruplas harm
onicas e circunfer
encia de Apol
onio
Teorema 1. (Bissetriz interna) A bissetriz interna AL do angulo A de um tri
angulo
AB
, ou seja,
ABC divide internamente o lado oposto BC na raz
ao
CA
AB
BL
=
LC
CA
em que L e o ponto de intersecc
ao da bissetriz interna com o lado BC.
Demonstrac
ao.
b

A
b

facil ver que


Seja R a intersecc
ao da paralela `
a bissetriz AL tracada pelo ponto C. E
BAL = CAL = ACR = ARC, com isso, AR = AC. Pelo teorema de Tales temos
que
AB
BL
=
.
AR
LC
Como AR = AC, ent
ao
BL
AB
=
.
AC
LC

POT 2012 - Geometria - Nvel 3 - Aula 15 - Prof. Ccero Thiago

Teorema 2. (Bissetriz externa) A bissetriz externa AL do angulo A de um tri


angulo
AB
, ou seja,
ABC divide externamente o lado oposto BC na raz
ao
CA
AB
BL
=
LC
CA
em que L e o ponto de intersecc
ao da bissetriz externa com o lado BC.
Demonstrac
ao.
E

R
b

facil ver que


Seja R a intersecc
ao da paralela `
a bissetriz AL tracada pelo ponto C. E
EAL = CAL = ACR = ARC, com isso, AR = AC. Pelo teorema de Tales temos
que
AB
BL
=
.
AR
LC
Como AR = AC, ent
ao
AB
BL
=
.
AC
LC
Defini
c
ao 1. Sejam A, C, B, D quatro pontos alinhados nessa ordem. Dizemos que
A, C, B, D formam uma divis
ao harmonica ou uma quadrupla harmonica se, e somente
se,
AD
AC
=
.
CB
DB
Teorema 3. Seja ABC um tri
angulo e sejam X, Y e Z pontos sobre os lados BC, CA e
AB, respecivamente. Seja W a interseccao da reta Y Z com o prolongamento do lado BC.
Entao B, X, C e W formam uma quadrupla harmonica se, e somente se, AX, BY e CZ
s
ao concorrentes.
Demonstrac
ao.

POT 2012 - Geometria - Nvel 3 - Aula 15 - Prof. Ccero Thiago

A
b

Z
b

b
b

Suponha que AX, BY e CZ estejam alinhados. Pelo teorema de Ceva temos que
BX CY AZ

= 1.
XC Y A ZB
Alem disso, pelo teorema de Menelaus temos que
W C BZ AY

=1
W B ZA Y B
BW
BX
=
e, portanto, B, X, C e W formam uma quadrupla harmonica.
XC
WC
BX
BW
Suponha que B, X, C e W formam uma quadrupla harmonica, ou seja,
=
.
XC
WC
Pelo teorema de Menelaus temos que
entao

W C BZ AY

= 1.
W B ZA Y B
Portanto,
XC BZ AY

= 1, e
BX ZA Y B
AX, BY e CZ s
ao concorrentes.
Teorema 4. (Circunferencia de Apol
onio) Sejam A e B dois pontos fixos e k 6= 1 e uma
AP
= k e uma circunconstante real. O lugar geometrico dos pontos P que satisfazem
PB
ferencia conhecida como circunfer
encia de Apol
onio.
Demonstrac
ao. Primeiro observe que sobre a reta AB existem exatamente dois pontos
do lugar geometrico. Por A e B trace segmentos paralelos M N e BQ com BQ = 1 e
M A = AN = k. Sejam R e S as interseccoes de QN e QM com AB, respectivamente.

POT 2012 - Geometria - Nvel 3 - Aula 15 - Prof. Ccero Thiago

Q
b

R
b

AR
AN
k
=
=
= k. Alem
RB
BQ
1
AM
k
AS
=
=
= k.
disso, tambem s
ao semelhantes os tri
angulos ASM e BSQ entao
SB
BQ
1
Portanto, R e S pertencem ao lugar geometrico. Suponha que P e um ponto do lugar
AP
AR
geometrico, ent
ao
=k=
, portanto P R e a bissetriz interna do angulo AP B e
PB
RB
AS
AP
=k=
ent
ao P S e a bissetriz externa do mesmo angulo, isto garante que P
como
PB
SB
est
a sobre a circunferencia de di
ametro RS (Porque o angulo RP S = 90 ?).

Como os tri
angulos AN R e BQR s
ao semelhantes entao

P
b

Reciprocamente, considere um ponto P sobre a circunferencia de di


ametro RS. Demonstraremos que P e um ponto do lugar geometrico. Pelo ponto B trace paralelas BE e BF
`as retas P R e P S de tal forma que intersectem AP em E e F , respectivamente. Como os
tri
angulos ARP e ABE s
ao semelhantes com lados paralelos, pelo teorema de Tales temos
AR
AP
=
= k. Pelo mesmo motivo os tri
angulos semelhantes ABF e ASP garantem
que
PE
RB
AS
AP
=
= k. Assim, P E = P F , ou seja, P e o ponto medio do segmento EF e
que
PF
SB
AP
AP
=
= k,
como o tri
angulo EBF e ret
angulo temos que P B = P E = P F . Portanto,
PB
PE
4

POT 2012 - Geometria - Nvel 3 - Aula 15 - Prof. Ccero Thiago

com isso podemos afirmar que P pertence ao lugar geometrico.

P
b

Teorema 5. Seja A, C, B e D uma quadrupla harmonica. Seja O um ponto que n


ao
pertence `a reta AB. Se uma paralela pelo ponto B `a OA intersecta OC e OD em pontos
P e Q entao P B = P Q.
Demonstrac
ao.

O
b

Q
b

C
b

Temos que os tri


angulos OAC e P CB s
ao semelhantes entao
AO
AC
=
.
PB
CB
Alem disso, os tri
angulos OAD e BQD s
ao semelhantes entao
AD
AO
=
.
BQ
BD
AC
AD
Como A, C, B e D e uma quadrupla harmonica entao
=
. Portanto, e facil conCB
DB
cluir que, P B = BQ.
Teorema 6. Sejam A, C, B e D quatros pontos colineares e seja O um ponto que n
ao pertence `a reta AB. Se a paralela `
a OA passando por B intersecta OC e OD em pontos P e Q,
5

POT 2012 - Geometria - Nvel 3 - Aula 15 - Prof. Ccero Thiago

respectivamente, tais que P B = BQ entao A, C, B e D formam uma quadrupla harmonica.


Demonstrac
ao.
O
b

Q
b

C
b

B
b

Temos que os tri


angulos OAC e P CB s
ao semelhantes entao
AC
AO
=
.
PB
CB
Alem disso, os tri
angulos OAD e BQD s
ao semelhantes entao
AO
AD
=
.
BQ
BD
AD
AC
=
. Portanto, e facil concluir que, A, C, B e D e uma
Como P B = BQ, ent
ao
CB
DB
quadrupla harmonica.
Teorema 7. Seja A, C, B e D uma quadrupla harmonica e um ponto O que n
ao pertence
`a reta AB. Sejam M , R, N e S as interseccoes de OA, OC, OB e OD, respectivamente,
com uma reta arbitraria. Ent
ao, M , R, N e S formam uma quadrupla harmonica.
Demonstrac
ao. Pelos pontos B e N trace paralelas `a OA que intersectam OC e OD,
facil ver que
respectivamente, nos pontos P , Q e L, T . E
PB
OB
BQ
=
=
.
LN
ON
NT
Como A, C, B e D formam uma quadrupla harmonica entao, pelo teorema 5, que P B =
BQ. Com isso, LN = N T e, portanto, M , R, N e S formam uma quadrupla harmonica
pelo teorema 6.

POT 2012 - Geometria - Nvel 3 - Aula 15 - Prof. Ccero Thiago

T
b

b
b

R
b

L
b

Problema 1. Seja ABCD um quadril


atero inscrito em uma circunferencia tal que AB = AD
e AC e um di
ametro. Seja P um ponto sobre o arco BC que n
ao contem A e D. Se P A
e P D intersectam o lado BC nos pontos E e F , respectivamente, e BE = 3 e EF = 2,
determine F C.
Soluc
ao. Como AB = AD ent
ao BP A = AP D = . Como AP C = 90 entao

F P C = 90 = e QP C = 90 = . Portanto, P E e P C s
ao bissetrizes
interna e externa do
angulo BP F , respectivamente e, com isso, B, E, F e C formam um
quadrupla harmonica ent
ao
BC
3
FC + 5
BE
=
=
F C = 10.
EF
FC
2
FC

POT 2012 - Geometria - Nvel 3 - Aula 15 - Prof. Ccero Thiago

P
b

F
b

E
b

O
b

Problema 2. (Turquia) Seja ABC um tri


angulo com AB 6= AC. As bissetrizes interna e
externa relativas ao vertice A intersectam a reta BC em D e E, respectivamente. Se os pes
das perpendiculares baixadas de um ponto F do crculo de di
amentro DE sobre as retas
BC, CA e AB s
ao K, L e M , respectivamente, prove que KL = KM .
Soluc
ao. O crculo de di
ametro DE e um crculo de Apolonio do tri
angulo ABC relativo
ao vertice A ent
ao
AB
FB
=
. (1)
FC
AC
Pela lei dos senos temos que
sin C
AB
=
. (2)
AC
sin B
De (1) e (2) temos que
FB
sin C
=

FC
sin B
F B sin B = F C sin C. (3)
Como o crculo de di
ametro BF passa pelos pontos K e M , entao
KM = F B sin B.

POT 2012 - Geometria - Nvel 3 - Aula 15 - Prof. Ccero Thiago

De maneira similar, temos que


KL = F C sin C.
Portanto, de (3), temos que KL = KM .

A
b

L
b

b
b

b
b

K E

Exerccios propostos
1. Em um tri
angulo ABC, BD e BE s
ao as bissetrizes interna e externa, respectivamente. Se AD = 5 e DC = 3, determine a medida do segmento CE.
2. Seja ABC um tri
angulo e sejam D o pe da bissetriz relativa ao vertice A, I o incentro e Ia o ex - incentro oposto ao vertice A. Prove que A, I, D e Ia formam uma
quadrupla harmonica.
3. Em um tri
angulo n
ao equilatero, a reta que passa pelo baricentro e pelo incentro e
paralela a um dos lados do tri
angulo. Demonstre que os lados do tri
angulo est
ao em
progress
ao aritmetica.
4. (Shortlist IMO) Seja ABC um tri
angulo, e sejam D, E, F os pontos de tangencia do
crculo inscrito no tri
angulo ABC com os lados BC, CA e AB respectivamente. Seja
X um ponto no interior do tri
angulo ABC tal que o crculo inscrito no tri
angulo
XBC tangencia XB, XC e BC em Z, Y e D, respectivamente. Prove que EF ZY e
inscritvel.

POT 2012 - Geometria - Nvel 3 - Aula 15 - Prof. Ccero Thiago

5. (TST Jr Balkan - Romenia) Seja ABC um tri


angulo ret
agulo com A = 90 e D um
ponto sobre o lado BC. Sejam E o simetrico de A com relacao a BD e F o ponto de
intersecc
ao de CE com a perpendicular a BC por D. Prove que AF, DE e BC s
ao
concorrentes.
Bibliografia
1. Geometra - Teora y pr
actica.
Fernando Alva Gallegos
2. College Geometry - An introduction to the modern geometry of the triangle and the
circle.
Nathan Altshiller - Court
3. Geometra
Radmila Bulajich Manfrino e Jose Antonio G
omez Ortega
4. Harmonic Division and its Applications
Cosmin Pohoata

10

Polos Olmpicos de Treinamento


Aula

Curso de Geometria - Nvel 3

15

Prof. Ccero Thiago

Qu
adruplas harm
onicas e circunfer
encia de Apol
onio
Teorema 1. (Bissetriz interna) A bissetriz interna AL do angulo A de um tri
angulo
AB
, ou seja,
ABC divide internamente o lado oposto BC na raz
ao
CA
AB
BL
=
LC
CA
em que L e o ponto de intersecc
ao da bissetriz interna com o lado BC.
Demonstrac
ao.
b

A
b

facil ver que


Seja R a intersecc
ao da paralela `
a bissetriz AL tracada pelo ponto C. E
BAL = CAL = ACR = ARC, com isso, AR = AC. Pelo teorema de Tales temos
que
AB
BL
=
.
AR
LC
Como AR = AC, ent
ao
BL
AB
=
.
AC
LC

POT 2012 - Geometria - Nvel 3 - Aula 15 - Prof. Ccero Thiago

Teorema 2. (Bissetriz externa) A bissetriz externa AL do angulo A de um tri


angulo
AB
, ou seja,
ABC divide externamente o lado oposto BC na raz
ao
CA
AB
BL
=
LC
CA
em que L e o ponto de intersecc
ao da bissetriz externa com o lado BC.
Demonstrac
ao.
E

R
b

facil ver que


Seja R a intersecc
ao da paralela `
a bissetriz AL tracada pelo ponto C. E
EAL = CAL = ACR = ARC, com isso, AR = AC. Pelo teorema de Tales temos
que
AB
BL
=
.
AR
LC
Como AR = AC, ent
ao
AB
BL
=
.
AC
LC
Defini
c
ao 1. Sejam A, C, B, D quatro pontos alinhados nessa ordem. Dizemos que
A, C, B, D formam uma divis
ao harmonica ou uma quadrupla harmonica se, e somente
se,
AD
AC
=
.
CB
DB
Teorema 3. Seja ABC um tri
angulo e sejam X, Y e Z pontos sobre os lados BC, CA e
AB, respecivamente. Seja W a interseccao da reta Y Z com o prolongamento do lado BC.
Entao B, X, C e W formam uma quadrupla harmonica se, e somente se, AX, BY e CZ
s
ao concorrentes.
Demonstrac
ao.

POT 2012 - Geometria - Nvel 3 - Aula 15 - Prof. Ccero Thiago

A
b

Z
b

b
b

Suponha que AX, BY e CZ estejam alinhados. Pelo teorema de Ceva temos que
BX CY AZ

= 1.
XC Y A ZB
Alem disso, pelo teorema de Menelaus temos que
W C BZ AY

=1
W B ZA Y B
BW
BX
=
e, portanto, B, X, C e W formam uma quadrupla harmonica.
XC
WC
BX
BW
Suponha que B, X, C e W formam uma quadrupla harmonica, ou seja,
=
.
XC
WC
Pelo teorema de Menelaus temos que
entao

W C BZ AY

= 1.
W B ZA Y B
Portanto,
XC BZ AY

= 1, e
BX ZA Y B
AX, BY e CZ s
ao concorrentes.
Teorema 4. (Circunferencia de Apol
onio) Sejam A e B dois pontos fixos e k 6= 1 e uma
AP
= k e uma circunconstante real. O lugar geometrico dos pontos P que satisfazem
PB
ferencia conhecida como circunfer
encia de Apol
onio.
Demonstrac
ao. Primeiro observe que sobre a reta AB existem exatamente dois pontos
do lugar geometrico. Por A e B trace segmentos paralelos M N e BQ com BQ = 1 e
M A = AN = k. Sejam R e S as interseccoes de QN e QM com AB, respectivamente.

POT 2012 - Geometria - Nvel 3 - Aula 15 - Prof. Ccero Thiago

Q
b

R
b

AR
AN
k
=
=
= k. Alem
RB
BQ
1
AM
k
AS
=
=
= k.
disso, tambem s
ao semelhantes os tri
angulos ASM e BSQ entao
SB
BQ
1
Portanto, R e S pertencem ao lugar geometrico. Suponha que P e um ponto do lugar
AP
AR
geometrico, ent
ao
=k=
, portanto P R e a bissetriz interna do angulo AP B e
PB
RB
AS
AP
=k=
ent
ao P S e a bissetriz externa do mesmo angulo, isto garante que P
como
PB
SB
est
a sobre a circunferencia de di
ametro RS (Porque o angulo RP S = 90 ?).

Como os tri
angulos AN R e BQR s
ao semelhantes entao

P
b

Reciprocamente, considere um ponto P sobre a circunferencia de di


ametro RS. Demonstraremos que P e um ponto do lugar geometrico. Pelo ponto B trace paralelas BE e BF
`as retas P R e P S de tal forma que intersectem AP em E e F , respectivamente. Como os
tri
angulos ARP e ABE s
ao semelhantes com lados paralelos, pelo teorema de Tales temos
AR
AP
=
= k. Pelo mesmo motivo os tri
angulos semelhantes ABF e ASP garantem
que
PE
RB
AS
AP
=
= k. Assim, P E = P F , ou seja, P e o ponto medio do segmento EF e
que
PF
SB
AP
AP
=
= k,
como o tri
angulo EBF e ret
angulo temos que P B = P E = P F . Portanto,
PB
PE
4

POT 2012 - Geometria - Nvel 3 - Aula 15 - Prof. Ccero Thiago

com isso podemos afirmar que P pertence ao lugar geometrico.

P
b

Teorema 5. Seja A, C, B e D uma quadrupla harmonica. Seja O um ponto que n


ao
pertence `a reta AB. Se uma paralela pelo ponto B `a OA intersecta OC e OD em pontos
P e Q entao P B = P Q.
Demonstrac
ao.

O
b

Q
b

C
b

Temos que os tri


angulos OAC e P CB s
ao semelhantes entao
AO
AC
=
.
PB
CB
Alem disso, os tri
angulos OAD e BQD s
ao semelhantes entao
AD
AO
=
.
BQ
BD
AC
AD
Como A, C, B e D e uma quadrupla harmonica entao
=
. Portanto, e facil conCB
DB
cluir que, P B = BQ.
Teorema 6. Sejam A, C, B e D quatros pontos colineares e seja O um ponto que n
ao pertence `a reta AB. Se a paralela `
a OA passando por B intersecta OC e OD em pontos P e Q,
5

POT 2012 - Geometria - Nvel 3 - Aula 15 - Prof. Ccero Thiago

respectivamente, tais que P B = BQ entao A, C, B e D formam uma quadrupla harmonica.


Demonstrac
ao.
O
b

Q
b

C
b

B
b

Temos que os tri


angulos OAC e P CB s
ao semelhantes entao
AC
AO
=
.
PB
CB
Alem disso, os tri
angulos OAD e BQD s
ao semelhantes entao
AO
AD
=
.
BQ
BD
AD
AC
=
. Portanto, e facil concluir que, A, C, B e D e uma
Como P B = BQ, ent
ao
CB
DB
quadrupla harmonica.
Teorema 7. Seja A, C, B e D uma quadrupla harmonica e um ponto O que n
ao pertence
`a reta AB. Sejam M , R, N e S as interseccoes de OA, OC, OB e OD, respectivamente,
com uma reta arbitraria. Ent
ao, M , R, N e S formam uma quadrupla harmonica.
Demonstrac
ao. Pelos pontos B e N trace paralelas `a OA que intersectam OC e OD,
facil ver que
respectivamente, nos pontos P , Q e L, T . E
PB
OB
BQ
=
=
.
LN
ON
NT
Como A, C, B e D formam uma quadrupla harmonica entao, pelo teorema 5, que P B =
BQ. Com isso, LN = N T e, portanto, M , R, N e S formam uma quadrupla harmonica
pelo teorema 6.

POT 2012 - Geometria - Nvel 3 - Aula 15 - Prof. Ccero Thiago

T
b

b
b

R
b

L
b

Problema 1. Seja ABCD um quadril


atero inscrito em uma circunferencia tal que AB = AD
e AC e um di
ametro. Seja P um ponto sobre o arco BC que n
ao contem A e D. Se P A
e P D intersectam o lado BC nos pontos E e F , respectivamente, e BE = 3 e EF = 2,
determine F C.
Soluc
ao. Como AB = AD ent
ao BP A = AP D = . Como AP C = 90 entao

F P C = 90 = e QP C = 90 = . Portanto, P E e P C s
ao bissetrizes
interna e externa do
angulo BP F , respectivamente e, com isso, B, E, F e C formam um
quadrupla harmonica ent
ao
BC
3
FC + 5
BE
=
=
F C = 10.
EF
FC
2
FC

POT 2012 - Geometria - Nvel 3 - Aula 15 - Prof. Ccero Thiago

P
b

F
b

E
b

O
b

Problema 2. (Turquia) Seja ABC um tri


angulo com AB 6= AC. As bissetrizes interna e
externa relativas ao vertice A intersectam a reta BC em D e E, respectivamente. Se os pes
das perpendiculares baixadas de um ponto F do crculo de di
amentro DE sobre as retas
BC, CA e AB s
ao K, L e M , respectivamente, prove que KL = KM .
Soluc
ao. O crculo de di
ametro DE e um crculo de Apolonio do tri
angulo ABC relativo
ao vertice A ent
ao
AB
FB
=
. (1)
FC
AC
Pela lei dos senos temos que
sin C
AB
=
. (2)
AC
sin B
De (1) e (2) temos que
FB
sin C
=

FC
sin B
F B sin B = F C sin C. (3)
Como o crculo de di
ametro BF passa pelos pontos K e M , entao
KM = F B sin B.

POT 2012 - Geometria - Nvel 3 - Aula 15 - Prof. Ccero Thiago

De maneira similar, temos que


KL = F C sin C.
Portanto, de (3), temos que KL = KM .

A
b

L
b

b
b

b
b

K E

Exerccios propostos
1. Em um tri
angulo ABC, BD e BE s
ao as bissetrizes interna e externa, respectivamente. Se AD = 5 e DC = 3, determine a medida do segmento CE.
2. Seja ABC um tri
angulo e sejam D o pe da bissetriz relativa ao vertice A, I o incentro e Ia o ex - incentro oposto ao vertice A. Prove que A, I, D e Ia formam uma
quadrupla harmonica.
3. Em um tri
angulo n
ao equilatero, a reta que passa pelo baricentro e pelo incentro e
paralela a um dos lados do tri
angulo. Demonstre que os lados do tri
angulo est
ao em
progress
ao aritmetica.
4. (Shortlist IMO) Seja ABC um tri
angulo, e sejam D, E, F os pontos de tangencia do
crculo inscrito no tri
angulo ABC com os lados BC, CA e AB respectivamente. Seja
X um ponto no interior do tri
angulo ABC tal que o crculo inscrito no tri
angulo
XBC tangencia XB, XC e BC em Z, Y e D, respectivamente. Prove que EF ZY e
inscritvel.

POT 2012 - Geometria - Nvel 3 - Aula 15 - Prof. Ccero Thiago

5. (TST Jr Balkan - Romenia) Seja ABC um tri


angulo ret
agulo com A = 90 e D um
ponto sobre o lado BC. Sejam E o simetrico de A com relacao a BD e F o ponto de
intersecc
ao de CE com a perpendicular a BC por D. Prove que AF, DE e BC s
ao
concorrentes.
Bibliografia
1. Geometra - Teora y pr
actica.
Fernando Alva Gallegos
2. College Geometry - An introduction to the modern geometry of the triangle and the
circle.
Nathan Altshiller - Court
3. Geometra
Radmila Bulajich Manfrino e Jose Antonio G
omez Ortega
4. Harmonic Division and its Applications
Cosmin Pohoata

10

Polos Olmpicos de Treinamento


Curso de Geometria - Nvel 3

Aula

16

Prof. Ccero Thiago

Transformac
oes geom
etricas I: Translac
oes
Seja ~v um vetor no plano e um segmento de medida a. Seja A um ponto do plano e seja
A um ponto tal que o segmento AA tem a mesma direcao e sentido do vetor ~v e possui
comprimento a. Nesse caso dizemos que o ponto A e obtido a partir do ponto A na direcao
e sentido do vetor ~v com dist
ancia a. Seja F a imagem de uma figura F a partir de uma
translacao e sejam A e B dois pontos quaisquer de F tais que A e B s
ao, respectivamente,
suas imagens por uma translac
ao. Como AA k BB e AA = BB entao o quadril
atero

AA B B e uma paralelogramo e, consequentemente, AB k A B e AB = A B . Portanto, se


F e a imagem de F por uma translac
ao entao os segmentos correspondentes dessas figuras
s
ao iguais, paralelos e possuem a mesma direcao e sentido.
Problema 1. Sejam AB e CD dois segmentos tais que AB k DC e AB = DC. Prove que
existe uma translac
ao que leva AB em CD.
Soluc
ao. Como AB k DC e AB = DC entao ABCD e um paralelogramo e, com isso,
AC k BD e AC = BD. Portanto existe uma translacao que leva AB em DC.
Problema 2. Sejam M e N os pontos medios dos lados de um quadril
atero AD e BC de
um quadril
atero ABCD. Prove que se 2M N = AB + CD entao AB k CD.
Soluc
ao.
Sejam M B e M C as imagens de AB e CD, respectivamente, obtidas por translacoes. Os
quadril
ateros AM B B e DM C C s
ao paralelogramos e, portanto,
BB k AM e BB = AM,
CC k DM e CC = DM.
Como AM = M D ent
ao BB k CC e BB = CC . Alem disso, BN = N C, com isso,
BN B CN C .

POT 2012 - Geometria - Nvel 3 - Aula 16 - Prof. Ccero Thiago

Portanto, B, N e C s
ao colineares. Seja M1 o ponto sobre a reta M N tal que M N = N M1 .
Como N C = N B e M N = N M1 ent
ao M B M1 C e um paralelogramo com M C = B M1 .
Temos que 2M N = AB + CD M M1 = M B + B M1 o que e um absurdo pois contraria
a desigualdade triangular. Portanto B perntece ao segmento M M1 , ou seja,
M B k M N k M C .
Assim, AB k M N e DC k M N .

C
b

D
b

C
b

M1

M
b

Exerccios
1. Prove que uma sequencia de duas translacoes pode ser substituda por uma simples
translac
ao.
2. Seja ABCD um trapezio com BC k AD e seja M a intersecao das bissetrizes dos
angulos A e B. Alem disso, seja N a intersecao bas bissetrizes dos angulos C e
D. Prove que 2M N = |AB + CD BC AD|.
3. Seja ABCD um quadril
atero convexo com AD = BC. Sejam E e F os pontos medios
dos segmentos CD e AB, respectivamente. Seja H a intersecao de AD e F E e G a
intersec
ao de BC e F E. Prove que
AHF = BGF.
4. Seja P um ponto no interior de um ret
angulo ABCD tal que BP C +AP D = 180 .
Determine BCP + DAP .
5. Seja P um ponto no interior de um paralelogramo ABCD de area S. Prove que
AP CP + BP DP S.
2

POT 2012 - Geometria - Nvel 3 - Aula 16 - Prof. Ccero Thiago

Bibliografia
1. Problems in plane and solid geometry - V.1 - Plane Geometry
Viktor Prasolov
2. Geometric transformations I
I.M. Yaglom
3. Euclidean and transformational geometry - A deductive inquiry
Shlomo Libeskind
4. Geometric transformations I
Kin Y. Li
Mathematical Excalibur - Volume 13, n
umero 2 - Maio/Junho/2008
5. Mathematical Miniatures
Titu Andreescu e Svetoslav Savchev

Polos Olmpicos de Treinamento


Aula

Curso de Geometria - Nvel 3

17

Prof. Ccero Thiago

Tri
angulo Pedal
Seja P um ponto qualquer no plano do tri
angulo ABC e sejam L, M e N as projecoes de
P sobre as retas AB, BC e CA. O tri
angulo LM N ser
a chamado de tri
angulo Pedal.
Teorema 1. Seja P um ponto qualquer no plano do tri
angulo ABC e sejam L, M e N as
projecoes de P sobre as retas AB, BC e CA. Entao, LN = AP sin A, LM = BP sin B
e M N = CP sin C.
Demonstrac
ao.

A
b

Aplicando a lei dos senos no tri


angulo ALN temos que
LN
= AP LN = AP sin A.
sin A
Analogamente, LM = BP sin B e M N = CP sin C.

POT 2012 - Geometria - Nvel 3 - Aula 17 - Prof. Ccero Thiago

Teorema 2. Seja ABC um tri


angulo e C(O, R) sua circunferencia circunscrita. Seja M
um ponto no plano do tri
angulo. Sejam A1 , B1 e C1 as projecoes de M sobre os lados do
tri
angulo. Ent
ao

|R2 OM 2 |
Area(A
1 B1 C1 )
.
=

4R2
Area(ABC)
Demonstrac
ao.
B
b

A1
b

X
b

C1
Z

M
b

b
b

B1

Temos que B1 C1 = AM sin A, A1 C1 = BM sin B e B1 C1 = CM sin C. Segue que


B1 C1
AM A1 C1
BM A1 B1
CM
=
,
=
,
=
.
BC
2R
AC
2R
AB
2R
Sejam X, Y e Z s
ao as intersec
oes de AM , BM e CM , respectivamente, com C(O, R).
Entao
A1 B1 C1 = A1 B1 M + M B1 C1 = A1 CM + M AC1
= ZY B + BY X = ZY X.
Analogamente, B1 C1 A1 = Y ZX e B1 A1 C1 = Y XZ.
A1 B1 C1 e XY Z s
ao semelhantes e
RA1 B1 C1
A1 B1
=
.
XY
R
2

Portanto, os tri
angulos

POT 2012 - Geometria - Nvel 3 - Aula 17 - Prof. Ccero Thiago

Como os tri
angulos M AB e M Y X s
ao semelhantes temos que
MX
XY
=
.
AB
MB
Combinando os resultados obtidos temos

R
Area(A
A1 B1 B1 C1 A1 C1
1 B1 C1 )
=

RA1 B1 C1
AB BC AC
Area(ABC)
=
=

MX MA MB

M B 2R
2R

|R2 OM 2 |
MA MX
=
.
4R2
4R2

Problema 1. Prove que os lados do tri


angulo pedal de qualquer ponto interior de um
tri
angulo equilatero s
ao proporcionais `
as dist
ancias de P aos vertices correspondentes.
Soluc
ao.
A
b

Z
b

Y
P

Pelo teorema 1 temos que ZY = P A sin A = P A sin 60 . Analogamente, XZ = P B sin 60


ZY
XZ
XY
e XY = P C sin 60 . Portanto,
=
=
= sin 60 .
PA
PB
PC
Problema 2. (Romenia) Seja P um ponto no interior do tri
angulo ABC. Prove que
PA

sin A
sin B
sin C
= PB
= PC
.
sin(BP C A)
sin(CP A B)
sin(AP B C)

Soluc
ao.

POT 2012 - Geometria - Nvel 3 - Aula 17 - Prof. Ccero Thiago

A
b

b
b

Z
b

P
b

b
b

Temos que ZY = P A sin A, ZX = P A sin B e XY = P C sin C. Alem disso,


ZXY = ZXP + Y XP = ZBS + Y CP = ACS + Y CP = P CS. Dessa forma,
BP C = P CS + P SC = ZXY + A ZXY = BP C A.
Aplicando a lei dos senos no tri
angulo XY Z obtemos imediatamente o resultado desejado.
Problema 3. Considere um tri
angulo ABC. Sejam AD e AE as bissetrizes interna e externa, respectivamente, do
angulo A. Seja F um ponto sobre o crculo circunscrito do
AED e K, L e M os pes das perpendiculares de F a BC, AB e AC, respectivamente.
Prove que KL = KM .
Problema 4. Seja ABC um tri
angulo inscrito em um crculo de raio R e seja P um ponto
no interior do tri
angulo. Prove que
PB
PC
1
PA
+
+
.
2
2
2
BC
CA
AB
R
Problema 5. Seja P um ponto no interior do tri
angulo ABC. Se pa , pb e pc s
ao as dist
ancias
de P aos lados do tri
angulo ABC, prove que
P A + P B + P C 2(pa + pb + pc ),
4

POT 2012 - Geometria - Nvel 3 - Aula 17 - Prof. Ccero Thiago

com igualdade acontecendo se, e somente se, o tri


angulo ABC e equilatero e P e o circuncentro. (Desigualdade de Erdos - Mordell)
Problema 6. Seja P um ponto no interior do tri
angulo ABC. Prove que

aP A + bP B + cP C 4 Area(ABC).
Problema 7. (Balkan) Seja ABC um tri
angulo acut
angulo. Sejam A1 , B1 e C1 as projecoes
do baricentro G sobre os lados do tri
angulo. Prove que

2
Area(A
1
1 B1 C1 )

9
4
Area(ABC)
Problema 8. (IMO) Seja P um ponto no interior de tri
angulo ABC. Prove que pelo menos
um dos angulos P AC, P BC e P CA e menor ou igual a 30 .
Bibliografia
1. Mathematical Reflections - the first two years
Titu Andreescu
2. Notas de aula do professor Marcelo Mendes de Oliveira.
3. Inequalities
Radmila Bulajich, Jose Antonio G
omez Ortega e Rogelio Valdez Delgado

Polos Olmpicos de Treinamento


Aula

Curso de Geometria - Nvel 3

18

Prof. Ccero Thiago

Transformac
oes geom
etricas II - Simetria e rotac
ao.
1. Simetria com rela
c
ao a um ponto.
Dizemos que o ponto A e o simetrico de A com relacao a um ponto fixo O se, e somente
se, o ponto O for o ponto medio do segmento AA .

b
b

Sejam F e F duas figuras simetricas com relacao ao ponto O e sejam AB e A B os seg facil ver que ABA B e um paralelogramo
mentos correspondentes nessas duas figuras. E
pois as diagonais cortam - se em seus pontos medios.
2. Simetria com rela
c
ao a uma reta.
Dizemos que o ponto A e o simetrico de A com relacao `a reta r se, e somente se, a reta r
e a mediatriz do segmento AA .

A
b

M
b

POT 2012 - Geometria - Nvel 3 - Aula 18 - Prof. Ccero Thiago

Problema 1. (Problema de Fagnano) Determine o tri


angulo de permetro mnimo inscrito
em um tri
angulo acut
angulo.
Soluc
ao. Seja ABC um tri
angulo acut
angulo. Queremos achar pontos M , N e P sobre
os lados BC, CA e AB, respectivamente, tais que o permetro do tri
angulo M N P seja
mnimo. Inicialmente vamos considerar uma vers
ao mais simples do problema. Fixe o
ponto P sobre o lado AB. Vamos achar os pontos M e N sobre BC e CA, respectivamente, tais que o tri
angulo M N P tenha permetro mnimo. (O mnimo ir
a depender da

escolha do ponto P .) Seja P o simetrico de P com relacao `a reta BC e P o simetrico de P


com relacao `
a reta AC. Ent
ao CP = CP = CP , P C B = P CB e P CA = P CA.

Se BCA = , ent
ao P CP = 2. Mas, 2 < 180 , pois < 90 . Consequetemente, a
reta P P intersecta os lados BC e AC do tri
angulo ABC nos pontos M e N , respectivamente, e o permetro do tri
angulo M N P e igual `a P P . De maneira semelhante, se X e
um ponto sobre BC e Y um ponto sobre AC, o permetro do tri
angulo XP Y e igual ao
comprimento da poligonal P XY P , que e maior ou igual a P P . Como isso, o permetro
do tri
angulo P XY e maior ou igual ao permetro do tri
angulo P M N e a igualdade ocorre
se, e somente se, X = M e Y = N . Agora, precisamos encontrar o ponto P sobre o lado
AB tal que o segmento P P tenha comprimento mnimo. Veja que P P e a base do
tri
angulo is
osceles P P C com
angulo P CP = 2 constante e lados CP = CP = CP .
Entao, basta escolher P sobre AB tal que CP = CP seja mnimo, mas isso acontece
quando P e o pe da altura relativa ao vertice C. Se P e o pe da altura relativa ao vertice
C entao M e N ser
ao os pes das outras duas alturas. Para provar isso, sejam M1 e N1
os pes das alturas do tri
angulo ABC relativas aos vertices A e B, respectivamente. Entao

BM1 P = BM1 P = BAC = CM1 N1 , mostrando que o ponto P est


a sobre a reta
M1 N1 . De maneira an
aloga, P est
a sobre a reta M1 N1 e, portanto, M = M1 e N = N1 .
Dessa forma, de todos os tri
angulos inscritos no tri
angulo ABC o tri
angulo ortico e o que
tem o menor permetro.
b

Y
b

b
b

POT 2012 - Geometria - Nvel 3 - Aula 18 - Prof. Ccero Thiago

P
b

C
b

N1
b

M1
b

Problema 2. (Cone Sul) Seja ABC um tri


angulo e sejam AN , BM e CP as alturas relativas aos lados BC, CA e AB, respectivamente. Sejam R, S as projecoes de N sobre os
lados AB, BC, respectivamente, e Q, W as projecoes de N sobre as alturas BM e CP ,
respectivamente. (a) Prove que R, Q, W , S s
ao colineares;
(b) Prove que M P = RS QW .
3. Rota
c
ao.
Seja O um ponto fixo de um plano orientado e um angulo dizemos que P e a imagem
de P por uma rotac
ao de centro O e
angulo se OP = OP e P OP = .
b

Teorema 1. Seja O um ponto fixo de um plano orientado e um angulo . Alem disso,


sejam P e Q pontos do plano distintos de O e suas imagens P e Q pela rotacao de centro
O e angulo . Ent
ao P Q = P Q e a medida do angulo formado pelas retas P Q e P Q e
.
Demonstrac
ao.
facil ver que os tri
E
angulos OP Q e OP Q s
ao congruentes pois P OQ = P OP
QOP = QOQ QOP = P OQ , OP = OP e OQ = OQ . Prolongue P Q ate
3

POT 2012 - Geometria - Nvel 3 - Aula 18 - Prof. Ccero Thiago

intersectar P Q no ponto M . O quadril


atero M QQ O e inscritvel pois P QO = P Q O

e, com isso, QM Q = QOQ = .

M
b

P
b

O
b

Q
b

Problema 3. (OBM) Na figura, o quadrado A B C D foi obtido a partir de uma rotacao


no sentido hor
ario do quadrado ABCD de 25 graus em torno do ponto medio de AB. Qual
e o angulo agudo, em graus, entre as retas AC e B D ?
(a) 5

(b) 25

(c) 45

(d) 65

(e) 85

D
b

A
b

B
b

Soluc
ao.
4

POT 2012 - Geometria - Nvel 3 - Aula 18 - Prof. Ccero Thiago

D
C

F
b

A
b

Pelo teorema 1 temos que o


angulo agudo entre AC e A C e CEC = 25 . Como A C e
B D s
ao as diagonais do quadrado entao o angulo entre eles e D OA = 90 . Portanto,
o angulo desejado ser
a EF O = 65 .
Problema 4. (Ponto de Fermat) Seja ABC um tri
angulo acut
angulo. Encontrar o ponto
interior que minimiza a soma AP + BP + CP .
Problema 5. (USAMO) Dois tri
angulos ABC e P QR como mostra a figura abaixo s
ao tais

que ADB = BDC = CDA = 120 . Prove que X = u + v + w.


b

A
b

u
b
b

w
b

c
b

b
v
a

POT 2012 - Geometria - Nvel 3 - Aula 18 - Prof. Ccero Thiago

Problema 6. Seja P um ponto no interior do quadrado ABCD. Prove que as perpendiculares baixadas desde A, B, C e D sobre P B, P C, P D e P A, respectivamente, s
ao
concorrentes.
Problema 7. (Cone Sul) Seja ABC um tri
angulo is
osceles com AB = AC. Uma reta l
passando pelo incentro I de ABC intersecta AB e AC em D e E, respectivamente. F e
G s
ao pontos sobre BC tais que BF = CE e CG = BD. Mostre que o angulo F IG e
constante.
Problema 8. Prove que composic
ao de duas rotacoes de angulos e , respectivamente, e
uma rotacao de
angulo de medida + . Se os centros das duas rotacoes forem diferentes
determine o centro da nova rotac
ao.
Problema 9. (Teorema de Napole
ao) Seja ABC um tri
angulo escaleno. Se exteriormente
s
ao construdos tri
angulos equilateros ABM , BCN e ACP , prove que os baricentros desses
tri
angulos s
ao vertices de um tri
angulo equilatero.

Programa Olmpico de Treinamento


Aula

Curso de Geometria - Nvel 3

Prof. Rodrigo Pinheiro

Problemas OBM - 1 Fase

Problema 1. A figura a seguir representa um Tangram, quebra-cabecas chines formado por


5 tri
angulos, 1 paralelogramo e 1 quadrado. Sabendo que a area do Tangram a seguir e
64cm2 , qual e a
area, em cm2 , da regi
ao sombreada?

a) 7.6
b) 8
c) 10.6
d) 12
e) 21.3
Problema 2. Na figura a seguir, ABC e um tri
angulo qualquer e ACD e AEB s
ao tri
angulos
e:
equil
ateros. Se F e G s
ao os pontos medios de EA e AC, respectivamente, a razao BD
FG
b

E
b

F
b

a)

1
2

POT 2012 - Geometria - Nvel 3 - Aula 5 - Prof. Rodrigo Pinheiro

b) 1
c)

3
2

d) 2
e) Depende das medidas dos lados de ABC.
Problema 3. Na figura, o lado AB do tri
angulo equil
atero ABC e paralelo ao lado DG do
quadrado DEF G. Qual e o valor do
angulo x?
a) 80
b) 90
c) 100
d) 110
e) 120

Problema 4. O jardim da casa de Maria e formado por cinco quadrados de igual area e
tem a forma da figura abaixo. Se AB = 10m, entao a area do jardim em metros quadrados
e:
a) 200

b) 10 5
c) 100
d)

500
3

e)

100
3

POT 2012 - Geometria - Nvel 3 - Aula 5 - Prof. Rodrigo Pinheiro

Problema 5. A figura abaixo e formada por tres quadrados de lado 1cm e um ret
angulo
que os contorna. A
area do ret
angulo e:

a) 3 2

b) 4 2
c) 6

d) 6 2
e) 8
Problema 6. Qual o menor
permetro inteiro possvel de um tri
angulo que possui um dos
3
lados com medida igual a 5. 2 ?
a) 8
b) 9
c) 10
d) 11
e) 12
Problema 7. Uma mesa de bilhar tem dimens
oes de 3m por 6m e tem cacapas nos seus
quatro cantos P , Q, R e S. Quando uma bola atinge na borda da mesa, sua trajet
oria
forma um angulo igual ao que o
angulo anterior formava. Uma bola inicialmente a 1 metro
da cacapa P , e batida do lado SP em direcao ao lado P Q, como mostra a figura. A quantos
metros de P a bola acerta o lado P Q se a bola cai na cacapa S ap
os duas batidas na borda
da mesa?
a) 1
b)

6
7

c)

3
4

d)

2
3

e)

3
5

POT 2012 - Geometria - Nvel 3 - Aula 5 - Prof. Rodrigo Pinheiro

Problema 8. No desenho temos AE = BE = CE = CD. Alem disso, e s


ao medidas

de angulos. Qual o valor da razao


a)

3
5

b)

4
5

c) 1
d)

5
4

e)

5
3

Problema 9. Em um tri
angulo ABC, A = 20 e B = 110 . Se I e o incentro (centro
da circunferencia inscrita) e O o circuncentro (centro da circunferencia circunscrita) do
tri
angulo ABC, qual a medida do
angulo IAO?
a) 20
b) 25
c) 30
d) 40
e) 35
Problema 10. Na figura abaixo, os pontos A, B e C s
ao colineares, assim como os pontos
D, E e F . As duas retas ABC e DEF s
ao paralelas. Sendo A1 , A2 e A3 as areas das
regi
oes destacadas na figura, podemos afirmar que:
a) A2 = 2A1 = 2A3
4

POT 2012 - Geometria - Nvel 3 - Aula 5 - Prof. Rodrigo Pinheiro

b) A2 = A1 + A3
c) A2 > A1 + A3
d) A2 < A1 + A3
e) A22 = A1 .A3

Problema 11. No tri


angulo ABC, m(ABC) = 140 . Sendo M o ponto medio de BC, N o
ponto medio de AB e P o ponto sobre o lado AC tal que M P e perpendicular a AC, qual
a medida do
angulo N M P ?
a) 40
b) 50
c) 70
d) 90
e) 100
Problema 12. No desenho, o ret
angulo cinza tem seus vertices sobre os lados do tri
angulo
equil
atero de
area 40cm2 . O menor lado do ret
angulo e um quarto do lado do tri
angulo. A
area do ret
angulo em cm2 e:
a) 5
b) 10
c) 15
d) 18
e) 22

POT 2012 - Geometria - Nvel 3 - Aula 5 - Prof. Rodrigo Pinheiro

Problema 13. Os pontos P , Q, R, S e T s


ao vertices de um polgono regular. Os lados
P Q e T S s
ao prolongados ate se encontrarem em X, como mostra a figura, e QXS mede
140 . Quantos lados os polgono tem?
a) 9
b) 18
c) 24
d) 27
e) 40

Problema 14. Uma figura no formato da cruz, formada por quadrados de lado 1, esta
inscrita em um quadrado maior, cujo os lados s
ao paralelos aos lados do quadrado tracejado,
cujos vertices s
ao vertices da cruz. Qual e a area do quadrado maior?
a) 9
b)

49
5

c) 10
d)

81
8

e)

32
3

Problema 15. Na figura, BC = 2BH. Quanto vale x?


a) 10
b) 15
c) 16
6

POT 2012 - Geometria - Nvel 3 - Aula 5 - Prof. Rodrigo Pinheiro

d) 20
e) 25

Problema 16. Em um tri


angulo ABC com ABC BAC = 50 , a bissetriz do angulo
ACB intersecta o lado AB em D. Seja E o ponto do lado AC tal que CDE = 90 . A
medida do angulo ADE e:
a) 25
b) 30
c) 40
d) 45
e) 50
Problema 17. Seja XOY = 90 . Sejam M e N os pontos medios de OX e OY , respectivamente. Dado que XN = 19 e Y M = 22, determine a medida do segmento XY .
a) 24
b) 26
c) 28
d) 30
e) 32
Problema 18. Uma circunferencia passando pelos vertices B, A, D do paralelogramo

ABCD encontra o segmento CD em Q. Sabendo que m(B AD)


= 60 e AD = 10, o
tamanho do segmento CQ e:
a) 10
b) 20
c)

5 3
2

POT 2012 - Geometria - Nvel 3 - Aula 5 - Prof. Rodrigo Pinheiro

d) 10 3
e) 15
Problema 19. Seja AB um segmento de comprimento 26 e sejam C e D pontos sobre o
segmento AB tais que AC = 1 e AD = 8. Sejam E e F pontos sobre uma semicircunferencia
de di
ametro AB, sendo EC e F D perpendiculares a AB. Quanto mede o segmento EF ?
a) 5

b) 5 2
c) 7

d) 7 2
e) 12
Problema 20. As alturas de um tri
angulo medem 12, 15 e 20. O maior angulo interno do
tri
angulo mede:
a) 72
b) 75
c) 90
d) 108
e) 120
Problema 21. Constr
oi-se o quadrado ABXY sobre o lado AB do hept
agono regular
ABCDEF G, exteriormente ao hept
agono. Determine a medida do angulo BXC, em
radianos.
a)

b)

3
7

c)

14

d)

3
14

e)

3
28

Gabarito

POT 2012 - Geometria - Nvel 3 - Aula 5 - Prof. Rodrigo Pinheiro

1) D
2) D
3) E
4) C
5) C
6) B
7) B
8) D
9) C
10) A
11) D
12) C
13) D
14) B
15) D
16) A
17) B
18) A
19) D
20) C
21) E

S-ar putea să vă placă și